Você está na página 1de 163

5

OBIECTIVE DE REFERIN I
EXEMPLE DE ACTIVITI DE NVAARE

1. Cunoaterea i nelegerea conceptelor, a terminologiei i a procedurilor de
calcul
Obiective de referin
La sfritul clasei a VI-a elevul va fi
capabil :
1.1.s utilizeze noiuni de logica
1.2.s foloseasc metode i principii
adecvate n rezolvarea unor probleme










1.3.s foloseasc diferite metode de
rezolvare a ecuaiilor i inecuaiilor i s
utilizeze ecuaii i inecuaii pentru
rezolvarea problemelor


1.4.s aplice criterii, proprieti i noiuni
de divizibilitate n demonstraii





1.5.s efectueze calcule cu numere ntregi
i raionale pozitive


1.6.s utilizeze matematica n rezolvarea
problemelor puse la alte discipline
Exemple de activiti de nvare
Pe parcursul clasei a VI-a se recomand
urmtoarele activiti :
-rezolvarea unor probleme de logica
-rezolvarea unor probleme folosind
principiul paritii sau partiia n clase
- folosirea metodei reducerii la absurd n
anumite demonstraii
-probleme care se rezolv folosind principiul
lui Diriclet
-probleme care se rezolv folosind principiul
invariantului
-rezolvarea de probleme folosind regula de
trei simpl sau regul de trei compus
-probleme de ordonare prin comparare
-probleme de numrare
-rezolvarea unor ecuaii i inecuaii dificile
folosind diverse tehnici
-rezolvarea unor probleme cu ajutorul
ecuaiilor
-formularea unor probleme pornind de la o
ecuaie
-exerciii de recunoatere a unor numere
naturale sau expresii divizibile cu alte
numere date folosind criteriile de
divizibilitate sau descompunerile n factori
-exerciii de calcul a numrului de divizori a
unui numr folosind descompunerea n
produs de puteri de numere prime
-calculul unor sume folosind diverse tehnici
-exerciii de determinare a valorii unor
expresii
-calculul unor probabiliti
- rezolvarea unor probleme de micare care
i au originea n fizic folosind relaiile ce
se stabilesc ntre mrimi : proporionalitate
direct sau invers
-probleme cu coninut practic care se rezolv
folosind reducerea la scar

6



1.7.s foloseasc metode specifice n
rezolvarea problemelor de geometrie





1.8.s recunoasc i s utilizeze
proprietile figurilor geometrice n
demonstraii

-probleme de numrare i interpretarea lor
folosind noiunea de probabilitate
-rezolvarea problemelor cu procente
-rezolvarea problemelor de coliniaritate i
concuren
-probleme de construcii geometrice
-rezolvarea de probleme folosind metoda
triunghiurilor congruente
-folosirea criteriilor de paralelism n
rezolvarea unor probleme
-folosirea n demonstraii a proprietilor
triunghiului isoscel i a triunghiului
echilateral


2.Dezvoltarea capacitii de a emite judecai de valoare pentru rezolvarea
problemelor inventiv i euristic-creative

Obiective de referin
La sfritul clasei a VI-a elevul va fi capabil

2.1.s analizeze, s elaboreze strategii de
rezolvare i s rezolve probleme dificile




2.2.s formuleze probleme pornind de la un
model sau enun parial



2.3.s gseasc metode de lucru valabile pentru
clase de probleme
Exemple de activiti de nvare
Pe parcursul clasei a VI-a se
recomand urmtoarele activiti :
-analizarea problemei n scopul
nelegerii ei
-elaborarea unui plan de rezolvare i
rezolvarea problemei
-verificarea rezultatului obinut i
analiza rezolvrii
-formularea unor concluzii pornind de
la o ipoteza data
-deducerea unor condiii necesare i
suficiente pentru demonstrarea unei
concluzii
-identificarea unor algoritmi de
rezolvare valabili pentru clase de
probleme
-analizarea mai multor metode de
rezolvare i alegerea celei mai eficiente




7
3.Dezvoltarea capacitii de a face conexiuni cognitive n cadrul disciplinei i a
ariei curriculare

Obiective de referin
La sfritul clasei a VI-a elevul va fi
capabil :
3.1.s utilizeze raionamente inductive
n rezolvarea problemelor dificile din
domeniile studiate

3.2.s-i nsueasc o gndire flexibil
i abstract specific matematicii

Exemple de activiti de nvare
Pe parcursul clasei a VI-a se recomand
urmtoarele activiti :
-exerciii i probleme n rezolvarea crora se
folosesc diferite raionamente
-folosirea soluiilor unei probleme pentru
rezolvarea altora din aceeai sfer cognitiv
-probleme din algebr care se rezolv cu
metode geometrice sau probleme de geometrie
care se rezolv algebric


4.Dezvoltarea capacitii de a comunica utiliznd limbajul matematic

Obiective de referin
La sfritul clasei a VI-a elevul va fi
capabil :

4.1.s diferenieze informaiile matematice
dintr-un enun dup natura lor


4.2.s formuleze reciproce ale unor
propoziii i s studieze valoarea lor de
adevr
Exemple de activiti de nvare
Pe parcursul clasei a VI-a se recomanda
urmatoarele activitat

-sesizarea informaiilor cu caracter general
dintr-o ipoteza
-redactarea demonstraiilor sau rezolvrilor
utiliznd terminologia adecvat
-formularea de propoziii reciproce,
analizarea lor i stabilirea valorii lor de
adevr


5.Dezvoltarea interesului i a motivaiei pentru studiul i aplicarea matematicii n
contexte variate

Obiective de referin
La sfritul clasei a VI-a elevul va fi
capabil :

5.1.s sesizeze importanta aplicrii
noiunilor de matematica n probleme cu
coninut practic
5.2.s manifeste originalitate n abordarea
unor metode alternative de rezolvare
5.3 s manifeste interes pentru folosirea
tehnologiilor informaiei n studiul
matematicii
Exemple de activiti de nvare
Pe parcursul clasei a VI-a se recomand
urmtoarele activiti

-argumentarea prin exemplificare


-utilizarea unor metode variate n rezolvarea
unei probleme
-utilizarea unor soft-uri pentru nvarea
matematicii

8
CONINUTURI
ALGEBR
1. Sume
2. Divizibilitatea n mulimea numerelor naturale
3. Cteva principii i metode de rezolvare a problemelor de matematic
3.1. Principiul paritii
3.2. Probleme de numrare
3.3. Principiul lui Dirichlet
3.4. Principiul invariantului
3.5. Probleme de logic
3.6. Probleme de ordonare
3.7. Metoda reducerii la absurd
4. Rapoarte i proporii
4.1. Scara unui plan
4.2. Scara unei hri
4.3. Probabiliti
4.4. Procente
4.5. Titlul unui aliaj
4.6. Proporii
4.7. ir de rapoarte egale
4.8. Proporionalitate direct. Proporionalitate invers
4.9. Regula de trei simpl. Regula de trei compus
5. Numere ntregi
5.1. Divizibilitate n mulimea numerelor ntregi
5.2. Determinarea valorii unei expresii ce depinde de un exponent natural
5.3. Ecuaii i inecuaii
GEOMETRIE
1. Segmente
2. Unghiuri
3. Geometria bazata pe raionament i demonstraie
3.1. Cazurile de congruen ale triunghiurilor
3.2. Metoda triunghiurilor congruente
3.3. Proprietile triunghiului isoscel i echilateral
3.4. Paralelism
3.5. Patrulatere
3.6. Concurena liniilor importante n triunghi
3.7. Probleme de coliniaritate
3.8. Probleme de concuren
3.9. Construcii geometrice

11
1. Calculul unor sume de numere

n multe probleme elevii aplic n rezolvarea lor calculul unor sume de numere
naturale consecutive, numere pare consecutive, numere impare consecutive, dar i
sume de numere raionale pozitive.
Parcurgnd aceast tem se face o pregtire pentru nelegerea ulterioar a
demonstraiei relative la calculul unor sume de numere folosind metoda induciei
matematice.

1.1.Introducerea simbolului sum i a proprietilor lui

n matematic pentru prescurtarea scrierii unor sume se folosete simbolul .
Prin

=
n
1 k
k
a nelegem sum de a
k
de la k =1 pn la k = n.
Prezentm n continuare cteva exemple de folosire a acestui simbol:
a) Suma primelor n numere naturale:
1 + 2 + 3 + .. + n se scrie

=
n
1 k
k
b) Suma ptratelor primelor numere naturale:
1
2
+ 2
2
+ 3
2
+ + n
2
se scrie

=
n
1 k
2
k
c) Suma cuburilor primelor n numere naturale se scrie:
1
3
+ 2
3
+ 3
3
+ .. + n
3
se scrie

=
n
1 k
3
k
Alte exemple de utilizare a simbolului sum:
d)

=
p
1 k
1 = 1 + 1 + .. + 1 = p
p termeni
e)

=
q
p i
(1)
2i
= 1 + 1 + 1 + .. + 1 = q p
de q p ori
f)

=
n
1 k
k
a = a
1
+ a
2
+ + a
n


1.2. Proprieti ale simbolului sum

1. Suma unei sume (diferene) este egal cu suma (diferena) sumelor:

=
n
1 k
(a
k
b
k
) =

=
n
1 k
a
k

=
n
1 k
b
k

12
2. Dac toi termenii sumei conin acelai factor el poate fi scos ca factor comun
n afara sumei:

=
n
1 k
a
k
=

=
n
1 k
a
k
,

Probleme rezolvate

R1.3.1.Suma primelor n numere naturale se calculeaz dup formula:

=
n
1 k
k =
2
) 1 n ( n +
(1)

Demonstraie:

=
n
1 k
k = 1 + 2 + 3 . + (n 1) + n

=
n
1 k
k = n + (n 1) + (n 2) + . + 2 + 1

2

=
n
1 k
k = (n + 1) + (n + 1) + . + (n + 1)
2

=
n
1 k
k = n(n + 1) : 2

=
n
1 k
k =
2
) 1 n ( n +

R1.3.2. Suma ptratelor primelor n numere naturale este dat de formula:

=
n
1 k
k
2
=
6
) 1 n 2 )( 1 n ( n + +
(2)
Demonstraie:
Calculm mai nti suma:

=
n
1 k
(2k 1) = 1 + 3 + 5 + . + 2n 1
innd seama de formula (1) i de proprietile simbolului sum avem:

=
n
1 k
(2k 1) =

=
n
1 k
2k +

=
n
1 k
(1) = 2

=
n
1 k
k +

=
n
1 k
(1) = 2 =
+
n
2
) 1 n ( n

= n(n + 1) n = n
2
+ n n = n
2
Aadar pentru orice k 1, avem:
1 + 3 + 5 + . + (2k 1) = k
2
(3)
+
13
Folosind relaia (3) avem:
1 = 1
2

1 + 3 = 2
2

1 + 3 + 5 = 3
2

.
1 + 3 + 5 + . + (2k 1) = k
2

.
1 + 3 + 5 + . + (2k 1) + . + (2n 1) = n
2


1n + 3(n 1) + 5(n 2) + . + (2k 1)(n k + 1) + ..+ (2n 1) =

=
n
1 k
k
2

Relaia precedent se poate scrie prescurtat astfel:

=
n
1 k
(2k 1)(n k + 1) =

=
n
1 k
k
2
(4)
innd seama de proprietile sumei relaia (4) se poate scrie:

=
n
1 k
(2k 1)(n k + 1) =

=
n
1 k
(2k 1)(n + 1) +

=
n
1 k
( 2k
2
+ k) =
= (n + 1)

=
n
1 k
(2k 1) 2

=
n
1 k
k
2
+

=
n
1 k
k = (n + 1) n
2
2

=
n
1 k
k
2
+
2
) 1 n ( n +

Deci (n + 1) n
2
2

=
n
1 k
k
2
+
2
) 1 n ( n +
=

=
n
1 k
k
2

3

=
n
1 k
k
2
= n
2
(n +1) +
2
) 1 n ( n +


3

=
n
1 k
k
2
=
2
) 1 n 2 )( 1 n ( n + +
: 3 de unde

=
n
1 k
k
2
=
6
) 1 n 2 )( 1 n ( n + +


R1.3.3. Numrul triunghiular este un numr de forma
2
) 1 n ( n +
, unde n este un
numr natural.
Denumirea este justificat pentru c aceste numere pot fi materializate n
triunghiuri dreptunghice alctuite din puncte. Se observ c numerele triunghiulare
se obin prin adunarea succesiv a numerelor din irul natural:
1; 1 + 2 = 3; 1 + 2 + 3 = 6; 1 + 2 + 3 + 4 = 10 ;; 1 + 2 + 3 + . + n =
2
) 1 n ( n +

14
Trebuie calculat suma:

=
n
1 k
2
) 1 k ( k +
=
2
1

=
n
1 k
k
2
+
2
1

=
n
1 k
k
innd seama de formulele (1) i (2), obinem:

=
n
1 k
2
) 1 k ( k +
=
2
1

6
) 1 n 2 )( 1 n ( n + +
+
2
1

2
) 1 n ( n +
=
=
2
) 1 n ( n +
=

+
+
2
1
6
1 n 2
2
) 1 n ( n +
=
+
6
4 n 2

=
2
) 1 n ( n +

6
) 2 n )( 1 n ( n
6
) 2 n ( 2 + +
=
+




Bibliografie
D. Constantinescu, Olimpiada de matematic clasele V-VIII, Ed. Teora 1999, pag 28-
52;pag 125-138
D. Andrica, V. Berinde, Al. Blaga, G.Both, O. Pop, Concursul Grigore Moisil Ed. I-XV,
Ed. Hub Press 22 Baia-Mare 2001, pag 39,45,78
D. Brnzei i colectivul: Matematica n concursurile colare, Ed. Paralela 45,
2000,2001,2002 pag 27-54,119-135(2000);pag 27-54,117-130(2001);pag 18-34,82-92(2002)
D. Brnzei, D. Zaharia, M. Zaharia : Aritmetic-Algebr-Geometrie, Ed.Paralela 45
2002, pag 5-21
Acad. N. Teodorescu coordonator Culegere de probleme pentru clasele V-VIII, SSM
1987, pag 53-68
Foaia matematic (Chiinu) 3/1996, pag 24-31

15
2 Divizibilitatea n mulimea numerelor naturale

Dintre toate operaiile aritmetice, cea mai capricioas este mprirea.
Ea dispune de proprieti speciale, de un caracter deosebit.
Toate particularitile mpririi au favorizat apariia unor noiuni ca: numere
prime, cel mai mare divizor comun, cel mai mic multiplu comun, criterii de
divizibilitate.
Dezvoltarea teoriei divizibilitii a dus treptat la o serioas extindere a ntregii
teorii a numerelor.
n multe probleme de determinare a unor numere naturale folosim noiunile
studiate la divizibilitatea numerelor.
Reamintim teorema mpririi cu rest i cele mai importante noiuni ale
divizibilitii numerelor.

2.1. Teorema mpririi cu rest

Pentru oricare dou numere naturale a i b cu b 0, exist i sunt unice dou
numere naturale q i r astfel nct a = bq + r i r < b.
a dempritul
b mpritorul
q ctul mpririi
r restul mpririi
Proprietatea 2.1.1. Dac adugm lui a un multiplu a lui b, restul mpririi nu se
schimb.
Fie a = bq + r | + mb
a + mb = bq + r + mb = b(q + m) + r = b q
1
+ r
Proprietatea 2.1.2. Dac nmulim dempritul i mpritorul cu un numr,
restul se nmulete cu acel numr.
Din a = bq + r | m, obinem
am = bqm + rm, unde r m < mb
Proprietatea 2.1.3. Dac numerele a i b se mpart cu un numr atunci i restul se
mparte cu acel numr.
Fie a = bq + r , r < b
Dac a = m a
1
i b = m b
1
, atunci avem a
1
m = b
1
m q + r | : m,
a
1
= b
1
q +
m
r

Proprietatea 2.1.4. Dac dou numere dau acelai rest la mprirea cu un numr m,
diferena lor este divizibil cu m.
Din a = m q
1
+ r i b = m q
2
+ r deducem a = m q
1
+ r -
b = m q
2
+ r
a b = m (q
1
-q
2
)


16
2.2. Divizibilitatea n N

Definiia 2.2.1. Numrul natural a este divizibil cu numrul natural b dac
exist numrul natural c astfel nct a = b c
Notm: a M b ( a se divide cu b )
b | a ( b divide pe a )
b este divizorul lui a
a este multiplul lui b
Obs. Numrul natural a este divizibil cu numrul natural b dac restul mpririi
lui a la b este zero.
Proprieti
Propoziia 2.2. 1. Dac a este divizor a lui b i c atunci este divizor i a lui b c.
Din a | b b = m
1
a
a | c c = m
2
a
nsumnd cele dou egaliti membru cu membru obinem:
b + c = m
1
a +m
2
a = a ( m
1
+m
2
)= m
3
a
Scznd cele dou egaliti, rezult c :
b c = m
1
a m
2
a = a( m
1
-m
2
) = a m
3
( b c)
Propoziia 2.2.2. Dac a este divizor a lui b i c , oricare ar fi numerele naturale x i y,
a va fi divizor i pentru bx + cy.
Din a | b b = m
1
a
a | c c = m
2
a
nmulim prima egalitate cu x i a doua cu y i obinem :
bx = m
1
a x
cy = m
2
a y
Adunm membru cu membru i obinem :
bx + cy = m
1
a x + m
2
a y = a ( m
1
x + m
2
y ) = m a a | ( bx +cy )
Propoziia 2.2.3. Dac a este divizor a lui b i b divizor a lui c atunci a este
divizor a lui c.
Din a | b b = m
1
a
b | c c = m
2
b
nlocuind n egalitatea a doua pe b obinem:
c = m
1
m
2
a = m a a | c
Proprietatea 2.2.4. Dac a | b i b | a atunci a = b .
Din a | b b = m
1
a
b | a a = m
2
b
Substituind n prima egalitate pe a obinem :
b = m
1
m
2
b | : b
1 = m
1
m
2
; m
1
, m
2
N m
1
= m
2
= 1 a = b

Definiia 2.2.2. N umrul natural p , p2 este prim dac se divide numai cu 1 i cu el
nsui.
1 i p se numesc divizorii mpirii.
17
Obs : 1
0
. Un numr care nu este prim se numete compus.
2
0
. Numrul 2 este singurul numr natural prim i par.
Propoziia 2.2.5.Cel mai mare divizor comun al numerelor naturale a i b este un
numr natural d , care :
divide pe a i b ;
este divizibil cu orice divizor a lui a i b.
Notm : c.m.m. d.c. sau ( a; b)
Obs : 1
0
. Dac ( a; b) = 1 , atunci numerele a i b se numesc prime ntre ele .
Propoziia 2.2.6. Cel mai mic multiplu comun al numerelor a i b este un numr
natural m , care :
este multiplu a lui a i b ;
orice alt multiplu a lui a i b se divide cu el .
Notaie : c.m.m.m. c. sau [a; b ]
Propoziia 2.2.7. Dac a i b sunt numere naturale atunci avem :
a b = (a ; b)[a; b]

2.3. Determinarea unor numere prime n condiii date

Probleme rezolvate

R2.3.1. Determinai numerele prime a i b tiind c 28 a + 21b =2030.
Soluie: 2030 M 2 21b M 2 ,dar 21 M 2
28a M 2
atunci, b M 2 i b este numr prim atunci b = 2.
nlocuim n egalitatea dat i obinem :
28a+212 = 2030
28a = 2030 42
28a =1988 |: 28
a = 71
Numerele sunt : a = 71 , b = 2 .
R2.3.2. S se gseasc numerele naturale p astfel nct numerele p , p
2
+ 4 ,
p
2
+ 6 s fie simultan prime .
Soluie :
( ) p numr natural prim el are una din formele: 5k , 5k + 1 , 5k + 2 , 5k + 3 ,
5k + 4.
Vom demonstra c p are forma 5k i cum p este prim rezult c p = 5 .
Fie p = 5k + 1 p
2
= (5k + 1 )
2
=M
5
+ 1
P
2
+4 =M
5
+1 + 4 =M
5
+ 5 = M
5
(p
2
+ 4) M 5
b) p = 5k + 2 p
2
= (5k +2 )
2
=M
5
+ 4 p
2
+6 = M
5
+ 10 = M
5

( p
2
+ 6 ) M 5
c) p = 5k + 3 p
2
= ( 5k + 3 )
2
= M
5
+ 9
p
2
+6 = M
5
+ 9+ 6 = M
5
+ 15 = M
5
(p
2
+ 6) M 5
d) p = 5k + 4 p
2
= ( 5k + 4 )
2
= M
5
+ 16 p
2
+4 =
18
= M
5
+ 16 + 4 = M
5
+ 20 =M
5
(p
2
+ 4 ) M 5
Din a), b) , c), d) rezlt c p este de forma p =5k i p numr prim atunci p =5 i p
2
+ 4 =
29 , p
2
+ 6 =31, deci sunt numere prime.
A doua soluie :
Ultima cifr a lui p poate fi 2 sau cifra impar : 1, 3, 5, 7, 9, atunci ptratul lui
va avea ultima cifr 4, 1, 9, 5 u( p
2
) = 4 u ( p
2
+ 6 ) = 0
( p
2
+ 6 )M 5 ; u ( p
2
) =1 u ( p
2
+ 4 ) = 5 ( p
2
+ 4 ) M 5 , u ( p
2
) = 9
u ( p
2
+ 6 ) =5 ( p
2
6 ) M 5 ; u ( p
2
) = 5 u ( p ) = 5 i p este primp=5.
R2.3.3. S se determine toate numerele naturale n i p pentru care numerele:
p , p+ 3
n
, p+ 3
n+1
. p + 3
n +2
. p+3
n +3
sunt prime.
Soluie:
Dac p este numr impar atunci numerele p + 3
n
, p + 3
n + 1
, p+ 3
n + 2
,
p + 3
n +3
sunt numere pare , deci nu sunt prime rezult c p este numr par i prim deci
p =2
Ultima cifr a puterilor consecutive a lui 3 poate fi : 1,3,7,9 atunci unul dintre
numerele p + 3
n
, p + 3
n+1
, p + 3
n +2
sau p+ 3
n +3
va avea ultima cifr 5 deci va fi divizibil
cu 5 i atunci nu va fi prim dect n cazul n care este egal cu 5.
A tunci : p + 3
n
=5 2 + 3
n
= 5 3
n
=3 n =1
p =2 ; p + 3
n
=5 ; p + 3
n+1
= 11 ; p +3
n+2
= 29 i p + 3
n +3
= 83 sunt numere
prime .
Dac p+ 3
n+1
= 5 3
n+1
= 3 n =0 , atunci avem :
p=2
p + 3
n
=3
p + 3
n+1
= 5
p + 3
n+2
= 11
p + 3
n+3
= 29 sunt numere prime .
Dac p + 3
n+2
= 5 3
n+2
= 3 imposibil.
Soluiile sunt : 1) p = 2 i 2) p = 2
n = 0 n = 1

2.4.Probleme care se rezolv folosind teorema mpririi cu rest, cel mai mare
divizor comun i cel mai mic multiplu comun

Probleme rezolvate

R2.4.1. Determinai cel mai mic numr natural care mprit la numerele
naturale a, b, c d resturile a k ; b k ; c k , k N* i k < min(a,b,c).
Soluie:
Fie n numrul cutat, atunci avem:
n = ac
1
+ a k n + k = a ( c
1
+ 1 ) = Ma
n = bc
2
+ b k + k n + k = b ( c
2
+ 1 ) = Mb
n = cc
3
+ c k n + k = c ( c
3
+ 1 ) = Mc
19
n + k este multiplu comun al numerelor a, b, c, i pentru c este cel mai mic rezult c
n + k =[a, b, c] n = [a,b,c] k.
n condiiile n care n
1
n n
2
vom determina multiplii comuni care
ndeplinesc condiia dat , apoi calculm numrul n.
Exemplu: Aflai cel mai mic numr natural care mprit pe rnd la
5,6,7,8, d resturile 4,5,6,7.
Soluie :
Fie n numrul , atunci:
n = c
1
5 + 4 n + 1 = 5 ( c
1
+ 1 ) = M
5

n = c
2
6 + 5 n + 1 = 6 ( c
2
+ 1 ) = M
6

n = c
3
7 + 6 +1 n + 1 = 7 ( c
3
+ 1 ) = M
7

n = c
4
8 + 7 n +1 = 8 ( c
4
+ 1 ) = M
8

n + 1 multiplu comun al numerelor 5,6,7,8 i pentru c este cel mai mic rezult c n + 1
=[5,6,7,8] n + 1 = 840 n = 839 .
n cazul n care se impune condiia ca n s fie cuprins spre exemplu ntre 800 i
2003 atunci n + 1 { 840; 2840; 3840 } problema avnd trei soluii distincte.
R2.4.2. Determinai cel mai mic numr natural care mprit la numerele naturale a, b,
c obinem de fiecare dat restul r , r < min (a,b,c) .
Soluie:
Fie n numrul care trebuie determinat :
n = a c
1
+ r n r = ac
1
= M
a

n = b c
2
+ r -r n r = bc
2
= M
b

n = c c
3
+ r n r =c c
3
= M
c

n r este multiplu comun al numerelor a, b, c i pentru c este cel mai mic n r
= [a,b,c] n = [a, b,c] + r.
Dac asupra lui n se impune o condiie vom considera toi multiplii comuni
care ndeplinesc condiia pentru a determina numrul n .
Exemplu: Determinai numerele naturale cuprinse ntre 1200 i 5200 care
mprite la 20 ;28 ;36 s dea de fiecare dat restul 5.
Soluie:
Fie n numrul, atunci avem :
n = 20c
1
+ 5 n 5 = 20 c
1
= M
20

n = 28c
2
+ 5 -5 n 5 = 28 c
2
= M
28

n = 36c
3
+ 5 n 5 = 36 c
3
= M
36
n 5 este multiplu comun al numerelor 20;28; 36.
Aflm c.m.m.m.c al numerelor [20;28;36] =1260
n 5 {1260;12602;12603;12604}
n 5 = 1260 n = 1265
n 5 = 2520 n = 2525
n 5 = 3780 n = 3785
n 5 = 5040 n = 5045
Problema are patru soluii: 1265 ; 2525 ; 3785 i 5045
20
R2.4.3.Numerele a,b,c mprite la acelai numr natural dau resturile r
1
, r
2
, r
3
. S se
afle numrul la care au fost mprite.

Soluie:
Fie n mpritorul, n < min (a;b; c)
a = nc
1
+ r
1
| -r
1
a - r
1
= nc
1
n | a-r
1

b = nc
2
+ r
2
| -r
2
b r
2
= nc
2
n | b-r
2

c = nc
3
+ r
3
| -r
3
c r
3
= nc
3
n | c-r
3

n este divizor comun al numerelor a r
1
, b r
2
, c r
3
, i n >max (r
1
;r
2
;r
3
)
Aflm cc.m.m.d.c. al numerelor a- r
1
; b- r
2
; c- r
3
; i lum pentru n valorile celui mai
mare divizor comun i divizorii si mai mari dect max (r
1
,r
2
,r
3
).
Exemplu: Numerele 1333 i 351 dau resturile 13 i respectiv 15 la mprirea
cu acelai numr natural diferit de zero. Aflai acest numr.
Soluie :
Fie n mpritorul , n > 15 1333 = n c
1
13 |-13 1320 = nc
1

351 = n c
2
- 15 |-15 336 = nc
2

n | 1320 i n | 336 n divizor comun al numerelor 1320 i 336.
Aflm c.m.m.d.c a celor dou numere: (1320; 336) = 2
3
3 = 24.
Singura soluie este n =24 pentru c divizorii ceilali alui 24 sunt mai mici dect 15.


2.5. Determinarea a dou numere naturale cnd cunoatem c.m.m.d.c. al lor i
produsul sau suma numerelor

Probleme rezolvate

R2.5.1. Determinai numerele a i b naturale pentru care: (a, b) = 15 i
ab = 6300
Soluie:
Din (a;b) =15 a = 15k i b = 15p unde k, p N* i (k; p) = 1
nlocuim pe a i b n relaia ab = 6300 i obinem : 15 k15p = 6300 | :225
kp = 12 1) k =1, p = 12 a= 15, b =180
2) k =12, p = 1 a = 180, b = 15
3) k = 3, p = 4 a = 45, b = 60
4) k = 4, p = 3 a = 60, b = 45

R2.5 2. S se afle numerele a i b naturale , tiind c cel mai mic multiplu
comun al lor este m i produsul lor este p.
Soluie:
Dac a, b N* atunci [ a;b](a;b) = ab
Din aceast relaie rezult c (a;b) = c
m
p
b] [a;
b a
notam ,
b] [a;
ab
= =

(a;b) = c a
= ck , b = cp , k,p N* , (k;p) = 1.
21
Rezolvarea se face analog cu problema precedent.
R2.5.3. Determinai numerele naturale a i b tiind c (a; b) = d i a + b = s.
Soluie:
(a; b) = d a = dk , b = dp , unde k,p N i (k;p) =1
nlocuim pe a i b n a + b = s i obinem : dk + dp = s | :d ,
k + p =
d
s
N , pentru c d | s. Determinm perechile de numere (k;p) ce verific
egalitatea , apoi numerele a i b.


2.6. Fracii reductibile . Fracii ireductibile

Pentru a demonstra c o fracie este ireductibil trebuie s artm c
numrtorul i numitorul ei sunt numere prime ntre ele , numrtorul i numitorul
fiind numere naturale .
Fie a, b N*, a i b sunt prime ntre ele dac ( a; b ) = 1 unde ( a; b ) este
c.m.m.d.c. al numerelor a i b.


Probleme rezolvate
R2.6.1. Se consider fracia : ,
2 3n
3 5n
+
+
n N*. Artai c fracia este ireductibil.
Soluie:
Presupunem c ( ) d astfel nct :
d | 5n + 3 i d | 3n +2
d | 3(5n + 3) i d | 5( 3n + 2 )
d | 5(3n + 2 ) 3( 5n +3 )
d | 15n +10- 15n 9 d | 1 d =1 numrtorul i numitorul sunt
numere naturale prime ntre ele rezult c fracia este ireductibil.
R2.6.2.Artai c fracia :
4 15n
3 10n
+
+
, n N este ireductibil.
Soluie:
Calculm c.m.m.m.c al numerelor 10 i15 [10 ; 15 ] = 30 , 30:10 =3; 30 :
15 = 2 . Fie d cel mai mare divizor comun al numerelor 10n +3 i 15n +4
d |10n + 3 i d | 15n+4
d | 3(10n + 3) i d | 2(15n +4)
d | 30n + 9 - 30n 8
d |1 d =1 fracia este ireductibil.

Reductibilitatea fraciilor
Pentru a arta c o fracie care depinde de o variabil natural este reductibil,
procedm astfel:
22
Fie fracia:
3 2n
1 3n
+
+
, n N.
Determinai numerele naturale n pentru care fracia este reductibil.
Soluie:
Fie d divizorul comun al numerelor 3n +1 i 2n + 3
d | 3n + 1 i d | 2n + 3
d | 2(3n + 1) i d | 3(2n + 3)
d | 6n 9 6n 2
d | 7 d = 7 pentru c 7 este numr prim
7 |3n + 1 i 7 | 2n + 3
7 | (3n + 1) (2n + 3)
7 | 3n + 1 - 2n -3
7 | n 2 n 2 = 7k , k N
n = 7k+ 2 n {2;9;16;23;;7k+2;}
Cel mai mic numr pentru care fracia este reductibil este n =2.




Bibliografie
C. Nstsescu,C. Ni, C. Vraciu, Aritmetic i algebr, EDP 1993
D. Buneag, F. Boboc, D. Piciu, Aritmetic i teoria numerelor, Ed. Universitaria
Craiova 1999
D. V. George, Cunotine vechi i noi despre divizibilitate, Ed. tiinific i
enciclopedic 1990
I. Petric i colectivul, Manual pentru clasa a VI-a, Ed. Petrion 1998
C. Popovici, I. Ligor, V. Alexianu, Matematic-Aritmetic-Algebr, EDP Bucureti
1996
G. Turcitu, I. Rizea, C. Basarab, M. Duncea, Manual clasa a VI-a, Ed. Radical 1998
T. Udrea, D. Nuescu, Manual clasa a VI-a, EDP 1998
Gheorghe i Alina Drugan; Ion i Mihaela Ghica, Matematica n concursurile colare,
Ed. Teora 1998, pag 34-38
A. Blaga, O.Pop, R. Pop. G. Buth, Matematica-Auxiliar la manualele de matematic,
Ed. Gil Zalu 2001, pag 20-28
D. Brnzei, D. i M. Goleteanu, S. Ulmeanu, V. Gorgot, I. erdean: Matematica n
concursurile colare, Ed. Paralela 45, 2000,2001,2002
D. Andrica, E. Jecan, D. Vlcan, I. Bogdan, Probleme calitative n matematica de
gimnaziu,Ed. Gil Zalu 1998, pag 21-44
C. Moroti, M. Giurgiu, D. Radu, R. tefan, A. Ciupitu, G. Drugan, I. Ghica, Mate-
matic-exerciii i probleme pentru clasa a VI-a, Ed. Meteor Press 2002, pag 12-17

23
3.Cteva principii i metode de rezolvare a problemelor de matematic

3.1. Principiul paritii

n matematica elementar ntlnim multe probleme care folosesc noiunea de
paritate. Principiul paritii const n separarea cazurilor pare i impare dintr-o situaie.
Regulile paritii:
- suma a dou numere pare este un numr par
- suma a dou numere impare este un numr par
- suma dintre un numr par i altul impar este un numr impar
- produsul a dou numere pare este un numr par
- produsul a dou numere impare este un numr impar
- produsul dintre un numr par i un numr impar este un numr par.
Prezentm n continuare cteva probleme rezolvate care folosesc principiul
paritii.
R3.1.1. Demonstrai c dac suma a dou numere ntregi este un numr impar,
produsul lor este un numr par.
Soluie. Fie a i b numerele. Din ipotez 1 2 + = + n b a , N n . Deci unul din
numerele a sau b este par. Fie k a 2 = . Atunci
1 ) ( 2 2 1 2 1 2 + = + = + = k n k n a n b , adic b este impar. Atunci b a este
produsul dintre un numr par i altul impar, deci va fi impar.
R3.1.2. Demonstrai c
n
2 N n n , 2 ( ) se poate scrie ca o sum de dou
numere naturale impare consecutive, iar
n
3 se poate scrie ca o sum de trei numere
naturale consecutive i ca sum a trei numere impare consecutive.
Soluie. Pentru orice 2 n , N n ,
n
2 este numr par. Avem:
) 1 2 ( ) 1 2 ( 2 2 2 2 2
1 1 1 1 1
+ + = + = =
n n n n n n

Pentru 2 n , N n , 1 2
1

n
i 1 2
1
+
n
sunt impare consecutive.
Pentru orice 2 n , N n , N
n
3 i
) 1 3 ( 3 ) 1 3 ( 3 3 3 3 3 3
1 1 1 1 1 1
+ + + = + + = =
n n n n n n n n

Numerele 1 3
1

n
,
n
3 i 1 3
1
+
n
sunt consecutive pentru 2 n .
Mai avem c
) 2 3 ( 3 ) 2 3 ( 3 3 3 3 3 3
1 1 1 1 1 1 1
+ + + = + + = =
n n n n n n n n
,
unde 2 3 , 3 , 2 3
1 1 1
+
n n n
sunt impare consecutive.
R3.1.3. Se consider irul numerelor naturale de la 1 la 1979 adic:
1,2,3,4,...,1977,1978,1979. Luai la ntmplare oricare dou numere din acest ir i
nlocuii-le cu modulul diferenei lor. La fiecare operaie de acest fel numrul
numerelor din ir scade cu unu (fiindc am nlocuit dou numere cu unul) i vom
obine, n final, un singur numr. Artai c acest numr este par.
Soluie. La fiecare etap a operaiei descrise, numrul numerelor impare din ir
rmne neschimbat sau descrete cu doi, deoarece dac, n primul caz, lum un numr
24
par i unul impar, modulul diferenei lor este impar, deci numrul impar l-am nlocuit
cu altul impar, iar n al doilea caz dac lum dou numere impare, modulul diferenei
lor este un numr par, deci numrul numerelor impare scade cu doi. n irul
1,2,3,...,1979 avem (1+1979):2 numere impare, adic 990.
La fiecare pas rmne un numr par de numere impare i atunci ultimul numr
va fi cu siguran par.
R3.1.4. Se consider numerele impare
k
n n n k ,..., , ,
2 1
. S se demonstreze c
printre numerele:
2
,
2
,...,
2
,
2
1 1 3 2 2 1
n n n n n n n n
k k k
+ + + +

exist un numr impar de
numere impare.
Soluie. Suma a dou numere impare este un numr par, deci numerele
2
,...,
2
,
2
1 3 2 2 1
n n n n n n
k
+ + +
sunt naturale. S presupunem c printre acestea se afl
un numr par de numere impare. Atunci suma lor
k
k
n n n
n n n n n n
+ + + =
+
+ +
+
+
+
...
2
...
2 2
2 1
1 3 2 2 1

este un numr par. Dar aceeai sum este suma unui numr impar de numere impare
deci este un numr impar. Contradicie. Deci presupunerea fcut a fost fals, deci
printre numerele considerate n ipotez exist un numr impar de numere impare.


3.2. Probleme de numrare

Probleme de numrare ntlnim n diverse situaii din viaa cotidian. n
matematica colar sunt frecvente problemele de numrare ca de exemplu: numrul
divizorilor unui numr, numrul triunghiurilor, numrul patrulaterelor dintr-o anumit
configuraie, numrul cifrelor unui numr, numrul termenilor unui ir, etc. Prezentm
n continuare cteva probleme care conduc la operaia de numrare.


3.2.1. Numrul divizorilor i suma divizorilor unui numr natural

3.2.1.1. a) Numrul divizorilor unui numr natural
Fie a un numr natural compus ce are urmtoarea descompunere n factori
primi:
n
n
p p p a

= ....
2 1
2 1
, unde
n
p p p ,..., ,
2 1
sunt numere prime iar
N N n
n
, ,..., ,
2 1
. Pentru a obine numrul divizorilor lui a formm tabelul:
termeni 1 ... p
... ... ... ... ...
termeni 1 ... p
termeni 1 ...
2 1 0
n
2 2
2
2
1
2
0
2
1 1
2
1
1
1
0
1
2
1
+
+
+

n n n n
n
p p p
p p p
p p p p
(1)
25
Observm c:
1) Oricare numr din tabel este un divizor pentru a.
2) Linia nti conine
1
+1 termeni, linia a doua conine
2
+1 termeni,...,
ultima linie conine
n
+1 termeni.
3) Dac nmulim pe rnd fiecare numr din linia nti cu fiecare numr din
linia a doua obinem ) 1 )( 1 (
2 1
+ + divizori ai lui a. nmulind apoi pe fiecare din
aceste numere cu fiecare numr din linia a treia obinem ) 1 )( 1 )( 1 (
3 2 1
+ + +
numere i fiecare din acestea sunt divizori ai lui a. Continund raionamentul obinem
) 1 )...( 1 )( 1 )( 1 (
3 2 1
+ + + +
n
numere care sunt divizori ai lui a.
4) n numrul acestor divizori este inclus numrul nsui i divizorul 1.
Am obinut astfel urmtoarea
Teorema 3.2.1. Numrul divizorilor numrului
n
n
p p p a

= ....
2 1
2 1
este
) 1 )...( 1 )( 1 (
2 1
+ + +
n
.
3.2.1.2. b) Suma divizorilor unui numr natural
S calculm nti suma:
n
x x x S + + + + = ... 1
2
(2)
Avem
1 1 2
...
+
+ + + + + =
n n n
x x x x x S x (3)
Din (3) i (2) sczute membru cu membru obinem:
) ... 1 ( ) ... (
2 1 3 2 n n n
x x x x x x x x S S x + + + + + + + + + =
+

care se mai scrie 1 ) 1 (
1
=
+ n
x x S , de unde
1
1
1

=
+
x
x
S
n
(4)
cu 1 x .
Scriem produsul de n sume, avnd termenii pe cele n linii din tabelul (1) i
obinem:
) ... 1 )...( ... 1 )( ... 1 (
2
2
2
2 2 1
2
1 1
2 1 n
n n n
p p p p p p p p p

+ + + + + + + + + + + + (5)
Cu relaia (4), (5) devine
1
1
...
1
1
1
1
1
2
1
2
1
1
1
2 1

+ + +
n
n
p
p
p
p
p
p
n

Am obinut astfel
Teorema 3.2.2. Suma divizorilor numrului
n
n
p p p a

= ....
2 1
2 1
este
1
1
...
1
1
1
1
1
2
1
2
1
1
1
2 1

=
+ + +
n
n
p
p
p
p
p
p
S
n


26
Probleme rezolvate
R3.2.1. Fie S suma divizorilor naturali ai numrului 2001. S se arate c 5S
este numr natural ptrat perfect.
Soluie. Fiindc 2001=3
1
23
1
29
1
, suma divizorilor numrului 2001 este:
30 24 4
1 29
1 29
1 23
1 23
1 3
1 3
2 2 2
=

= S
Atunci
2 2 3
120 ) 5 3 2 ( 5 = = S , deci 5S este ptrat perfect.
R3.2.2. S se arate c ptratul produsului tuturor divizorilor naturali ai
numrului 2001 este 2001
8
.
Soluie. Avem urmtoarea
Lema 3.2.1. Dac
n
d d d ,..., ,
2 1
sunt toi divizorii naturali ai numrului n
atunci avem relaia:
k
k
n d d d =
2
2 1
) ... ( (*)
Fiindc 1 i n sunt i ei divizori, considernd
k
d d d < < < ...
2 1
obinem:
1 1
2 1
,..., ,
d
n
d
d
n
d
d
n
d
k
k k
= = =


relaii care nmulite membru cu membru dau
1 1
2 1
... ...
d
n
d
n
d
n
d d d
k k
k

=
de unde
k
k
n d d d =
2
2 1
) ... ( .
n cazul nostru
1 1 1
29 23 3 2001 = , numrul divizorilor lui 2001 este:
(1+1)(1+1)(1+1)=8. Pentru cei opt divizori naturali ai numrului 2001 avem relaia (*)
8 2
8 2 1
2001 ) ... ( = d d d .

3.3. Principiul lui Dirichlet

Matematicianul german Peter Gustav Dirichlet (1805-1859) a elaborat un
principiu extrem de simplu cu aplicaii neateptate n variate domenii, principiu care-i
poart numele i pe care-l enunm mai jos, fiind o metod de demonstraie de tipul
urmtor.
"Dac repartizm 1 + n obiecte n n cutii, atunci cel puin dou obiecte vor fi
n aceeai cutie."
Justificare: Considerm cazul cel mai nefavorabil aeznd n fiecare cutie cte
un obiect. Deci am folosit n cutii i n obiecte. Obiectul cu numrul 1 + n trebuie pus i
el ntr-o cutie oarecare. Dar n acea cutie exist deja un obiect. Aadar n acea cutie
exist deja un obiect pus anterior. n acea cutie vor fi dou obiecte.
Forma general a principiului lui Dirichlet este urmtoarea:
"Dac aezm 1 + kn obiecte n n cutii, atunci cel puin 1 + k obiecte, N k ,
vor fi n aceeai cutie."
27
n literatura matematic principiul lui Dirichlet este ntlnit i sub denumirea
de "principiul cutiei", cu precizarea c denumirea de "cutie" desemneaz "grupe de
obiecte", stabilite dup anumite criterii, iar "obiectele" desemneaz lucruri, numere,
figuri geometrice, etc. Prezentm n continuare cteva probleme ale cror soluii se
bazeaz pe principiul de mai sus.

Probleme rezolvate
R3.3.1. La un turneu de ah au participat 2 n ahiti. S se demonstreze c
n orice moment al turneului dinaintea ultimei runde cel puin doi ahiti au acelai
numr de victorii.
Soluie. n orice moment al turneului dinaintea ultimei runde, fiecare ahist a
jucat maximum 2 n partide i a putut obine 2 ,..., 2 , 1 , 0 n victorii, deci n total
1 n posibiliti (cutii). Fiindc la turneu au participat n ahiti rezult c cel puin doi
ahiti au acelai numr de victorii naintea ultimei runde.
R3.3.2. Artai c n orice mulime format din 5 numere naturale exist dou
a cror diferen este divizibil cu 4.
Soluie. La mprirea unui numr cu 4 obinem unul din resturile 0,1,2,3 (deci
patru cutii). Fiindc avem 5 numere (5 obiecte i 4 cutii) rezult c cel puin dou
numere vor da acelai rest la mprirea cu 4. Ele sunt de forma r k x + = 4 i
r l y + = 4 . Atunci diferena lor este ) ( 4 l k y x = , adic un numr divizibil cu 4.
R3.3.3. ntr-o coal sunt 367 elevi. S se demonstreze c exist cel puin doi
elevi care-i serbeaz ziua n aceeai zi a anului.
Soluie. Un an are 365 sau 366 zile. Considernd cazul cel mai nefavorabil
cnd n fiecare zi a anului ar fi nscut cte un elev, nseamn c n total ar fi nscui
365 sau 366 elevi, dar n total sunt 367 elevi. Deci al 367-lea elev a fost i el nscut
ntr-o zi a anului n care a mai fost nscut un elev. Deci ntr-o zi s-au nscut 2 elevi,
deci cei doi i vor serba ziua de natere n aceeai zi.
R3.3.4. Fiind date 1 + n numere naturale ) 0 ( n atunci cel puin dou dintre
ele dau acelai rest la mprirea cu n.
Soluie. Folosim teorema mpririi cu rest. Fiind date numerele naturale a i b
) 0 ( b exist n mod unic numerele naturale q i r astfel ca
r q b a + = cu b r < .
n cazul problemei noastre numerele fiind mprite la n exist pentru rest n
valori posibile: 0,1,2,...,n-1. Fiindc mprim n+1 numere vor exista n+1 resturi, dintre
care cel mult n sunt diferite. Rezult c cel puin dou dintre cele n+1 numere mprite
la n dau acelai rest.
R3.3.5. S se arate c oricum am alege 7 numere ptrate perfecte (distincte)
exist cel puin dou a cror diferen se divide cu 10.
Soluie. Dac a este numrul a crui ptrat este
2
a atunci la mprirea cu 10 a
lui a obinem unul din resturile: 0,1,2,3,4,5,6,7,8,9. Atunci
2
a , la mprirea cu 10 va
da unul din resturile: 0,1,4,5,6,9. Fiindc avem 7 ptrate perfecte i numai resturile
0,1,4,5,6,9, deci exist cel puin dou ptrate perfecte care dau acelai rest la mprirea
cu 10, deci diferena lor se divide cu 10.

28
3.4. Principiul invariantului

Invariantul este o mrime, o relaie, sau o proprietate care rmne neschimbat
n urma aplicrii sau interveniei unei transformri.
Deci o situaie iniial este supus n mod repetat unor transformri. De obicei
se cere s se demonstreze c n urma acestor transformri nu se poate ajunge la o
anumit form. Aceasta se poate face alegnd caracteristica obiectului care a fost supus
transformrii, adic "invariantul" transformrii. Dac n final obiectul nu posed
"invariantul" atunci el nu poate fi obinut n urma transformrilor descrise.

Probleme rezolvate

R3.4.1. Considerm un numr natural cruia i schimbm n mod arbitrar
ordinea cifrelor. Este posibil ca diferena dintre numrul iniial i cel final s fie 2003?
Soluie. Restul mpririi numrului la 9 este acelai cu restul mpririi sumei
cifrelor sale la 9. Suma cifrelor este aceeai, rezult c restul mpririi numrului la 9
este un invariant.
R3.4.2. Pe o tabl sunt scrise semne de "+" i "". tergem dou semne i le
nlocuim cu un semn, dup urmtoarea regul: dac cele dou semne terse sunt
identice le nlocuim cu "+", iar dac tergem dou semne diferite le nlocuim cu "".
Artai c ultimul semn care rmne dup un numr de pai nu depinde de ordinea
alegerii perechilor.
Soluie. n acest caz paritatea numrului de minusuri va fi invariantul. Dac la
nceput numrul de minusuri este impar, ultimul semn care va rmne este minus, iar
dac la nceput numrul de minusuri este par, la sfrit va rmne plus.
R3.4.3. Trei greieri se gsesc pe o dreapt n ordinea: A, B, C. Ei ncep s sar
capra, adic s sar unul peste altul (dar nu peste doi odat). Pot fi n aceeai ordine
dup 2003 srituri?
Soluie. n urma unei srituri de acest fel numrul perechilor de greieri
inversai crete sau se micoreaz cu 1 (proprietatea invariant). Dup un numr impar
de srituri (2003) va exista un numr impar de perechi de greieri inversai. Deci nu se
poate obine ordinea iniial (ce nu conine o astfel de pereche).
R3.4.4. O camer are dimensiunile podelei de 7m i 10m. n cele patru coluri
ale camerei se aeaz cte un dulap avnd baza ptrat cu latura de 1m. S se arate c
rmne din suprafaa podelei nu poate fi acoperit cu plci dreptunghiulare de
dimensiuni 3m1m.
Soluie. Se mparte camera ntr-o reea de ptrate cu latura 1m pe care le
vopsim n trei culori: rou, alb, negru ca mai jos:
RANRANRANR
ANRANRANRA
NRANRANRAN
RANRANRANR
ANRANRANRA
NRANRANRAN
RANRANRANR
29
Obinem 24 de R, 23 de A, 23 de N. Eliminnd colurile rmn 20 de ptrele
roii, 23 de ptrele albe, 23 de ptrele negre. Dar oricum am aeza o plac de 31
ea acoper un ptrel rou, unul alb i unul negru. Dac s-ar putea acoperi suprafaa cu
un numr ntreg de plci ar trebui s existe acelai numr de ptrele pentru fiecare
culoare.


3.5. Probleme de logic

Am inclus aici cteva probleme a cror rezolvare se realizeaz printr-o serie de
judeci logice ce solicit inventivitate, perspicacitate, etc. i foarte puin calcul.

Probleme rezolvate

R3.5.1. Mama a observat c din dulap au disprut cinci tablete de ciocolat.
Ele puteau fi luate de cei trei copii: A, B, C. Fiind trai la rspundere, ei au dat mai
nti urmtoarele rspunsuri:
A: N-am luat nici o ciocolat!
B: N-am luat nici o ciocolat!
C: N-am luat nici o ciocolat!
Dup un nou "interogatoriu" copiii au fcut urmtoarele declaraii:
A: B a luat mai multe tablete dect C!
B: (ctre A): Mini!
C: Toate au fost luate de A i B!
A (ctre C): Mini!
Aflai cte tablete de ciocolat au fost luate de ctre fiecare copil, tiind c
fiecare a fcut attea declaraii false cte tablete de ciocolat a luat.
Soluie. Fiindc au "disprut" 5 ciocolate i s-au fcut 7 declaraii, rezult c
cinci declaraii erau false iar 2 (7-5) adevrate.
La al doilea "interogatoriu" prima afirmaie a lui A este fie adevrat i atunci
afirmaia lui B este fals, fie este fals i atunci afirmaia lui B este adevrat. Tot din
"interogatoriul" al doilea afirmaia lui C este fie adevrat i atunci cea de-a doua
afirmaie a lui A este fals, fie este fals i atunci cea de-a doua afirmaie a lui A este
adevrat. Deci rezult c cele dou afirmaii adevrate au fost fcute la cel de-al
doilea "interogatoriu", deci la primul "interogatoriu" toi copiii au fcut declaraii false,
de unde rezult c fiecare din cei trei copii a luat cel puin o ciocolat. Deci a doua
afirmaie a lui C este fals, deci C a luat dou tablete de ciocolat. B face numai dou
afirmaii, deci el nu poate lua mai multe ciocolate dect C, rezult c la al doilea
"interogatoriu" prima afirmaie a lui A este fals, deci afirmaia lui B este adevrat.
Deci A a luat dou ciocolate, B a luat o ciocolat, iar C a luat dou ciocolate.
R3.5.2. ntr-un bloc locuiesc familiile A, B, C, D, E, F, G, H, I, K, L, M, N, O,
P, R. La parter i la fiecare etaj locuiesc cte dou familii. Se mai tie c: Familia A
locuiete cu dou etaje mai jos ca familia B, iar aceasta cu ase etaje mai sus ca familia
C. Familiile F i G locuiesc la acelai etaj. Familia M locuiete cu patru etaje mai sus
30
ca familia N i cu dou etaje mai jos ca familia F. Un etaj deasupra familiei N locuiete
familia O. Familia A locuiete cu trei etaje mai sus ca familia R, iar familia P locuiete
cu cinci etaje mai jos dect familia G.
a) Cte etaje are blocul?
b) La ce etaj locuiete familia A?
Soluie. a) Fiindc la parter i la fiecare etaj locuiesc dou familii, iar n tot
blocul locuiesc 16 familii, rezult c blocul are opt nivele (parter i apte etaje).
b) Diagramele alturate (stabilite conform enunului) pun n eviden modul
cum sunt distribuite n bloc familiile N, O, M, F, G, P precum i C, R, A, B. Blocul
avnd opt nivele, rezult c familia N poate locui numai la parter sau la etajul nti.
Aceeai remarc i pentru familia C. Familiile N i C nu pot locui la acelai nivel,
pentru c ar trebui ca familiile O, P, R s locuiasc la acelai etaj, situaie imposibil,
pentru c la un nivel pot locui numai dou familii. Dac familia N locuiete la parter
atunci familia C ar trebui s locuiasc la etajul nti, situaie imposibil deoarece ar
rezulta c familiile O, P, C locuiesc la acelai etaj. Dac familia N locuiete la etajul
nti, atunci familia C locuiete la parter. Urmrind comparativ diagramele observm
c este o situaie posibil pentru c la un nivel pot locui numai dou familii.
Acestea fiind precizate putem stabili distribuiile familiilor n bloc: Familiile F
i G la etajul apte, familia B la etajul ase, familia M la etajul cinci, familia A la etajul
patru, familiile P i O la etajul doi, familiile N i R la etajul nti, iar familia C la
parter. n cele ase locuri neocupate se vor distribui familiile D, E, H, I, K, L dup
voie.
R3.5.3. La un turneu de fotbal particip 15 echipe, fiecare dintre acestea
jucnd cu toate celelalte. Pentru victorie se acord 3 puncte, pentru meci egal 2 puncte,
iar pentru nfrngere un punct. n clasamentul ntocmit la sfritul turneului nu exist
echipe cu acelai numr de puncte. tiind c ultima echip are 21 de puncte, s se arate
c prima a fcut cel puin un meci nul.
Soluie. Fiecare echip a disputat 14 meciuri. Numrul meciurilor disputate a
fost 105
2
15 14
=

deoarece fiecare meci a fost numrat de dou ori (i cnd a jucat A


cu B i cnd a jucat B cu A). Fiindc echipa clasat pe ultimul loc are 21 de puncte, iar
n clasament nu sunt echipe cu acelai numr de puncte rezult c numrul de puncte
este mai mare sau egal cu
F
M
O
N
G
P
B
A
R
C
31
420
2
15 14
15 21 ) 14 21 ...( ) 2 21 ( ) 1 21 ( 21 =

+ = + + + + + +
Fiindc la fiecare meci s-au acordat 4 puncte, rezult c numrul de puncte
acordat a fost 1054=420. Deci echipele au obinut punctajele: 21, 22,23,...,34, 35.
S artm c echipa de pe locul nti cu 35 puncte a fcut cu siguran cel puin
un meci nul. Presupunem c nu a fcut nici un meci nul. Atunci dac x este numrul
victoriilor i y numrul nfrngerilor avem:
14 = + y x i 35 3 = + y x ,
de unde
2
21
= x i
2
7
= y . Dar x, y trebuie s fie naturale. Deci presupunerea fcut
este fals, atunci echipa de pe locul nti a fcut cel puin un meci nul.

R3.5.4. La un concurs de atletism particip trei echipe:
3 2 1
, , A A A , fiecare cu
trei concureni. Concurentul care sosete primul primete 18 puncte, cel care sosete al
doilea 16 puncte, cel care sosete al treilea 14 puncte,..., cel care sosete ultimul
primete dou puncte. Punctajul unei echipe este suma punctelor obinute de cei trei
reprezentani ai si. Aflai ce loc a ocupat fiecare echip tiind c:
i) Primele trei locuri au fost ocupate de concureni de la echipe diferite.
ii) Fiecare concurent de la echipa
2
A avea n faa sa un concurent de la echipa
1
A .
iii) Concurenii echipei
3
A au sosit unul dup altul.
Soluie. Din prima i a treia condiie rezult c cei trei reprezentani ai echipei
3
A au sosit al treilea, al patrulea i al cincilea. Din primele dou condiii rezult c
primul a sosit un concurent de la echipa
1
A , iar al doilea un concurent de la echipa
2
A .
Din cele de mai sus i din a doua condiie rezult c al aselea i al optulea au sosit
concurenii de la echipa
1
A , iar al aptelea i al noulea au fost concurenii de la echipa
2
A . Deci echipa
1
A a acumulat 18+8+4=30 (puncte), echipa
2
A a acumulat
16+6+2=24 (puncte), iar echipa
3
A a acumulat 14+12+10=36 (puncte). Deci pe locul
nti se afl echipa
3
A , pe locul doi echipa
1
A , iar pe locul trei echipa
2
A .
R3.5.5. Opt ahiti particip la un turneu, jucnd fiecare cu fiecare. Pentru
fiecare victorie un juctor primete un punct, pentru remiz un jumtate de punct, iar
pentru nfrngere nu primete nici un punct. La sfritul turneului primii doi clasai au
obinut punctaje diferite, iar cel de-al doilea a obinut attea puncte cte au obinut
ultimii patru ahiti mpreun. S se afle cum s-a ncheiat partida dintre ahitii clasai
pe locurile trei i cinci.
Soluie. Fiindc au fost opt juctori i fiecare a jucat cu fiecare, un ahist a
jucat apte partide i ar fi putut ctiga cel mult apte puncte (cnd nvingea n toate
cele apte partide). Ultimii patru ahiti au jucat ntre ei ase partide. (Dac D C B A , , ,
sunt ultimii ahiti, au jucat: A cu B , A cu D, B cu C , B cu D i C cu D).
32
Deci ultimii patru ahiti au realizat mpreun cel puin ase puncte. Deci ahistul de pe
locul doi a obinut cel puin ase puncte (deoarece el a obinut un numr egal de puncte
cu suma ultimilor patru). Fiindc primii doi juctori au punctaje diferite nseamn c al
doilea a obinut exact ase puncte, cci dac obinea 6,5 puncte primii doi aveau acelai
punctaj, iar dac ar fi obinut apte puncte era pe primul loc. Deci ahitii de pe
ultimele patru locuri au obinut exact ase puncte, aceasta nseamn c ei au pierdut
toate partidele jucate mpotriva primilor patru clasai. Deci ahistul de pe locul cinci a
pierdut partida susinut cu cel de pe locul trei.
R3.5.6. n trei couri sunt mere. Cte mere sunt n fiecare co tiind c n
primele 2 mpreun este un mr, n ultimele 2 mpreun este cel puin un mr, iar n
ultimul i al treilea mpreun, numai unul.
Soluie. Dac mrul din primele 2 couri s-ar afla n primul co, atunci din a
treia condiie ar rezultat c n al treilea co nu se afl nici un mr, deci al doilea i al
treilea co ar fi goale i astfel nu ar avea loc a doua condiie. n concluzie primul co
este gol, al doilea co conine un mr, iar al treilea co conine tot un mr.


3.6. Probleme de ordonare

Pentru a stabili care dintre dou numere a i b este mai mare, putem folosi mai
multe procedee, dintre care cele mai des ntrebuinate sunt:
1) Stabilim semnul diferenei b a .
Dac 0 > b a , atunci b a > .
Dac 0 = b a , atunci b a = .
Dac 0 < b a , atunci b a < .
2) Dac numerele a i b sunt pozitive i 0 b , comparm raportul
b
a
cu 1.
Dac 1 <
b
a
, atunci b a < .
Dac 1 =
b
a
, atunci b a = .
Dac 1 >
b
a
, atunci b a > .
3) n unele situaii este suficient s demonstrm existena unui numr c situat
ntre cele dou numere (exemplu: din b c a < < , rezult b a < ).
n unele situaii avem nevoie de metode ingenioase pentru a rezolva
problemele. Exist cazuri cnd operaia de ordonare ajut la dovedirea egalitii a dou
numere x i y prin stabilirea simultan a inegalitilor y x i x y .

Probleme rezolvate

R3.6.1. Comparai numerele
11
31 cu
14
17 .
33
Soluie.
14 14 14 4 56 55 11 5 11 11
17 16 ) 2 ( 2 2 ) 2 ( 32 31 < = = < = = < .
R3.6.2. Scriei n ordine cresctoare numerele:
7 44 34 22
63 , 2 , 3 , 4 .
Soluie. = = < = = < = = <
17 3 51 22 22 2 44 42 7 6 7 7
) 2 ( 2 4 ) 2 ( 2 2 ) 2 ( 64 63
34 17 2 17 17
3 ) 3 ( 9 8 = = < = . Deci
34 22 44 7
3 4 2 63 < = < .
R3.6.3. S se arate c pentru orice numere naturale a i b avem
1440
1800
240
360
7
5
3
2
+
+
<
+
b
b
a
a

Soluie.
120 120 3 360
8 ) 2 ( 2 = = ;
120 120 2 240
9 ) 3 ( 3 = = .
Atunci obinem c
240 360
3 2 < , de unde rezult c
240 360
3 2 + < + a a i deci
1
3
2
240
360
<
+
+
b
a

360 360 5 1800
3125 ) 5 ( 5 = = ;
360 360 4 1440
2401 ) 7 ( 7 = =
Rezult c
1440 1800
7 5 > , de unde rezult c
1440 1800
7 5 + > + b b i deci 1
7
5
1400
1800
>
+
+
b
b
.
Deci prima fracie din ipotez este subunitar iar a doua este supraunitar i atunci
relaia cerut este adevrat.
R3.6.4. Comparai fraciile A i B unde
1997 2
1997 3 2
2
1
...
2
1
2
1
2 ... 2 2 2
+ + +
+ + + +
= A i
1331 2
1331 2
3
1
...
3
1
3
1
3 ... 3 3
+ + +
+ + +
= B
Soluie. Amplificm prima fracie cu
1998
2 i a doua cu
1332
3 i obinem:
666 666 3 1998
1997 1998
1997 3 2 1998
8 ) 2 ( 2
2 ... 2 2
) 2 ... 2 2 2 ( 2
= = =
+ + +
+ + + +
= A
666 666 2 1332
1331 2
) 1331 2 1332
9 ) 3 ( 3
3 ... 3 3
3 ... 3 3 ( 3
= = =
+ + +
+ + +
= B .
Deci B A > .


3.7. Metoda reducerii la absurd

Metoda reducerii la absurd este o metod specific de demonstraie n
matematic. La baza acestei metode st una din legile fundamentale ale logicii clasice:
legea terului exclus, ce are urmtorul enun:
Din dou propoziii contradictorii una este adevrat, cealalt fals, iar a treia
posibilitate nu exist.
34
Legea terului exclus nu ne precizeaz care din cele dou propoziii este
adevrat i care este fals.
Cnd la dou propoziii contradictorii aplicm legea terului exclus este
suficient s stabilim c una dintre ele este fals pentru a deduce c cealalt este
adevrat.
Metoda reducerii la absurd const n a admite n mod provizoriu, ca adevrat
propoziia contradictorie propoziiei de demonstrat, apoi pe baza acestei presupuneri se
deduc o serie de consecine care duc la un rezultat absurd, deoarece ele contrazic sau
ipoteza problemei date sau un adevr stabilit mai nainte. Mai departe raionm astfel:
dac presupunerea ar fi fost adevrat, atunci n urma raionamentelor logic corecte ar
fi trebuit s ajungem la o concluzie adevrat, deoarece am ajuns la o concluzie fals,
nseamn c presupunerea noastr a fost fals. Aceasta duce la concluzia c
presupunerea fcut nu este posibil i rmne ca adevrat concluzia propoziiei date.
Metoda reducerii la absurd nu se reduce la propoziia c "a demonstra o
propoziie este acelai lucru cu a demonstra contrara reciprocei ei", deoarece pot aprea
i situaii n care nu se contrazice ipoteza ci o alt propoziie (un rezultat cunoscut, o
axiom, o teorem). Metoda reducerii la absurd se folosete att n rezolvarea
problemelor de calcul (de aflat) ct i la rezolvarea problemelor de "demonstrat".
Metoda este des utilizat n demonstrarea teoremelor reciproce, precum i n
demonstrarea teoremelor de unicitate.


Probleme rezolvate

R3.7.1. Artai c pentru orice N n fracia
3 26
4 39
+
+
n
n
este ireductibil.
Soluie. Presupunem c fracia este reductibil i fie ) 3 26 , 4 39 ( + + = n n d cu
*
N d , 1 d . Din ) 4 39 ( | + n d i ) 3 26 ( | + n d obinem c ) 8 78 ( | + n d i
) 9 78 ( | + n d , de unde rezult c )] 8 78 ( 9 78 [ | + + n n d , deci 1 | d , de unde rezult
1 = d . Fals.
R3.7.2. S se arate c nu exist numere ntregi a pentru care numerele
9
5 14 + a
i
12
5 17 a
s fie simultan ntregi.
Soluie. Presupunem c exist numere ntregi a astfel ca
9
5 14 + a
i
12
5 17 a

s fie simultan ntregi, adic pentru Z c b, , b
a
=
+
9
5 14
i c
a
=

12
5 17
, de unde
b a 9 5 14 = + i c a 12 5 17 = i scznd prima relaie din a doua obinem:
b c a 9 12 10 3 = , de unde b c a 9 12 3 10 + = sau ) 3 4 ( 3 10 b c a + = . Atunci
obinem c 3 divide pe 10, ceea ce este absurd.
35
R3.7.3. Considerm trei drepte diferite
3 2 1
, , d d d concurente ntr-un punct O.
Artai c cel puin unul din unghiurile formate are msura mai mare sau cel puin
egal cu 60.
Soluie. Folosim metoda reducerii la absurd. Presupunem concluzia fals, adic
nu exist un unghi cu msura mai mare sau egal cu 60. Atunci cele ase unghiuri
formate ar avea suma msurilor mai mic dect 360. Am ajuns la o contradicie
deoarece suma msurilor unghiurilor n jurul unui punct este 360. Deci presupunerea
fcut este fals, deci exist cel puin un unghi cu msura de 60.



36
4. Rapoarte i proporii

Rapoarte
Raportul numerelor raionale a i b, 0 b , este expresia
b
a
; a i b se numesc
termenii raportului.
Ctul termenilor unui raport se numete valoarea raportului.
Exemplu: valoarea raportului
7
5 , 3
este 0,5.
Termenii unui raport se exprim ntotdeauna cu aceeai unitate de msur.
Aplicaiile rapoartelor n practic sunt: scara unui plan, scara unei hri,
probabilitatea realizrii unui eveniment, procente, titlul unui aliaj.

4.1. Scara unui plan

Prin scara unui plan nelegem raportul dintre distana din plan i distana din
realitate dintre aceleai dou puncte, ambele distane fiind exprimate cu aceeai unitate
de msur.
Remarc. De obicei, numrtorul raportului prin care se exprim scara este 1.
Model. Figura de mai jos reprezint planul unui apartament. Acest plan este
realizat la scara
100
1
. Aceasta nseamn c la 1 cm din desen corespund, n realitate,
100cm. Cu alte cuvinte, n plan lungimea sufrageriei este de 5cm, iar n realitate este de
500cm, adic de 5m.
La planul din figur s se determine:
a) limea, n centimetri, a dormitorului
b) dimensiunile, n centimetri, ale buctriei
c) perimetrul, n centimetri, a holului
d) aria, n cm
2
a sufrageriei.
Soluie.
a) Limea dormitorului de 3m, din realitate, este n plan de 3cm.
b) Dimensiunile de 2m i 3m ale buctriei, din realitate, sunt n plan de 2cm,
respectiv 3cm.
c) Holul are dimensiunile de 8m i 2m, n realitate, deci n plan ele vor fi 8cm
i 2cm, rezult c perimetrul holului n plan este de 20cm.
d) Sufrageria are dimensiunile de 5m i 4m, n realitate, deci n plan 5cm i
4cm, rezult c aria sufrageriei n plan este 20cm
2
.


Probleme rezolvate

R4.1.1. Care este scara planului unei grdini, dac o latur a grdinii, care are
125m, este reprezentat n plan printr-un segment lung de 25cm?
37
Soluie. Aplicnd definiia scrii unui plan, ca fiind raportul dintre distana din
plan i distana din realitate, ambele exprimate n aceeai unitate de msur, scara
planului este
12500
25
, adic
500
1
.
R4.1.2. O grdin n form de dreptunghi, are pe un plan cu scara de
300
1

dimensiunile de 4cm i 5cm. Ce suprafa, n hectare, are grdina n teren?
Soluie. n planul cu scara
300
1
, lungimea de 1 cm corespunde la 300 cm din
realitate. Dimensiunile grdinii vor fi 4300cm i 5300cm, adic 12m i 15m. Aria
grdinii este de 0,018ha.
R4.1.3. Planul unui parc are scara de
200
1
.
a) n plan se afl un loc de form circular, cu raza de 1m, ce reprezint lacul.
Ci centimetri are raza cercului n plan?
b) Spaiul de joac pentru copii este, n teren, un ptrat cu aria de 100m
2
. Ce
arie are n plan spaiul de joac pentru copii?
Soluie. a) n planul cu scara de
200
1
, lungimea de 1cm corespunde la 200cm
din teren. Dac raza cercului este n teren 1m, adic 100cm, ea corespunde n plan unei
lungimi de 0,5cm.
b) Latura ptratului din teren are 10m, deci 1000cm, iar n plan latura
ptratului are 1000:200=5cm. Rezult c aria ptratului n plan este de 25cm
2
.


4.2. Scara unei hri

Prin scara unei hri nelegem raportul dintre distana de pe hart i distana
din realitate dintre aceleai dou puncte, distanele fiind msurate cu aceeai unitate de
msur, iar numrtorul raportului prin care se exprim scara este 1.
Model. n figura de mai jos, harta Romniei este realizat la scara
10000000
1
,
aceasta nsemnnd c la 1cm de pe hart corespund 10000000cm=100km n realitate
(teren).
38


De exemplu, distana pe osea, dintre Bucureti i Braov, pe hart, este 17mm,
iar distana din teren d o determinm astfel:
170km cm 17000000
7 , 1
10000000
1
= = = d
d
.
Distana real dintre oraele Bacu i Bucureti este de 300km. Care este
distana, n centimetri, pe harta cu scara
10000000
1
?
Avem cm 3
30000000 10000000
1
= = x
x
.


4.3. Probabilitate

Definiie. Probabilitatea realizrii unui eveniment este raportul dintre numrul
cazurilor favorabile realizrii evenimentului i numrul cazurilor posibile (ale
experienei).
Remarc. Probabilitatea realizrii unui eveniment este un numr mai mare sau
egal cu 0 i mai mic sau egal cu 1.
Evenimentul imposibil are probabilitatea 0.
Evenimentul sigur are probabilitatea 1.
Model. ntr-o urn sunt bile numerotate de la 1 la 50. Care este probabilitatea
ca extrgnd o singur bil numrul obinut s fie ptrat perfect?
Soluie. n total, exist 50 cazuri posibile i 7 cazuri favorabile (apariia
numrului 1, 4, 9, 16, 25, 36, 49) deci probabilitatea cerut este
50
7
= p .

39
Probleme rezolvate

R4.3.1. Care este probabilitatea ca aruncnd dou zaruri, s obinem dou fee
nsumnd: a) 9 puncte; b) un numr prim de puncte.
Soluie. La aruncarea a dou zaruri exist 66=36 cazuri posibile.
a) Numrul cazurilor favorabile obinerii sumei 9 puncte este 4 (3+6, 4+5, 5+4,
6+3), deci probabilitatea cerut este
36
4
, adic
9
1
.
b) Numrul cazurilor favorabile obinerii sumei un numr prim de puncte este
15 (1+1, 1+2, 2+1, 1+4, 2+3, 3+2, 4+1, 1+6, 2+5, 3+4, 4+3, 5+2, 6+1, 5+6, 6+5),
rezult c probabilitatea cerut este
36
15
, adic
12
5
.
R4.3.2. ntr-un co sunt 6 plicuri albe i 4 plicuri roii. Un copil, legat la ochi,
extrage dou plicuri. Calculai probabilitatea evenimentelor:

1
E : s extrag dou plicuri de aceeai culoare

2
E : s extrag dou plicuri de culori diferite.
Soluie. Numrul cazurilor posibile este 910=90.
a) Numrul cazurilor favorabile realizrii evenimentului
1
E : 65+43=42,
probabilitatea realizrii evenimentului
1
E este
90
42
, adic
15
7
) (
1
= E p .
b) Numrul cazurilor favorabile realizrii evenimentului
2
E : 64+46=48 (sau
90-42); probabilitatea realizrii evenimentului
2
E este
90
48
, adic
15
8
) (
2
= E p .
Remarc. Probabilitatea realizrii evenimentului
2
E se putea calcula i
15
8
15
7
1 = , pentru c singurele situaii posibile la extragerea a dou plicuri din co
este ca ele s fie de aceeai culoare sau de culori diferite.
R4.3.3. O carte cu 270 de pagini este deschis la ntmplare. S se determine
probabilitatea evenimentelor urmtoare:
A: numrul paginii din stnga este numr par
B: numrul paginii din dreapta este multiplu de 5
C: numrul paginii din stnga este multiplu de 6
D: numrul paginii din dreapta este divizibil cu 7.
Soluie. Numrul paginii din stnga este ntotdeauna par, deci 1 ) ( = A P .
Numrul paginii din dreapta este ntotdeauna numr impar, deci trebuie s
numrm multipli impari ai lui 5, mai mici sau egali cu 265; ei sunt 51, 53, 55,...,
553=265, deci n total 27 1
2
1 53
= +

cazuri favorabile, de unde


5
1
135
27
) ( = = B P .
40
Numrul paginii din stnga este numr par ntotdeauna i el trebuie s fie
multiplu de 6 mai mic dect 270; obinem 61, 62, 63,..., 645=270, 45 cazuri
favorabile, de unde
3
1
135
45
) ( = = C P .
Numrul paginii din dreapta este numr impar, divizibil cu 7 i mai mic dect
270, obinem 71, 73, 75,..., 737, deci n total 19 1
2
1 37
= +

cazuri favorabile, de
unde
135
19
) ( = D P .


4.4. Procente

Definiie. Un raport de forma
100
p
, Q p , 0 p , se numete raport
procentual. Scrierea p% nseamn
100
p
i se citete "p la sut" sau "p procente".
Pentru a afla ct reprezint p% dintr-un numr dat a, calculm a
p

100
.
Pentru a afla un numr necunoscut x cnd tim c p% din x reprezint b,
calculm
100
:
p
b x = .
Model 1. La faza naional a olimpiadei de matematic particip 600 de elevi.
Din numrul total de participani 5% primesc premiul I, 10% premiul al II-lea, 15%
premiul al III-lea i 20% premii speciale i meniuni. Ci elevi primesc premiul I, dar
premiul al II-lea, dar premiul al III-lea? Ce procent din numrul elevilor care au primit
premii speciale i meniuni reprezint numrul elevilor cu premiul I?
Soluie. Pentru a afla ci elevi au obinut premii i meniuni, avem:
30 600
100
5
= elevi primesc premiul I, 60 600
100
10
= elevi primesc premiul al II-lea,
90 600
100
15
= elevi primesc premiul al III-lea i 120 600
100
20
= elevi primesc
premii speciale i meniuni.
Apoi, 30 120
100
=
x
, de unde 25 = x , deci 25% din numrul elevilor care au
primit meniuni i premii speciale reprezint numrul elevilor care au primit premiul I.
Model 2. Pentru a cumpra un tricou, o persoan pltete 150000lei, ceea ce
reprezint 30% din suma pe care o are. Ce sum are persoana?
41
Soluie. tim c 150000
100
30
= s , unde s este suma pe care o are persoana.
De aici rezult c
30
100
150000 = s , deci 500000 = s , persoana deine suma de
500000lei.


Probleme rezolvate

R4.4.1. O suprafa de 150ha este arat n trei zile, astfel: n prima zi 40% din
suprafa, a doua zi 30% din rest, iar a treia zi ce a mai rmas.
a) Cte hectare s-au arat zilnic?
b) Ce procent din ntreaga suprafa s-a arat a doua zi? Dar a treia zi?
Soluie. a) n prima zi s-au arat 60 150
100
40
= ha. Restul dup prima zi este
150-60=90ha. n a doua zi s-au arat 27 90
100
30
= ha, iar a treia zi restul, adic 90-
27=63ha.
b) Avem 27 150
100
=
x
, de unde rezult c 18 = x , deci a doua zi s-a arat
18% din suprafaa total.
La fel, 63 150
100
=
y
, de unde rezult 42 = y , deci a treia zi s-a arat 42% din
suprafaa total (sau 100%-40%-18%).
R4.4.2. Dup ce un turist a parcurs 38% dintr-un drum, constat c i-au mai
rmas de parcurs cu 4,8 km mai mult dect a parcurs. Ce lungime are drumul i ct a
parcurs turistul?
Soluie. Dac dintr-un drum se parcurg 38%, rezult c rmne din el de
parcurs 62%, deci 4,8km reprezint diferena dintre partea rmas i partea parcurs,
deci 24% din drum. Avem 8 , 4
100
24
= x , unde x este lungimea drumului. Rezult
20 = x , drumul are o lungime de 20km.
Turistul a parcurs 6 , 7 20
100
38
= km.
R4.4.3. Dup dou reduceri consecutive de preuri, prima de 10%, iar a doua
de 20%, un obiect cost 153000lei. Care a fost preul iniial al acestui obiect?
Soluie. Notm cu x preul iniial al obiectului. Prima reducere de pre este
x
100
10
i preul obiectului dup prima reducere este de x
100
90
; a doua reducere este
42
de x x
100
18
100
90
100
20
= , iar dup a doua reducere costul obiectului este
x x x
100
72
100
18
100
90
= , ceea ce reprezint 153000lei.
Avem 153000
100
72
= x , de unde rezult 212500 = x , deci preul iniial al
obiectului a fost 212500lei.
Remarc. Problema poate fi rezolvat i folosind metoda mersului invers.
Preul final, 153000lei reprezint 80% din preul obiectului dup prima ieftinire. Se
poate calcula preul dup prima ieftinire 191250
100
80
: 153000 = lei. Preul de
191250lei reprezint 90% din preul iniial. Calculm preul iniial
212500
100
90
: 191250 = lei.
R4.4.4. Un autocar are de parcurs un traseu n patru etape, astfel: n prima
etap parcurge 30% din traseu, n a doua etap parcurge 20% din rest, n a treia etap
25% din noul rest i i mai rmn pentru a patra etap 126km de parcurs. Ce lungime
are drumul?
Soluie. Se noteaz cu x lungimea drumului. n prima etap se parcurge
x
100
30
, rest x
100
70
; n a doua etap se parcurge x x
100
14
100
70
100
20
= , rest
x x x
100
56
100
14
100
70
= ; n a treia etap se parcurge x x
100
14
100
56
100
25
= , rest
x x x
100
42
100
14
100
56
= , ceea ce reprezint 126km. Avem x
100
42
=126, de unde
42
100
126 = x , 300 = x . Lungimea drumului a fost de 300km.
R4.4.5. Numrul bc reprezint 4% din numrul abc . S se calculeze
c b a + + ) 0 ( b .
Soluie. tim c abc bc =
100
4
, deci ) 100 (
25
1
bc a bc + = , de unde rezult c
bc a bc + =
25
1
4 , adic a bc 4
25
24
= . De aici se deduce c 25 M bc , pentru c 4a este
natural.
Dac 25 = bc rezult c 24=4a, deci a=6, 13 = + + c b a . Dac 50 = bc sau
75 = bc nu se obine a cifr.


43
4.5. Titlul unui aliaj

Definiie. Titlul unui aliaj este raportul dintre masa metalului preios coninut
de aliaj i masa aliajului.
aliajului masa
pretios metalului masa
aliajului Titlul = , deci
M
m
T = .
Observaie. Asemntor titlului unui aliaj, se poate defini concentraia unei
soluii (amestec).
Concentraia soluiei (amestecului)
ui) (amestecul solutiei masa
substantei masa
=
Model.
1. Se face un aliaj, topind la un loc, 16g aur i 234g cupru. Care este titlul
aliajului?
Soluie.
aliajului masa
pretios metal masa
aliajului Titlul = , deci
250
16
234 16
16
=
+
= T , de unde
rezult titlul aliajului 0,064.
2. Concentraia de sare dintr-o soluie este 17%. Ce cantitate de sare se gsete
n 27,5kg de soluie?
Soluie. Concentraia soluiei reprezint raportul dintre masa substanei i masa
soluiei. Avem
5 , 27 100
17 x
= , de unde 5 , 27 17 100 = x , deci 675 , 4 = x . n 27,5kg
soluie se afl 4,675g sare.


Probleme de amestec i aliaje

Frecvent n practic se ntlnesc probleme de acest tip. n funcie de datele i
cerinele lor n general, aceste probleme se mpart n dou categorii.


Probleme de amestec i aliaj de categoria I

n aceste probleme se cunosc:
a) cantitile care se amestec:
n
m m m ,..., ,
2 1

b) calitile lor:
n
c c c ,..., ,
2 1
.
Se cere: c) calitatea amestecului.
Calitatea diverselor produse, substane, aliaje etc. care se amestec, se exprim
prin: grade de temperatur, lei, grade de trie sau n cazul aliajelor prin titlu.
Teorema 4.5.1. Dac amestecm produse de calitile
n
c c c ,..., ,
2 1
n
cantitile
n
m m m ,..., ,
2 1
) ( N n , atunci calitatea amestecului este dat de relaia:
44
n
n n
m m m
c m c m c m
C
+ + +
+ + +
=
...
...
2 1
2 2 1 1
(1)
Demonstraie. Vom demonstra teorema n ipoteza c produsele respective sunt
aliaje cu titlurile
n
t t t ,..., ,
2 1
(deci
1 1
t c = ,
2 2
t c = ,...,
n n
t c = ) i n cantitile
n
m m m ,..., ,
2 1
. Deci s artm c titlul noului aliaj este:
n
n n
m m m
t m t m t m
T
+ + +
+ + +
=
...
...
2 1
2 2 1 1

Fie
1
'
1
1
m
m
t = ,
2
'
2
2
m
m
t = ,...,
2
'
m
m
t
n
n
= , unde
' '
2
'
1
,..., ,
n
m m m sunt cantitile de
metal preios din fiecare aliaj. Masa total a metalului preios din aliajul obinut prin
topire la un loc a aliajelor date este:
n n n
t m t m t m m m m m + + + = + + + = ... ...
2 2 1 1
' '
2
'
1

Masa total a aliajului nou obinut este
n
m m m M + + + = ...
2 1
. Deci titlul
noului aliaj este:
n
n n
m m m
t m t m t m
M
m
T
+ + +
+ + +
= =
...
...
2 1
2 2 1 1

Observaii. 1) Expresia (1) exprim media aritmetic ponderat a numerelor
n
c c c ,..., ,
2 1
care au ponderile
n
m m m ,..., ,
2 1
.
2) Media aritmetic ponderat se obine de fapt ca o medie aritmetic obinuit
innd seama c fiecare numr intr n aceast medie cu o anumit pondere.
3) Media aritmetic a unor numere este o medie aritmetic ponderat n care
fiecare pondere este egal cu 1.


Probleme de amestec i aliaj de categoria a II-a

n aceste probleme se cunosc:
a) calitile produselor care se amestec
b) calitatea amestecului
c) cantitatea total a amestecului.
Se cer: d) cantitile care se amestec.
Teorema 4.5.2. Dac amestecm dou produse de caliti
1
c , respectiv
2
c , n
cantitile
1
m , respectiv
2
m i obinem un amestec de calitate c, atunci are loc relaia:
2
1
1
2
m
m
c c
c c
=

(2)
45
Demonstraie. Din teorema (1) obinem
2 1
2 2 1 1
m m
c m c m
c
+
+
= , care prin nlocuire
n relaia (2) conduce la o propoziie adevrat:
2
1
2 1 2
2 1 1
2 2 1 1 1 2 1 1
2 2 2 1 2 2 1 1
2 1
2 2 1 1
1
2
2 1
2 2 1 1
1
2
) (
) (
m
m
c c m
c c m
c m c m c m c m
c m c m c m c m
m m
c m c m
c
c
m m
c m c m
c c
c c
=

=
+
+
=
+
+

+
+
=




Probleme rezolvate

R4.5.1. Se amestec 5kg de bomboane cu preul 54000lei/kg cu 2kg de
bomboane cu preul de 48000lei/kg i cu 3kg de bomboane cu preul de 66000lei/kg.
Ct este preul unui kilogram de bomboane ce rezult n urma amestecului celor trei
caliti de bomboane?
Soluie. Folosim relaia (1) i obinem preul unui kilogram de amestec:
56400
3 2 5
66000 3 48000 2 54000 5
=
+ +
+ +
lei.
R4.5.2. Un aliaj de fier i nichel are titlul de 0,600, iar un alt aliaj din aceleai
metale are titlul 0,250. Se topesc aceste aliaje mpreun i rezult un alt aliaj cu masa
de 14kg. Ct este masa fiecrui aliaj, dac titlul noului aliaj este 0,300?
Soluie. Vom folosi formula (2), unde 300 , 0 = c , 600 , 0
1
= c , 250 , 0
2
= c ,
1
m este masa primului aliaj,
2
m este masa celui de al doilea aliaj. Deci:
2
1
300 , 0 600 , 0
250 , 0 300 , 0
m
m
=

. Obinem
6
1
2
1
=
m
m
. tiind c 14
2 1
= + m m i
6
1
2
1
=
m
m
,
obinem 2
1
= m kg i 12
2
= m kg.
Remarc. Problema se poate rezolva i cu ajutorul formulei (1). Fie
2 1
, m m
masele celor dou aliaje folosite. Vom avea 300 , 0
250 , 0 600 , 0
2 1
2 1
=
+
+
m m
m m
i
14
2 1
= + m m . Dac
1 2
14 m m = , avem 14 3 , 0 25 , 0 ) 14 ( 6 , 0
1 1
= + m m , de
unde 7 , 0 25 , 0 6 , 0
1 1
= m m , deci rezult 2
1
= m i 12
2
= m .
R4.5.3. Se topesc mpreun dou aliaje formate din aceleai metale, care au
masele de 3kg i respectiv 2kg. Titlul primului aliaj este 0,150, iar titlul noului aliaj
este 0,400. Aflai titlul celui de al doilea aliaj.
Soluie. Aplicm formula (1), unde 3
1
= m kg, 150 , 0
1
= t , 2
2
= m kg i
400 , 0 = T . Avem:
2 1
2 2 1 1
m m
t m t m
T
+
+
= , iar prin nlocuire se obine:
46
400 , 0
2 3
2 150 , 0 3
2
=
+
+ t
. Efectund calculele 2 45 , 0 2
2
= + t , de unde rezult c
775 , 0
2
= t . Titlul celui de al doilea aliaj este 0,775.
R4.5.4. O soluie de ap cu alcool cntrete 600g i are concentraia de 0,250.
Ct alcool trebuie s adugm pentru a se obine o soluie cu concentraia de 0,400?
Soluie. Se calculeaz cantitatea de alcool existent n 600g soluie, innd cont
de definiia concentraiei (raport dintre masa alcoolului i masa soluiei). Avem:
600
250 , 0
a
= , de unde 150 = a g alcool. Notm cu x cantitatea de alcool care se
adaug pentru a obine o soluie de concentraie 0,400 i avem:
x
x
+
+
=
600
150
400 , 0 , de
unde 240 4 , 0 150 + = + x x , deci 90 6 , 0 = x , iar 150 = x . Trebuie s adugm 150g
de alcool pentru a obine o soluie de concentraie 0,400.
R4.5.5. Un inel din aur de 14 carate are 6g. Printr-o nou prelucrare inelul are
18 carate. S se afle masa inelului dup prelucrare.
Soluie. Facem precizarea c n tehnic, atunci cnd metalul preios dintr-un
aliaj este aurul, titlul se exprim n carate (k). Aurul pur are titlul 24k, deci dac un
aliaj are titlul 18k, nseamn c din ntreaga mas a aliajului 18 pri sunt aur, iar 6
pri sunt din metal nepreios; titlul este 750 , 0
24
18
= sau 18k.
n cazul acestei probleme se pot ivi dou situaii:
a) Printr-un procedeu oarecare se separ metalul nepreios din coninutul
inelului i se ndeprteaz din acesta o cantitate, astfel nct aliajul respectiv s aib
18k.
Fie x cantitatea de metal nepreios care se ndeprteaz pentru ca inelul s aib
titlul de 18k. Inelul conine: 3,5 g 6
24
14
= g aur. Deci, 5 , 3
24
18
) 6 ( = x , de unde se
obine
3
1
1 = x g. Inelul va cntri
3
2
4
3
1
1 6 = g.
b) Se adaug aur pur astfel nct aliajul obinut s aib titlul de 18k. Fie y
cantitatea de aur pur ce trebuie adugat. Deci, y y + = + 5 , 3
24
18
) 6 ( . Rezolvnd
aceast ecuaie se obine 4 = y . Inelul va avea n final masa 6+4=10g.


4.6. Proporii

Definiie. Egalitatea a dou rapoarte se numete proporie.
Termenii celor dou rapoarte se numesc termenii proporiei.
Orice proporie are patru termeni.
47
Forma general a unei proporii este:
d
c
b
a
= , Q d c b a , , , , 0 b , 0 d .
Termenii a i d se numesc extremii proporiei.
Termenii b i c se numesc mezii proporiei.
Proprietatea fundamental a proporiilor: n orice proporie produsul extremilor
este egal cu produsul mezilor.
Aflarea unui termen necunoscut al unei proporii: fiind dat proporia
d
c
b
a
= ,
conform proprietii fundamentale a proporiilor c b d a = , de unde rezult c
d
bc
a = ,
a
bc
d = ,
c
ad
b = i
b
ad
c = .
Deci:
extrem celalalt
mezilor produsul
extrem un =

mez celalalt
extremilor produsul
mez un =
Definiie. Unul dintre extremii sau mezii, egali ntre ei, ai unei proporii se
numete media proporional (geometric) a celorlali doi termeni.


Proporii derivate cu aceeai termeni

Regul. Dac ntr-o proporie se schimb extremii ntre ei lsnd mezii
neschimbai, se obine tot o proporie, numit proporie derivat cu aceiai termeni ca
proporia iniial.
Din proporia
d
c
b
a
= , aplicnd regula de mai sus obinem proporia
a
c
b
d
= cu
aceiai termeni ca proporia iniial.
Regul. Dac ntr-o proporie se schimb mezii ntre ei lsnd extremii
neschimbai, se obine tot o proporie, numit proporie derivat cu aceiai termeni ca
proporia iniial.
Din proporia
d
c
b
a
= , aplicnd regula de mai sus obinem proporia
d
b
c
a
= cu
aceiai termeni ca proporia iniial.
Regul. Dac ntr-o proporie se schimb extremii ntre ei i mezii ntre ei, se
obine tot o proporie, numit proporie derivat cu aceiai termeni ca proporia iniial.
Din proporia
d
c
b
a
= , aplicnd regula de mai sus obinem proporia
a
b
c
d
= cu
aceiai termeni ca proporia iniial.
Remarc. Ultima regul de obinere a proporiilor derivate cu aceiai termeni
se mai poate enuna i astfel: dac ntr-o proporie se inverseaz rapoartele se obine tot
o proporie numit proporie derivat cu aceiai termeni ca proporia iniial.
48
Observaie. n general, fiind date patru numere distincte d c b a , , , , care
formeaz proporia
d
c
b
a
= , cu aceste numere se mai pot forma proporiile:
1)
a
c
b
d
= ,
d
b
c
a
= .
a
b
c
d
= i
2)
c
d
a
b
= ,
b
d
a
c
= ,
c
a
d
b
= ,
b
a
d
c
= .
Ultimele proporii sunt identice cu cele de la 1), datorit simetriei relaiei de
egalitate.
Model. Scriei toate proporiile cu termenii: 3, 6, 7, 14.
Soluie. Se constat c 314=67. Dac 3 i 14 sunt extremi, iar 6 i 7 sunt
mezi, avem
14
7
6
3
= . Obinem:

3
7
6
14
= (prin schimbarea extremilor ntre ei)

14
6
7
3
= (prin schimbarea mezilor ntre ei)

3
6
7
14
= (prin inversarea rapoartelor)
Dac 3 i 14 sunt mezi, iar 6 i 7 sunt extremi, avem
7
14
3
6
= . Obinem:

6
14
3
7
= (prin schimbarea extremilor ntre ei)

7
3
14
6
= (prin schimbarea mezilor ntre ei)

6
3
14
7
= (prin inversarea rapoartelor).


Proporii derivate cu ali termeni

Fie proporia
d
c
b
a
= . Conform proprietii fundamentale a proporiilor, avem
c b d a = .
Regula 1. Dac amplificm unul din rapoartele unei proporii cu un numr,
diferit de zero, obinem o proporie cu ali termeni.
49
Avem c b d a = . nmulind ambii membri ai egalitii cu numrul n, obinem
bcn adn = , de unde rezult
dn
cn
b
a
= sau
d
c
bn
an
= .
Prin procedeul indicat de regula 1 se poate obine o infinitate de proporii cu
ali termeni dect cei ai proporiei iniiale.
Exemplu. Fie proporia
12
3
20
5
= . Prin amplificarea primului raport cu 2,
obinem
12
3
40
10
= , o proporie cu ali termeni. Prin amplificarea celui de al doilea
raport cu 10, obinem
120
30
20
5
= , o proporie cu ali termeni.
Regula 2. Dac simplificm unul din rapoartele unei proporii cu un numr,
diferit de zero, obinem o proporie cu ali termeni.
Fie proporia
d
d
b
a
= , avem c b d a = . nmulim ambii membri ai egalitii
cu
n
1
) 0 ( n , obinem
n
c b
n
d a
=

, ceea ce se poate scrie c


n
b
d
n
a
= sau
d
c
n b
n a
=
:
:
. Asemntor,
n d
n c
b
a
:
:
= .
Exemplu. Fie proporia
10
6
15
9
= . Prin simplificarea primului raport cu 3,
obinem
10
6
5
3
= , o proporie cu ali termeni. Prin simplificarea celui de al doilea
raport cu 2, obinem
5
3
15
9
= , o proporie cu ali termeni.
Regula 3. Dac nmulim ambii numrtori (sau ambii numitori) ai unei
proporii cu un numr, diferit de zero, obinem tot o proporie, dar cu ali termeni.
Fie proporia
d
c
b
a
= , avem c b d a = . nmulim ambii membri ai proporiei
cu n i obinem bcn adn = , de unde prin nmulirea ambilor membri cu
bd
1
i
realizarea simplificrilor, obinem
bd
bcn
bd
adn
= , adic
d
cn
b
an
= .
Asemntor din bc ad = , prin nmulirea ambilor membri cu
bdn
1
i
efectuarea simplificrilor, obinem
bdn
bc
bdn
ad
= , adic
dn
c
bn
a
= .
50
Exemplu. Fie proporia
25
10
5
2
= .
Prin nmulirea numrtorilor cu 3, obinem
25
30
5
6
= , iar prin nmulirea
numitorilor cu 4, obinem
100
10
20
2
= , proporii derivate cu ali termeni.
Regula 4. Dac mprim ambii numrtori (sau ambii numitori) ai unei
proporii cu un numr, diferit de zero, obinem tot o proporie, dar cu ali termeni.
Fie proporia
d
c
b
a
= , avem c b d a = . nmulind ambii membri cu
n
1

) 0 ( n , obinem
n
bc
n
ad
= , ceea ce se poate scrie
n
c
b d
n
a
= . Dup nmulirea
ambilor membri cu
bd
1
i realizarea simplificrilor, obinem
bd
n
c
b
bd
d
n
a

, adic
d
n c
b
n a : :
= .
Asemntor din bc ad = , nmulind ambii membri cu
n
1
, ) 0 ( n , obinem
c
n
b
n
d
a = . Dup nmulirea ambilor membri cu
n
d
n
b

1
i efectuarea simplificrilor
obinem:
n
d
n
b
c
n
b
n
d
n
b
n
d
a

, adic
n d
c
n b
a
: :
= .
Exemplu. Fie proporia
12
8
6
4
= .
Prin mprirea numrtorilor cu 4, obinem
12
2
6
1
= , iar prin mprirea
numitorilor cu 3, obinem
4
8
2
4
= , proporii derivate cu ali termeni.
Observaii. Cu ajutorul celor patru reguli se pot obine o infinitate de proporii
derivate cu ali termeni dect ai proporiei iniiale.
Fie proporia
d
c
b
a
= . Conform celor patru reguli se pot obine urmtoarele
proporii derivate cu ali termeni:
51
dn
cn
b
a
= ;
d
c
bn
an
= ;
d
c
n b
n a
=
:
:
,
n d
n c
b
a
:
:
= ,
d
cn
b
an
= ;
dn
c
bn
a
= ;
d
n c
b
n a : :
= ;
n d
c
n b
a
: :
= ) 0 ( n .
Proprietate. Fiind dat proporia
d
c
b
a
= , din ea se pot deduce urmtoarele
proporii derivate cu ali termeni:
P.1.
c d
c
a b
a
+
=
+

Fie proporia
d
c
b
a
= , avem c b d a = , conform proprietii fundamentale a
proporiilor. Adunnd la ambii membri produsul ac, rezult ac bc ac ad + = + , de
unde scond factor comun, ) ( ) ( a b c c d a + = + . mprind ambii membri cu
) )( ( c d a b + + avem
) )( (
) (
) )( (
) (
c d a b
a b c
c d a b
c d a
+ +
+
=
+ +
+
, adic
c d
c
a b
a
+
=
+
.
P.2.
d
d c
b
b a +
=
+
. Se demonstreaz asemntor, adunnd produsul bd n
ambii membri ai egalitii bc ad = . Avem bd bc bd ad + = + , scoatem factor comun
) ( ) ( d c b b a d + = + , iar dup nmulirea ambilor membri cu
bd
1
, obinem
bd
d c b
bd
b a d ) ( ) ( +
=
+
, adic
d
d c
b
b a +
=
+
.
P.3.
d c
c
b a
a

( , 0 b a 0 d c ). Se demonstreaz asemntor
celorlalte. Se scad din ac ambii membri ai egalitii bc ad = , rezult
bc ac ad ac = , scond factor comun obinem ) ( ) ( b a c d c a = . mprind
ambii membri cu ) )( ( b a d c , avem
) )( (
) (
) )( (
) (
b a d c
b a c
b a d c
d c a


=


, adic
d c
c
b a
a

.
P.4.
d
d c
b
b a
=

. Se demonstreaz scznd produsul bd din ambii membri ai


egalitii bc ad = , rezult bd bc bd ad = , scond factor comun avem
) ( ) ( d c b b a d = , iar dup mprirea ambilor membri cu bd, avem
bd
d c b
bd
b a d ) ( ) (
=

, adic
d
d c
b
b a
=

.
52
P.5.
d c
d c
b a
b a

+
=

+
( 0 b a , 0 d c ). Se demonstreaz mprind
membru cu membru egalitile de la P.2 i P.4, adic:
d
d c
d
d c
b
b a
b
b a

+
=

+
, care se poate
scrie
d c
d
d
d c
b a
b
b
b a

+
=

+
, adic
d c
d c
b a
b a

+
=

+
.
P.6.
d c
d c
b a
b a
+

=
+

( 0 +b a , 0 + d c ). Se demonstreaz inversnd
rapoartele n proporia de la P.5.
P.7.
d c
b a
d c
b a

=
+
+
( , 0 + d c 0 d c ). Se demonstreaz schimbnd mezii
ntre ei n proporia de la P.5.
Model. Fie proporia
4
6
8
12
= . S se obin proporii derivate cu ali termeni.
Soluie.
Aplicnd R.1 (n=3), avem
4 3
6 3
8
12

= i obinem
12
18
8
12
= .
Aplicnd R.2 (n=4), avem
4
6
4 : 8
4 : 12
= i obinem
4
6
2
3
= .
Aplicnd R3. (n=5), avem
4
5 6
8
5 12
=

i obinem
4
30
8
60
= .
Aplicnd R.4 (n=2), avem
2 : 4
6
2 : 8
12
= i obinem
2
6
4
12
= .
Aplicnd P.1, avem
6 4
6
12 8
12
+
=
+
i obinem
10
6
20
12
= .
Aplicnd P2., avem
4
4 6
8
8 12 +
=
+
i obinem
4
10
8
20
= .
Aplicnd P.3, avem
4 6
6
8 12
12

i obinem
2
6
4
12
= .
Aplicnd P.4 avem
4
4 6
8
8 12
=

i obinem
4
2
8
4
= .
Aplicnd P.5 avem
4 6
4 6
8 12
8 12

+
=

+
i obinem
2
10
4
20
= .
Aplicnd P.6, avem
4 6
4 6
8 12
8 12
+

=
+

i obinem
10
2
20
4
= .
53
Aplicnd P.7, avem
4 6
8 12
4 6
8 12

=
+
+
i obinem
2
4
10
20
= .


Probleme rezolvate

R4.6.1. Se d 6 , 0 =
b
a
. S se afle
b
b a
3
3 2 +
.
Soluia 1. Din
5
3
=
b
a
, prin nmulirea numrtorilor cu 2, vom avea
5
6 2
=
b
a
,
iar prin nmulirea numitorilor cu 3, obinem
15
6
3
2
=
b
a
sau
5
2
3
2
=
b
a
. Adunm
numitorii la numrtor i obinem
5
5 2
3
3 2 +
=
+
b
b a
, de unde
5
7
3
3 2
=
+
b
b a
.
Soluia 2. Din
5
3
=
b
a
, schimbnd mezii ntre ei obinem k
b a
= =
5 3
(s-a notat
prin k valoarea rapoartelor
3
a
i
5
b
). Din k
a
=
3
rezult k a 3 = , iar din k
b
=
5
rezult
k b 5 = . Atunci:
5
7
15
21
5 3
5 3 3 2
3
3 2
= =

+
=
+
k
k
k
k k
b
b a
.
Soluia 3. Din
5
3
=
b
a
rezult
5
3b
a = i atunci:
5
7
3
1
5
21
3
3
5
6
3
3
5
3
2
3
3 2
= =
|
.
|

\
|
+
=
+
=
+
b
b
b
b
b
b
b a
.
R4.6.2. S se afle numerele naturale x i y, diferite de 0, astfel ca
3 2
y x
= i
3
24
) ( 5 4
=
+ + + y x y x
.
Soluia 1. Din 3
24
) ( 5 4
=
+ + + y x y x
, obinem prin efectuarea calculelor de la
numrtor 3
24
6 9
=
+ y x
sau 3
24
) 2 3 ( 3
=
+ y x
, adic 3
8
2 3
=
+ y x
, de unde rezult c
24 2 3 = + y x . Dar
3 2
y x
= i conform proprietii fundamentale a proporiilor
54
y x 2 3 = , care se nlocuiete n relaia precedent obinndu-se 24 2 2 = + y y sau
6 = y . Dar
3
2y
x = , deci 4 = x .
Soluia 2. Notm k
y x
= =
3 2
deci obinem k x 2 = i k y 3 = . Atunci
3
24
) ( 5 4
=
+ + + y x y x
devine 3
24
) 3 2 ( 5 3 2 4
=
+ + + k k k k
, iar dup efectuarea
calculelor obinem 3
24
36
=
k
, de unde 2 = k . Din k x 2 = i k y 3 = vom obine
4 = x , 6 = y .
R4.6.3. S se afle trei numere, tiind c raportul dintre primul i al doilea este
0,(6), raportul dintre al doilea i al treilea este 0,8(3), iar produsul dintre primul i al
treilea numr este 9331,2.
Soluie. Notm n ordine cele trei numere cu x, y, z. Din datele problemei,
obinem
3
2
=
y
x
,
6
5
=
z
y
i 2 , 9331 = xz . Din
3
2
=
y
x
i
5
6
=
y
z
, prin nmulire
membru cu membru se obine
5
4
2
=
y
xz
, de unde rezult
5
4 2 , 9331
2
=
y
, deci
11664
2
= y , sau
2 3 2 2
) 3 2 ( = y . Obinem 108 = y sau 108 = y . Din
3
2
=
y
x
,
rezult
3
2y
x = , deci 72 = x sau 72 = x . Din
6
5
=
z
y
, rezult
5
6y
z = , deci
6 , 129 = z sau 6 , 129 = z .
R4.6.4. tiind c 5
2 3
9 20
=

+
a b
b a
, s se arate c a este 20% din b.
Soluie. Din relaia dat rezult c ) 2 3 ( 5 9 20 a b b a = + , iar dup efectuarea
calculelor a b b a 10 15 9 20 = + , de unde b a 6 30 = sau
30
6
=
b
a
, deci
5
1
=
b
a
. Se
schimb mezii ntre ei i se obine k
b a
= =
5 1
, de unde k a = i k b 5 = . Vom avea:
a b
x
=
100
, adic k k
x
= 5
100
, de unde 20 = x , deci 20% din a reprezint b.
R4.6.5. S se afle ariile a dou dreptunghiuri, tiind c raportul lungimilor lor
este
3
4
, raportul limilor lor este
9
7
, iar diferena ariilor este 4.
55
Soluie. Notm L i L' lungimile celor dou dreptunghiuri i l, l' limile celor
dou dreptunghiuri. Avem raportul lungimilor
3
4
'
=
L
L
i raportul limilor
9
7
'
=
l
l
.
Diferena ariilor celor dou dreptunghiuri este 4 ' ' = l L Ll . Din
3
4
'
=
L
L
i
9
7
'
=
l
l
,
prin nmulirea membru cu membru, obinem
27
28
' '
=
l L
Ll
, apoi facem proporii derivate
27
27 28
' '
' '
=

l L
l L Ll
, adic
27
1
' '
4
=
l L
, deci 108 ' ' = l L . Din 4 108 = Ll , rezult
112 = Ll . Deci ariile celor dou dreptunghiuri sunt 112 i 108.
R4.6.6. Suma a dou fracii cu acelai numrtor este
15
1
1 . Raportul
numitorilor este
3
1
. S se afle cele dou fracii.
Soluie. Fie
b
a
i
c
a
cele dou fracii ) 0 , ( c b . Avem
15
16
= +
c
a
b
a
, de unde
rezult c
15
16 ) (
=
+
bc
c b a
. Raportul numitorilor este
3
1
=
c
b
, de unde proporia
3
4
=
+
c
c b
. Prin nlocuire n relaia dinainte, avem
15
16
3
4
=
b
a
, de unde
5
4
=
b
a
. Suma
celor dou fracii este
15
16
, deci
5
4
15
16
=
c
a
,
15
4
=
c
a
. Fraciile sunt
5
4
i
15
4
.


4.7. ir de rapoarte egale

Definiie. Un ir de rapoarte cu aceeai valoare, scrise sub forma
... = = =
f
e
d
c
b
a
, se numete ir de rapoarte egale.
Observaii. 1) Orice pereche de rapoarte din ir formeaz o proporie.
2) Amplificnd succesiv un raport cu mai multe numere diferite de zero, se
obine un ir de rapoarte egale:
... = = =
bk
ak
bn
an
b
a

Fie irul de rapoarte egale
bk
ak
bn
an
b
a
= = . S considerm raportul dintre suma
numrtorilor i suma numitorilor
bk bn b
ak an a
+ +
+ +
. Scoatem factorul comun a la
56
numrtor i b la numitor i obinem
) 1 (
) 1 (
k n b
k n a
+ +
+ +
, care se simplific i rezult
b
a
.
Deci,
bk bn b
ak an a
bk
ak
bn
an
b
a
+ +
+ +
= = = .
Proprietatea irului de rapoarte egale. ntr-un ir de rapoarte egale, raportul
dintre suma numrtorilor i suma numitorilor este egal cu fiecare din celelalte
rapoarte.
n general, dac
f
e
d
c
b
a
= = , atunci
f d b
e c a
f
e
d
c
b
a
+ +
+ +
= = = .
Model. tiind c
4
3
= = =
f
e
d
c
b
a
, s se calculeze:
a)
f d b
e c a
+ +
+ +
; b)
f d b
e c a
5 4 3
5 4 3
+ +
+ +
; c)
2 2 2
2 2 2
f d b
e c a
+ +
+ +
.
Soluie. a) Dac
f
e
d
c
b
a
= = , atunci
f d b
e c a
f
e
d
c
b
a
+ +
+ +
= = = , dar
4
3
=
b
a
,
deci
4
3
=
+ +
+ +
f d b
e c a
.
b) Dac
f
e
d
c
b
a
= = , atunci prin amplificarea primului raport cu 3, al celui de
al doilea cu 4 i al celui de al treilea cu 5, se obine
f
e
d
c
b
a
5
5
4
4
3
3
= = , de unde rezult c
f d b
e c a
f
e
d
c
b
a
5 4 3
5 4 3
5
5
4
4
3
3
+ +
+ +
= = = , dar
4
3
3
3
= =
b
a
b
a
, deci
4
3
5 4 3
5 4 3
=
+ +
+ +
f d b
e c a
.
c) Dac
4
3
= = =
f
e
d
c
b
a
, rezult c
16
9
2
2
2
2
2
2
= = =
f
e
d
c
b
a
, prin ridicarea la
ptrat a fiecrui raport. Rezult, aplicnd proprietatea irului de rapoarte egale c
2 2 2
2 2 2
2
2
2
2
2
2
f d b
c b a
f
e
d
c
b
a
+ +
+ +
= = = , dar
16
9
2
2
=
b
a
, deci
16
9
2 2 2
2 2 2
=
+ +
+ +
f d b
c b a
.
Remarc. Dac k
f
e
d
c
b
a
= = = , atunci k
pf
pe
nd
nc
mb
ma
= = = , de unde
k
pf nd mb
pe nc ma
=
+ +
+ +
, 0 , , p n m .


57
Probleme rezolvate

R4.7.1. tiind c
4
5
5
3
3
2 c b a
= = i c 119 = + + c b a , s se afle a, b, c.
Soluia 1. Dac
4
5
5
3
3
2 c b a
= = , atunci
5
4
3
5
2
3
c b a
= = , de unde rezult aplicnd
proprietatea irului de rapoarte egale
30
119
30
119
30
119
119
5
4
3
5
2
3
5
4
3
5
2
3
= = =
+ +
+ +
= = =
c b a c b a
.



Din 30
2
3
=
a
, rezult a=45, din 30
3
5
=
b
, rezult b=50 i din 30
5
4
=
c
, rezult c=24.
Soluia 2. Notm valoarea comun a rapoartelor cu k i avem:
k
c b a
= = =
4
5
5
3
3
2
, de unde
2
3k
a = ,
3
5k
b = ,
5
4k
c = . nlocuind n 119 = + + c b a ,
se obine 119
5
4
3
5
2
3
= + +
k k k
, de unde se obine dup efectuarea calculelor
119
30
111
=
k
, adic k=30. Atunci 45
2
30 3
=

= a , 50
3
30 5
=

= b i 24
5
30 4
=

= c .
R4.7.2. S se determine numerele x, y, z naturale, tiind c
8 3
y x
= ,
2 6
z y
= i
a) 123 = + + z y x ; b) 25 8 5 = + z y x ; c) 6489
2 2 2
= + + z y x .
Soluie. Fiind date proporiile
8 3
y x
= i
2 6
z y
= se poate forma un ir de
rapoarte egale astfel: nmulim numitorii primei proporii cu 3 i nmulim numitorii
celei de a doua proporii cu 4, vom obine dou proporii derivate cu ali termeni i
anume,
24 9
y x
= i
8 24
z y
= , de unde rezult
8 24 9
z y x
= = .
a) Aplicnd proprietatea irului de rapoarte egale, obinem:
3
41
123
8 24 9 8 24 9
= =
+ +
+ +
= = =
z y x z y x
, de unde x=27, y=72, z=24.
58
b) Avem k
z y x
= = =
8 24 9
, k fiind valoarea comun a fiecrui raport, atunci
k x 9 = , k y 24 = , k z 8 = . Prin nlocuire n 25 8 5 = + z y x , se obine
25 64 24 45 = + k k k , adic 25 5 = k , de unde k=5. Avem 45 5 9 = = x ,
120 5 24 = = y i 40 5 8 = = z .
Remarc. Dac
8 24 9
z y x
= = , atunci prin amplificarea primului raport cu 5 i
al celui de al treilea lui 8, obinem
64
8
24 45
5 z y x
= = , de unde aplicnd proprietatea
irului de rapoarte egale se obine
5
5
25
64 24 45
8 5
64
8
24 45
5
= =
+
+
= = =
z y x z y x
.
Avem 5
9
=
x
, deci x=45, 5
24
=
y
, deci y=120 i 5
8
=
z
, deci z=40.
c) Dac
8 24 9
z y x
= = , atunci
64 576 81
2 2 2
z y x
= = i aplicnd proprietatea irului
de rapoarte egale 9
721
6489
64 576 81 64 576 81
2 2 2 2 2 2
= =
+ +
+ +
= = =
z y x z y x
. Avem 9
81
2
=
x
, x
natural, deci 3
9
=
x
, de unde x=27, 9
576
2
=
y
, y natural, deci 3
24
=
y
, de unde y=72,
9
64
2
=
z
, z natural, deci 3
8
=
z
, de unde z=24.
Remarc. Dac k
z y x
= = =
8 24 9
, atunci k x 9 = , k y 24 = , k z 8 = . Prin
nlocuire n 6489
2 2 2
= + + z y x , vom avea 6489 64 576 81
2 2 2
= + + k k k adic
6489 721
2
= k , de unde k=3. Obinem x=27, y=72, z=24.
R4.7.3. Fie
) 3 ( , 0 ) 1 ( , 0
y x
= i
) 7 ( , 0 ) 5 ( , 0
z y
= cu 0 x , 0 y , 0 z . S se
afle x, y, z, tiind c 123 = + + z y x .
Soluie. Efectund transformrile, se obine
3
1
9
1
y x
= i
9
7
9
5
z y
= . nmulim
ambii membri ai celor dou egaliti cu
9
1
i obinem
9
1
3
1
9
1
9
1
=
y x
, adic
3 1
y x
= i
59
9
7
9
1
9
5
9
1 z y
= , adic
7 5
z y
= . n egalitatea
3 1
y x
= nmulim fiecare membru cu
5
1
i
obinem
3 5
1
1 5
1 y x
= , adic
15 5
y x
= (1).
n egalitatea
7 5
z y
= , nmulim fiecare membru cu
3
1
i obinem
7 3
1
5 3
1 z y
= ,
adic
21 15
z y
= (2).
Din (1) i (2) rezult irul de rapoarte egale
21 15 5
z y x
= = i aplicnd
proprietatea irului de rapoarte egale avem
3
41
123
21 15 5 21 15 5
= =
+ +
+ +
= = =
z y x z y x
.
Deci, 3
5
=
x
, de unde x=15, 3
15
=
y
, de unde y=45 i 3
21
=
z
, de unde z=63.
R4.7.4. Fie a, b, c trei numere nenule, astfel nct:
) ( 9 5 2 c b a x c b a + + = = = .
S se determine valoarea lui x.
Soluie. Relaia dat se poate scrie
x
c b a c b a
1
9
1
5
1
2
1
+ +
= = = . Aplicnd
proprietatea irului de rapoarte egale:
9
1
5
1
2
1
9
1
5
1
2
1
+ +
+ +
= = =
c b a c b a
, rezult c
9
1
5
1
2
1 1
+ + =
x
, adic
90
73 1
=
x
, de unde
73
90
= x .
R4.7.5. S se afle numere a, b, c, tiind c
6 4 3
c b a
= = i 576 = abc .
Soluie. Notm k
c b a
= = =
6 4 3
, de unde k a 3 = , k b 4 = i k c 6 = .
nlocuind n 576 = abc , se obine 576 6 4 3 = k k k , de unde rezult 8
3
= k , deci
k=2. Avem 6 2 3 = = a , 8 2 4 = = b , 72 12 6 = = c .


60
4.8. Proporionalitate direct. Proporionalitate invers

Definiie. ntre dou mulimi finite de numere exist o proporionalitate
direct, dac se poate forma un ir de rapoarte egale, diferite de zero, astfel nct
numrtorii rapoartelor s fie elementele unei mulimi, iar numitorii elementele
celeilalte mulimi.
Exemplu. ntre mulimile {2,6,4} i {10,30,20} se stabilete o
proporionalitate direct, deoarece
20
4
30
6
10
2
= = .
Observaie. Dac elementele unei mulimi A finite de numere se pot obine
prin nmulirea elementelor unei mulimi B cu un numr dat n (n0), atunci ntre cele
dou mulimi exist o proporionalitate direct.
ntr-adevr, fie mulimea B={a,b,c}. Prin nmulirea elementelor ei cu numrul
n (n0), obinem mulimea A={an,bn,cn}. Cu elementele celor dou mulimi, A i B, se
poate forma un ir de rapoarte egale
cn
c
bn
b
an
a
= = (valoarea rapoartelor este
n
1
);
deci ntre cele dou mulimi A i B am stabilit o proporionalitate direct.
Exemple. 1) ntre mulimea ciocolatelor i mulimea costurilor lor se stabilete
o proporionalitate direct.
2) ntre viteza de deplasare i spaiul parcurs de un mobil n micare uniform,
se stabilete o proporionalitate direct.
3) ntre spaiul parcurs de un mobil cu vitez constant i timpul n care se
efectueaz deplasarea se stabilete o proporionalitate direct.
4) ntre numrul de robinete cu acelai debit i volumul de lichid acumulat se
stabilete o proporionalitate direct.
Model. S se determine trei numere direct proporionale cu 3, 9, 12, dac suma
lor este 40.
Soluie. Fie x, y, z cele trei numere. Vom avea
12 9 3
z y x
= = i 40 = + + z y x .
Aplicnd proprietatea irului de rapoarte egale, avem
3
5
24
40
12 9 3 12 9 3
= =
+ +
+ +
= = =
z y z z y x
. Se deduce c 5
3
5 3
=

= x , 15
3
5 9
=

= y i
20
3
5 12
=

= z .
Remarc. Se poate aplica i metoda: k
z y x
= = =
12 9 3
, de unde k x 3 = ,
k y 9 = , k z 12 = i nlocuind n 40 = + + z y x , obinem 40 24 = k , deci
3
5
= k .
Rezult 5 = x , 15 = y , 20 = z .
Definiie. ntre dou mulimi finite de numere exist o proporionalitate
invers, dac se poate forma un ir de produse egale, diferite de zero, astfel nct
61
mulimea primilor factori ai produselor s fie una din mulimi, iar mulimea celorlali
factori ai produselor s fie cealalt mulime.
Exemplu. ntre mulimile {9,12,18} i {4,3,2} se stabilete o proporionalitate
invers, deoarece 94=123=182.
Observaie. Dac mprim un numr dat, diferit de zero, cu elementele unei
mulimi finite de numere nenule, obinem o alt mulime astfel nct ntre cele dou
mulimi s existe o proporionalitate invers.
ntr-adevr, numrul n mprit succesiv la elementele mulimii A={a,b,c}, se
obine mulimea
)
`

=
c
n
b
n
a
n
B , , . Cu elementele acestor dou mulimi putem forma un
ir de produse a cror valoare este n; deci
c
n
c
b
n
b
a
n
a = = (valoarea produselor este
n); deci ntre cele dou mulimi A i B s-a stabilit o proporionalitate invers.
Exemple. 1) ntre numrul robinetelor, cu acelai debit i timpul de umplere al
unui rezervor se stabilete o proporionalitate invers.
2) ntre numrul muncitorilor i timpul de realizare a unei anumite lucrri, se
stabilete o proporionalitate invers.
3) ntre viteza constant de parcurgere a unei distane i timpul de deplasare, se
stabilete o proporionalitate invers.
4) ntre numrul de bancnote i valoarea bancnotelor cu care se pltete o
anumit sum, se stabilete o proporionalitate invers.
Remarc. ntre elementele mulimilor } , , { c b a A = i } , , { p n m B = se
stabilete o proporionalitate invers, deci p c n b m a = = . Aceast relaie este
echivalent cu:
p
c
n
b
m
a
1
:
1
:
1
: = = sau
p
c
n
b
m
a
1 1 1
= = , de unde rezult c ntre
elementele mulimilor {a,b,c} i
)
`

p n m
1
,
1
,
1
s-a stabilit o proporionalitate direct.
Model. Dou numere sunt invers proporionale cu numerele 0,2 i 0,5. Suma
dintre dublul primului numr i al doilea numr este 24. S se afle aceste numere.
Soluie. Notnd x primul numr i y al doilea numr, relaiile dintre acestea,
conform problemei sunt:
2
1
5
1
= y x i 24 2 = + y x . Avem k
y x
= =
2 5
, de unde
k x 5 = , k y 2 = i prin nlocuire n 24 2 = + y x se obine 24 2 10 = + k k , deci
2 = k . Numerele sunt 10 = x , 4 = y .


62
Probleme rezolvate

R4.8.1. Un ir de 5 numere este format astfel nct primele 3 numere sunt
direct proporionale cu 4, 5, 6, iar ultimele 3 numere sunt invers proporionale cu 4, 5,
6.
a) S se afle cele mai mici 5 numere naturale care satisfac cerinele puse.
b) S se afle cele 5 numere care satisfac condiiile cerute, dac suma lor este
476.
Soluie. a) Fie a, b, c, d, e cele 5 numere. Conform enunului
6 5 4
c b a
= = i
6
1
5
1
4
1
e d c
= = . n ultimul ir de rapoarte egale nmulim toi numitorii cu 60 i obinem
10 12 15
e d c
= = . Pentru a obine un raport comun aflm c.m.m.m.c. al numerelor 6 i 15
(numitorii lui c), care este 30. n relaia
6 5 4
c b a
= = , nmulim toi numitorii cu 30:6=5
i n relaia
10 12 15
e d c
= = nmulim toi numitorii cu 30:15=2 i obinem:
30 25 20
c b a
= = i
20 24 30
e d c
= = , de unde rezult c:
20 24 30 25 20
e d c b a
= = = = .
Cele mai mici numere naturale care satisfac aceast condiie, vor fi cele pentru care
valoarea comun a rapoartelor este 1; deci numerele cutate sunt 20, 25, 30, 24, 20.
b) Din 4
119
476
20 24 30 25 20 20 24 30 25 20
= =
+ + + +
+ + + +
= = = = =
e d c b a e d c b a
,
rezult a=80, b=100, c=120, d=96, e=80.
R4.8.2. S se determine numrul abc , tiind c numerele ab , bc , ca sunt
direct proporionale cu numerele 3, 2, 6, iar suma cifrelor numrului abc este
divizibil cu 7.
Soluie. Scriem c ab , bc , ca sunt direct proporionale cu 3, 2, 6 i apoi
aplicm proprietatea irului de rapoarte egale:
=
+ + + + +
=
+ +
+ +
= = =
11
10 10 10
6 2 3 6 2 3
a c c b b a ca bc ab ca bc ab

c b a
c b a
+ + =
+ +
=
11
) ( 11
. Dar, suma cifrelor este divizibil cu 7 i este un numr
mai mic dect 27 (suma maxim este suma cifrelor numrului 999). Aceast sum
poate fi 7, 14 sau 21.
63
a) Dac 7 = + + c b a , atunci 21 = ab , 14 = bc , 42 = ca , adic a=2, b=1,
c=4, deci 214 = abc .
b) Dac 14 = + + c b a , atunci 42 = ab , 28 = bc , 84 = ca , adic a=4, b=2,
c=8, deci 428 = abc .
c) Dac 21 = + + c b a , atunci 63 = ab , 42 = bc , 126 = ca , imposibil.
VI.R4.8.3. O sum de bani a fost distribuit la trei persoane direct proporional
cu numerele
3
1
,
5
1
,
6
1
. n acest mod o persoan constat c primete cu 46200 lei mai
mult dect dac aceeai sum s-ar fi distribuit invers proporional cu 12, 10, respectiv 15.
a) Care a fost suma de bani?
b) Ct a primit fiecare din cele trei persoane?
Soluie. Notm cu s suma total de bani, cu a, b, c sumele ce revin celor trei
persoane distribuite direct proporional cu
3
1
,
5
1
,
6
1
i cu x, y, z sumele ce revin celor
trei persoane dac ar fi distribuite invers proporional cu 12, 10, 15. Avem:
10
7
3
1
5
1
6
1
3
1
5
1
6
1
s c b a c b a
=
+ +
+ +
= = = (1) i
4
1
15
1
10
1
12
1
15
1
10
1
12
1
s z y x z y x
=
+ +
+ +
= = = (2).
Din relaia (1) rezult c
21
5s
a = .
7
2s
b = i
21
10s
c = , iar din relaia (2)
rezult c
3
s
x = ,
5
2s
y = i
15
4s
z = . Comparm sumele obinute de fiecare persoan
prin cele dou procedee de mprire:
3 21
5 s s
< ,
5
2
7
2 s s
< i
15
4
21
10 s s
> . Doar a treia
persoan primete mai mult prin mprirea sumei direct proporional cu numerele
3
1
,
5
1
,
6
1
. Deci, 46200lei reprezint diferena dintre cele dou sume, avem
46200
15
4
21
10
=
s s
. Efectund calculele obinem 46200
15 21
84
15 21
150
=

s s
, de unde
46200
15 21
66
=

s
; rezult
66
15 21 46200
= s , deci s=7002115, s=220500. Suma
iniial a fost de 220500lei.
64
b) Pentru a calcula ce sum revine fiecrei persoane, avem
52500
21
220500 5
=

= a ; 63000
7
220500 2
=

= b i 105000
21
220500 10
=

= c .
Cele trei persoane au primit 52500lei, 63000lei i 105000lei.
R4.8.4. Numerele x z z y y x + + + , , sunt direct proporionale cu numerele 4,
6, 8.
a) Aflai valoarea raportului
2 2 2
z y x
yz xz xy
+ +
+ +
.
b) Dac a,b,c{1,2,...,9}, a c b a , s se determine valorile maxime i
minime ale raportului
2 2 2
z y x
cyz bxz axy
+ +
+ +
.
Soluie. a) Avem k
x z z y y x
=
+
=
+
=
+
8 6 4
, de unde k y x 4 = + ,
k z y 6 = + i k x z 8 = + , iar prin adunare membru cu membru a celor trei egaliti
obinem k z y x 18 2 2 2 = + + , deci k z y x 9 = + + . Dac k z y x 9 = + + i
k y x 4 = + , rezult c k z 5 = . Dac k z y x 9 = + + i k z y 6 = + , rezult c
k x 3 = . Dac k z y x 9 = + + i k x z 8 = + , rezult c k y = . Se obine
35
23
25 9
5 15 3
2 2 2
2 2 2
2 2 2
=
+ +
+ +
=
+ +
+ +
k k k
k k k
z y x
yz xz xy
.
b) Valoarea maxim a raportului
2 2 2
z y x
cyz bxz axy
+ +
+ +
se obine cnd b=9, c=8 i
a=7 (deoarece xy yz xz > > ) i este
5
28
35
196
35
5 8 15 9 3 7
2
2 2 2
= =
+ +
k
k k k
.
Valoarea minim a raportului
2 2 2
c b a
cyz bxz axy
+ +
+ +
se obine cnd b=1, c=2 i a=3
(deoarece xy yz xz > > ) i este
35
34
35
5 2 15 1 3 3
2
2 2 2
=
+ +
k
k k k
.
R4.8.5. Aflai numerele a, b, c naturale, tiind c numerele c b a , ,
2 3
sunt
direct proporionale cu 8, 4, 2 i
c b a
24 1 1
= + .
Soluie. Avem
6
2 3
2 4 8
k
c b a
= = = , de unde rezult c
6 3
8k a = ,
6 2
4k b = i
6
2k c = , deci
2
2k a = ,
3
2k b = i
6
2k c = . Prin nlocuire n relaia
c b a
24 1 1
= + , se
65
obine
6 3 2
2
24
2
1
2
1
k k k
= + . Dup efectuarea calculelor obinem
6 3
2
24
2
1
k k
k
=
+
, de unde
rezult c 24 ) 1 (
3
= + k k , dar k fiind numr natural avem 3 2 ) 1 (
3 3
= + k k , deci k=2.
Se obine a=8, b=16, c=128.
R4.8.6. Se dau numerele naturale a, b, c, d astfel nct 7 5 = b a , c este 60%
din b, iar raportul dintre c i d este 1,5. S se arate c a, b, c, d sunt invers
proporionale cu numerele 0,(142857); 0,2;
3
1
; 0,5.
Soluie. Dac 7 5 = b a , atunci
5 7
b a
= . Se tie c c este 60% din b, deci
b c =
5
3
, de unde rezult c
3 5
c b
= . Avem
2
3
=
d
c
, de unde
2 3
d c
= . Se poate scrie
urmtorul ir de rapoarte egale:
2 3 5 7
d c b a
= = = . Conform definiiei proporionalitii
directe rezult c numerele a, b, c, d sunt direct proporionale cu 7, 5, 3, 2, de unde
rezult c a, b, c, d sunt invers proporionale cu numerele
2
1
,
3
1
,
5
1
,
7
1
. innd cont c
) 142857 ( , 0
7
1
= , 2 , 0
5
1
= i 5 , 0
2
1
= , avem a, b, c, d sunt invers proporionale cu
numerele 0,(142857); 0,2;
3
1
; 0,5.


4.9. Regula de trei simpl. Regula de trei compus

Regula de trei simpl

Vom considera probleme n care intervin dou mulimi de cte dou numere
ntre care exist o proporionalitate direct sau o proporionalitate invers, iar unul din
numerele unei mulimi este necunoscut.
Procedeul care se folosete pentru determinarea numrului necunoscut dintr-
una din dou mulimi, alctuite fiecare din cte dou elemente, ntre care exist o
proporionalitate direct sau invers, se numete regula de trei simpl.
Aplicarea acestui procedeu, numit regula de trei simpl, pornete de la
aezarea datelor problemei ntr-o schem, care conduce la aflarea unui termen
necunoscut dintr-o proporie (n cazul mrimilor direct proporionale) sau la aflarea
unui factor necunoscut al unui produs, cnd cunoatem produsul i cellalt factor (n
cazul mrimilor invers proporionale). Practic, schema conduce la rezolvarea unei
ecuaii cu o singur necunoscut.
66
Model 1. 18kg de mere cost 126000lei. Ct cost 13kg de mere de aceeai
calitate?
Soluie. Aceast problem poate fi rezolvat prin metoda reducerii la unitate:
1) Aflm preul unui kilogram de mere. 126000:18=7000lei.
2) Aflm preul a 13kg de mere. 700013=91000lei.
Prin regula de trei simpl, datele problemei se aeaz astfel:

18kg mere..............................126000lei
13kg mere.............................. x

Aceast schem se citete: "Dac 18kg de mere cost 126000lei, atunci 13kg
de mere vor costa x lei".
Stabilim ce fel de proporionalitate exist ntre cele dou mulimi: a cantitilor
i a costurilor. Pentru aceasta considerm mulimea cantitilor {18,13} i mulimea
costurilor {126000,x}. ntre aceste dou mulimi exist o proporionalitate direct,
deoarece putem forma un ir de rapoarte egale,
13 18
126000 x
= , valoarea lor comun
fiind tocmai preul unui kilogram de mere. Apoi aflm termenul necunoscut al
proporiei:
18
13 126000
= x , deci x=91000(lei).
Model 2. 15 muncitori pot termina o lucrare n 20 zile. n cte zile vor termina
lucrarea 25 de muncitori?
Soluie. Aceast problem poate fi rezolvat prin metoda reducerii la unitate:
1) Aflm n cte zile termin lucrarea un muncitor. 1520=300zile.
2) Aflm n cte zile termin lucrarea 25 muncitori. 300:25=12zile.
Prin regula de trei simpl datele problemei se aeaz astfel:

15 muncitori..............................20 zile
25 muncitori.............................. x

Aceast schem se citete astfel: "Dac 15 muncitori termin lucrarea n 20 de
zile, atunci 25 muncitori o vor termina n x zile".
Stabilim ce fel de proporionalitate exist ntre cele dou mulimi: a numrului
de muncitori i a numrului de zile n care ei pot termina lucrarea. Pentru aceasta
considerm mulimile {15,25} i {20,x}. ntre aceste dou mulimi exist o
proporionalitate invers, deoarece putem forma un ir de produse egale 1520=25x,
valoarea lor comun fiind tocmai numrul de zile n care un muncitor poate termina
lucrarea. Apoi, aflm factorul necunoscut:
25
20 15
= x , deci x=12(zile).
n practic, exist obiceiul ca, dup determinarea tipului de proporionalitate,
modul de aflare a necunoscutei s fie indicat, direct pe schem, printr-o sgeat care
indic nmulirea i scrierea literelor "d.p." (pentru proporionalitate direct), respectiv
"i.p." (pentru proporionalitate invers).
67
Model 1.

d.p.
18kg mere..............................126000lei

13kg mere.............................. x

lei 91000
18
13 126000
=

= x

Model 2.

i.p.
15muncitori..............................20zile

25muncitori.............................. x

zile 12
25
15 20
=

= x


Probleme rezolvate

R4.9.1. Un motociclist mergnd cu viteza de 60km/h strbate o distan n
48minute. Cu ce vitez trebuie s mearg pentru a parcurge aceeai distan n
45minute?
Soluie. Efectum transformrile:
60
48
min 48 = h
5
4
= h, 45min=
60
45
h
4
3
= h.
Prin regula de trei simpl, datele problemei se aeaz astfel:


i.p.

5
4
h..............................60km/h

4
3
h.............................. x

64
3
4
5
4
60
4
3
5
4
60
= =

= x (km/h)
68
R4.9.2. Un muncitor efectueaz 130 piese n 4 ore 20 min. Cte piese
realizeaz muncitorul n 8 ore?
Soluie. Efectum transformarea: 4 h 20 min=
60
20
4 h
3
1
4 = h.
Prin regula de trei simpl, datele problemei se aeaz astfel:
d.p.

3
1
4 h..............................130piese

8h .............................. x

240
13
3
8 130
3
13
8 130
= =

= x (piese)

Probleme propuse

P4.9.1. Dac din 80kg fin se produc 180 pini, ce cantitate de fin este
necesar pentru obinerea a 72 de pini?
P4.9.2. Trei robinete, avnd acelai debit, umplu un rezervor n 6ore. n ct
timp vor umple rezervoarele dou robinete cu acelai debit?
P4.9.3. Un copil a economisit 15 bancnote de cte 10000lei. Cte bancnote de
50000lei primete n schimbul lor?
P4.9.4. Un muncitor face n 6 ore, 108 piese. Dac lucreaz n acelai ritm cte
piese, de acelai fel, face n 5 ore?
P4.9.5. La o ferm se planificase o cantitate de furaje pentru 40 vite pe timp de
60 zile. Pentru cte zile va ajunge aceeai cantitate de furaje, dac s-au mai cumprat 8
vite?
P4.9.6. Pentru transportul lemnelor de la munte la un depozit s-au comandat
40 de vagoane cu o capacitate de 15t fiecare. S-au folosit, ns, vagoane cu o capacitate
de 20t. Cte vagoane au fost necesare?
P4.9.7. O brigad de 24 de muncitori trebuia s sape 120m de an. 4
muncitori nu au lucrat. Ci metri de an au spat ceilali muncitori?
P4.9.8. La un magazin s-a adus 500 sticle de ulei pentru care trebuia s se
ncaseze 19.000.000lei. Dup ce s-au vndut 300 sticle, ce sum urmeaz s se
ncaseze?
P4.9.9. La acoperirea unei podele erau necesari 50m linoleum lat de 0,75m.
Ci metri linoleum sunt necesari pentru acoperirea aceleiai podele, dac se folosete
linoleum lat de 1,2m?
P4.9.10. Din 120kg ap de mare se obin 300g sare. Ce cantitate de ap de
mare este necesar pentru a obine 15kg sare?


69
Rspunsuri

R: P4.9.1. 32kg fin
R: P4.9.2. 9 ore
R: P4.9.3. 3 bancnote
R: P4.9.4. 90 piese
R: P4.9.5. 50 zile
R: P4.9.6. 30 vagoane
R: P4.9.7. 100m
R: P4.9.8. 7.600.000lei
R: P4.9.9. 31,25m
R: P4.9.10. 6000kg

Regula de trei compus

Vom considera acum probleme n care intervin mai multe mulimi de cte dou
numere, ntre unele din ele existnd o proporionalitate direct, iar ntre altele o
proporionalitate invers.
Regula de trei compus este un procedeu de aflare a unui numr necunoscut,
ntr-o problem n care intervin mai multe mrimi, cu cte dou valori, ntre unele
existnd o proporionalitate direct, iar ntre altele o proporionalitate invers.
Aplicarea acestui procedeu, numit regula de trei compus, pornete de la
aezarea datelor problemei ntr-o schem; apoi, se stabilesc tipurile de proporionalitate
ce exist ntre mrimea necunoscut i fiecare din celelalte mrimi, indicndu-se
nmulirea prin sgei, iar n final se efectueaz nmulirile i mpririle ce conduc la
aflarea numrului necunoscut. La acest procedeu s-a ajuns datorit modului de
rezolvare a acestui tip de probleme cu ajutorul aplicrii succesive a regulii de trei
simpl.
Model. Cinci muncitori pot termina o lucrare n 15zile, dac lucreaz cte 8ore
pe zi. n ct timp vor termina aceeai lucrare 10 muncitori, lucrnd cte 6ore pe zi?
Soluie. Datele problemei se aeaz dup urmtoarea schem:
5 muncitori..............................8h/zi..............................15zile
10muncitori..............................6h/zi.............................. x

Pentru a avea doar dou mrimi i a putea aplica, astfel, regula de trei simpl,
considerm constant numrul muncitorilor i problema se transform n:
i.p.
5 muncitori..............................8h/zi..............................15zile

5 muncitori..............................6h/zi.............................. y

20
6
8 15
=

= y (zile)

70
Deci, 5 muncitori, lucrnd cte 6 ore pe zi termin lucrarea n 20 de zile. Acum
numrul de 6h/zi, fiind constant, trebuie s aflm n ct timp vor termina lucrarea cei
10 muncitori:
i.p.

5 muncitori..............................6h/zi..............................20zile

10muncitori..............................6h/zi.............................. x

10
10
5 20
=

= x (zile)

Practic, regula de trei compus cuprinde urmtoarele etape:
1) Aezarea datelor problemei n schem:

5 muncitori..............................8h/zi..............................15zile
10muncitori..............................6h/zi.............................. x

2) Stabilirea tipului de proporionalitate ce exist ntre mulimea ce conine
necunoscuta i, succesiv, celelalte mulimi:
ntre mulimea zilelor i cea a muncitorilor exist o proporionalitate invers
ntre mulimea zilelor i cea a orelor de lucru zilnic exist o proporionalitate
invers.
Precizm aceste proporionaliti prin sgei, pe schem:

i.p.

i.p.
5 muncitori..............................8h/zi..............................15zile

10muncitori..............................6h/zi.............................. x

10
6 10
8 5 15
=


= x (zile)

Remarc. Aceeai problem se poate rezolva prin metoda reducerii la unitate
fcnd urmtorul raionament: dac 5 muncitori, lucrnd cte 8h/zi, termin lucrarea n
15zile, atunci 1muncitor, lucrnd cte 8h/zi, termin lucrarea n 515zile. Rezult c
1muncitor, lucrnd cte o or pe zi, termin lucrarea n 5158zile. Rezult, n
continuare, c un muncitor, lucrnd cte 6h/zi, va termina lucrarea n
6
8 15 5
zile.
71
Deci, 10muncitori, lucrnd cte 6h/zi, va termina lucrarea n
10 6
8 15 5


zile, adic n
10zile. Acest raionament se aeaz sub forma urmtoarei scheme:

5muncitori..........................8h/zi..........................15zile
10muncitori........................6h/zi.......................... x

1muncitor...........................8h/zi..........................155
1muncitor...........................1h/zi..........................1558
1muncitor...........................6h/zi..........................
6
8 5 15

10muncitori.........................6h/zi......................... 10
10 6
8 5 15
=


zile

Probleme rezolvate

R4.9.3. ntr-o tabr, n 12zile, 150 de elevi consum 900kg pine. Ce
cantitate de pine este necesar pentru 70 de elevi, pentru 18 zile?
Soluie. 1) Aezarea datelor problemei n schem

150elevi..............................12zile..............................900kg
70elevi................................18zile.............................. x

2) Stabilirea tipului de proporionalitate ce exist ntre mulimea ce conine
necunoscuta i, succesiv, celelalte mulimi, preciznd aceste proporionaliti prin
sgei, pe schem:

150elevi..............................12zile..............................900kg
d.p. d.p.
70elevi................................18zile.............................. x

630
12 150
18 70 900
=


= x kg

R4.9.4. 6 muncitori pot termina o lucrare n 12zile. Dup 4zile de lucru
echipei de muncitori i se altur nc 2 muncitori. n ct timp se va executa toat
lucrarea?
Soluie. Dac echipa poate termina lucrarea n 12zile, atunci dup 4zile de
lucru echipa a efectuat
12
4
, adic
3
1
din lucrare. Deci, trebuie s aflm n cte zile 8
muncitori fac
3
2
din lucrare.
72
Aezm datele problemei i stabilim tipul de proporionalitate ce exist ntre
mulimea ce conine necunoscuta i, succesiv, celelalte mulimi, preciznd aceste
proporionaliti prin sgei, pe schem:

i.p.

6muncitori..............................1lucrare..............................12zile
d.p.
8muncitori..............................
3
2
lucrare............................. x

6
1 8
3
2
6 12
=


= x zile

Lucrarea s-a efectuat n 4zile+6zile, deci n 10zile.
R4.9.5. O echip de 20 muncitori, lucrnd cte 6ore pe zi, pot face 24piese n
10zile. Cte zile sunt necesare pentru ca o alt echip de 15 muncitori s fac 360piese,
lucrnd cte 8ore pe zi?
Soluie. Aezm datele problemei n schem i stabilim tipul de propor-
ionalitate ce exist ntre mulimea ce conine necunoscuta i, succesiv, celelalte
mulimi, preciznd aceste proporionaliti prin sgei, pe schem:

i.p.

i.p.
20muncitori....................6h/zi....................240piese....................10zile
d.p.
15muncitori....................8h/zi....................360piese.................... x

15
240 8 15
360 6 20 10
=


= x zile



73
5. Numere ntregi

n cadrul temei se vor studia i aplica noiunile de numr ntreg, opusul unui
numr, modulul unui numr ntreg, divizibilitatea n mulimea numerelor ntregi,
determinarea valorii unei expresii ce depind de un exponent natural, rezolvarea de
ecuaii i inecuaii.
5.1. Divizibilitatea n
Reamintim noiunile necesare abordrii temei.
Mulimea numerelor ntregi se noteaz cu .
= {,., 3; 2; 1; 0; +1; +2; +3; .}
* = {0}
Definiia 5.1.1. Opusul unui numr ntreg a este numrul ntreg a cu
proprietatea c punctele de pe ax corespunztoare numerelor a respectiv a se afl la
egal distan fa de origine.
Exemplu: opusul numrului 7 este numrul 7
opusul numrului 6 este 6.
Scriem: (2) = 2.
Definiia 5.1.2. Distana pe ax ntre punctul corespunztor numrului ntreg a
i origine se numete modulul numrului a sau valoarea absolut i se noteaz cu a.
Exemple:5 = 5; 1 = 1; 0 = 0; 3 = 3; 1 = 1; 7 = 7.
Definiia 5.1.3.
a dac a > 0
() a ; a = 0 dac a = 0
a dac a < 0
Proprieti:
1. Modulul oricrui numr ntreg pozitiv este mai mare dect zero.
() a *; a 0
2. Modulul unui numr ntreg este egal cu zero dac i numai dac numrul este
egal cu zero.
a = 0 a = 0
3. Dou numere opuse au modulele egale.
() a ; a = a
4. Dac o sum de module este egal cu 0 atunci fiecare modul este egal cu zero.
5. Dac a
1
+ a
2
+ .+ a
n
= 0 atunci a
1
= 0 i a
2
= 0 i
a
n
= 0
Definiia 5.1.4. Numrul ntreg a divide numrul ntreg b dac exist numrul
ntreg c astfel nct b = ac.
Notm: ab (numrul a divide numrul b).
ba ( numrul b se divide cu numrul a).
Numrul a se numete divizorul lui b, iar b este multiplul lui a dar i a lui c.
Exemplu: 728 pentru c exist numrul 4 astfel nct 28 = (7)4

74
Proprieti ale relaiei de divizibilitate:
1. 1a, oricare ar fi a ;
2. aa, oricare ar fi a ;
3. a0, oricare ar fi a *;
4. Dac ab i bc atunci ac, () a, b * i c ;
5. Dac ab i ba atunci a = b, () a, b *;
6. Dac ab sau ac atunci abc,() a *i b, c ;
7. Dac ab i ac atunci a(bc),() a *i b, c ;
8. Dac ac i bc i (a; b) = 1 atunci (ab)c,() a, b *i b, c ;
9. Dac a1 sau a1 atunci a = 1.
10. ab(a)ba(b)(a)(b).

Definiia 5.1.5. Un numr ntreg d este divizor comun al numerelor a i b dac
da i db.
Definiia 5.1.6. Numrul ntreg d este cel mai mare divizor comun al
numerelor a i b dac:
1) da i db;
2) orice divizor d a lui a i b, comun, divide pe d.
Definiia 5.1.7. Dou numere ntregi nenule a i b se numesc prime ntre ele
dac (a; b) = 1.
Definiia 5.1.8. Numrul ntreg m este cel mai mic multiplu comun al
numrului a i b dac:
1) a i b au multiplu comun pe m;
2) orice alt multiplu comun a lui a i b este i multiplul lui m.

Probleme rezolvate
R5.1.1. Determinai mulimea divizorilor numrului:
a) 12; b) 20; c) 7
Soluie:
D
12
= {1; 2; 3; 4; 6;12}.
D
20
= {1; 2; 4; 5; 10;20}.
D
7
= {1; 7}. Numrul 7 se numete prim pentru c are ca divizor numai pe 1 i 7.
R5.1.2. Determinai n astfel nct (2n + 1)(3n + 4).
Soluie:
2n + 13n +4 i 2n + 12n + 1 atunci
2n + 12(3n + 4)
i 2n + 16n + 8 (6n + 3)
2n + 13(2n + 1)
2n + 16n + 8 6n 3
2n + 152n + 1 D
5
= { 1; 5}


75
2n + 1= 11
2n = 0: 2
n = 0

2n + 1= 11
2n = 2: 2
n = 1

2n + 1= 5
2n = 5 1
n = 4:2
n = 2

2n + 1= 5
2n = 6: 2
n = 3
n {0, 1, 2, 3}
Soluia a II-a:
Determinai numerele ntregi n pentru care:
1 n 2
4 n 3
+
+

1 n 2
4 n 3
+
+
dac
1 n 2
) 4 n 3 ( 2
+
+

=
+
+
+
+
=
+
+ +
=
+
+
1 n 2
5
1 n 2
3 n 6
1 n 2
5 3 n 6
1 n 2
8 n 6

= 3 +
1 n 2
5
+
dac
1 n 2
5
+

2n + 15 2n+ 1 D
5
= {1; 5}
n {0; 1; 2; 3}.
R5.1.3. Determinai numerele ntregi x i y astfel nct: (x + y)(2y 1) = 43
Soluie:
Din 43 = (x + 1)(2y 1) rezult c
x + 143 i 2y 143 i x + 1 i 2y 1 au acelai semn, atunci avem:
1) x + 1 = 1
2y 1 = 43
2) x + 1 = 43
2y 1 = 1
3) x + 1 = 1
2y 1 = 43
4) x + 1 = 43
2y 1 = 1

Soluiile sunt:
1) x = 0
y = 22

2) x = 42
y = 1

3) x = 2
y = 21

4) x = 44
y = 0



76
R5.1.4. Determinai n pentru care fracia
1 n
5 n 2
+
+
este reductibil.
Soluie:
Fie d cel mai mare divizor comun al numerelor 2n + 5 i n + 1 atunci avem:
d2n + 5 i dn + 1
d2n + 5 i d2(n + 1)
d2n + 5 2(n + 1)
d2n + 5 2n 2
d3, 3 este numr prim, rezult c d = 3.
Atunci 32n + 5 i 3n + 1
32n + 5 (n + 1)
32n + 5 n 1
3n + 4
n + 4 = 3k, k
n = 3k 4

5.2. Determinarea valorii unei expresii ce depinde de un exponent natural

Definiia 5.2.1. Dac a este numr ntreg i n numr natural, n 2 atunci
puterea n a lui a este: a
n
= aaaa

n factori
a se numete baza puterii;
n exponentul puterii.
Dac a > 0 atunci a
n
> 0
Dac a < 0 atunci a
n
> 0 dac n = 2k, k
a
n
< 0 dac n = 2k + 1, k

Oricare ar fi a i n = 1 avem a
1
= a.
Oricare ar fi a * i n = 0 avem a
0
= 1.
0
0
nu are sens.

Reguli de calcul cu puteri
1. a
m
a
n
= a
m+n
, a , m, n
2. a
m
:a
n
= a
mn
, a , m, n , m n
3. (a
m
)
n
= a
mn
, a , m, n ;
4. (ab)
n
= a
n
b
n
, a , b i n ;
5. (a:b)
n
= a
n
:b
n
, a, b , b 0, n ;


77
Probleme rezolvate
R5.2.1. Fie E(n) = (1)
n
n, n *. S se calculeze:
E(1) + E(2) + E(3) + + E(2003)
Soluie:
E(1) = (1)
1
1 = 1
E(2) = (1)
2
2 = 12 =2
E(3) = (1)
3
3 = 13 = 3
E(4) = (1)
4
4 = 14 = 4

E(2003) = (1)
2003
2003 = 12003 = 2003.

E(1) + E(2) + E(3) + E(4) ++E(2001) + E(2002) + E(2003) =
= (1 + 2) + (3 + 4) + + (2001 + 2002) 2003 =
= 1 + 1 + 1+ + 1 2003 =

1001 termeni
= 1001 2003 = 1002
R5.2.2. Calculai:
[2
333
3
222
+27
75
81
15
+729
17
512
37
3
225
+ 9
30
]: (23
222
4
167
)
Soluie:
2
333
= (2
3
)
111
= 8
111

3
222
= (3
2
)
111
= 9
111

2
333
< 3
222
2
333
3
222
< 0
2
333
3
222
= 2
333
+ 3
222

27
75
= (3
3
)
75
= 3
225

81
15
= (3
4
)
15
= 3
60

27
75
81
15
> 0 27
75
81
15
=27
75
81
15

729
37
= 3
637
= 3
222
> 2
333

512
37
= 2
937
= 2
333

3
222
2
333
> 0 3
222
2
333
= 3
222
2
333

Avem:
(3
222
2
333
+ 3
225
3
60
+ 3
222
2
333
3
225
+ 3
60
): (23
222
4
167
) =
= (23
222
22
333
):(23
222
2
334
) = (23
222
2
334
):(23
222
2
334
) = 1
R5.2.3. Fie numrul:
A = 243(1)
n
342(1)
n + 1
+ 456 ( )
1990 k k
2
1
+
654 ( )
1 p p
2
1
+ +

unde k, p, n . Artai c A 5.
Soluie:
k
2
k + 1990 = k(k 1) + 1990
k(k 1)2 ( produs de nr.nat.consec.)
19902
(k
2
k + 1990) 2 ( ) 1 1
1990 k k
2
=
+

p
2
+ p + 1 = p(p + 1) + 1
p(p + 1)2 (produs de nr.nat.consec.) p (p + 1) + 1 impar ( ) 1 1
1 p p
2
=
+ +

Atunci: A = (1)
n
243 342(1)
n+1
+ 456 + 654 = 243(1)
n
342(1)
n+1
+ 1110 =
= (1)
n
[243 342(1)] + 1110 = (1)
n
585 + 1110
1) n nr. pare A = 585 + 1110 = 1695
2) n nr. impare A = 585 + 1110 = 525
+
8
111
< 9
111

3
225
> 3
60
27
75
> 81
15


78
5.3. Ecuaii n mulimea numerelor ntregi. Inecuaii n mulimea numerelor ntregi
n cadrul temei se vor studia ecuaii de forma ax + b = 0, x (a *; b
) ax + b = c, x ( a *; b, c ), i ecuaii reductibile la acestea, ecuaii de
forma ax
2
+b = 0, x (a *; b ), ecuaii cu modul, (ax + b)(cx + d) = m, x
(a, c *; b,d , m ).
Definiia 5.3.1. O ecuaie este o propoziie cu o variabil scris sub forma
unei egaliti, semnul egal apare o singur dat n scrierea ei.
Exemplu: 1) 3x 6 = 0 x ;
2) 2x 7 = 4 x
3) 2(x + 1) = 3x x {1, 2, 3}
4) x
2
4 = 0 x {1; 2; 2; 5}
Variabila ntr-o ecuaie se numete necunoscut.
Exist ecuaii cu o necunoscut, cu dou necunoscute i aa mai departe.
Necunoscutele se noteaz cu: x, y, z
Mulimea valorilor necunoscutelor pentru care propoziia este adevrat este
mulimea soluiilor ecuaiei.
Exemplu: 2(x + 1) = 3x
pt. x = 1 2(1 + 1) = 31, pt x = 2 2(2 + 1) = 32
22 = 31 (F) 23 = 32 (A)
pt. x = 3 2(3 + 1) = 33;
24 = 33 (F)
Mulimea soluiilor este {2}. i se noteaz S = {2}.
A rezolva o ecuaie nseamn a determina mulimea soluiilor.
Definiia 5.3.2. Dou ecuaii cu o necunoscut sunt echivalente dac au
aceeai mulime de soluii.
O ecuaie de forma ax + b = 0, x , unde a i b sunt numere ntregi, iar a 0, va fi
numit ecuaie liniar cu o necunoscut (sau ecuaie de gradul I cu o necunoscut).

Proprieti de echivalen:
1. Dac adunm sau scdem acelai numr ntreg n fiecare membru al unei
ecuaii, obinem o ecuaie, echivalent cu prima.
2. Dac nmulim sau mprim ambii membri ai unei ecuaii cu acelai
numr ntreg, diferit de zero, obinem o ecuaie echivalent cu prima.

Probleme rezolvate
R5.3.1. Rezolvarea ecuaiei ax + b = 0, x , (a *, b )
ax + b = 0b
ax + b b = b
ax = b:a
x = b : a
Dac ab atunci x = b : a este soluia.
Dac ab atunci ecuaia nu are soluii n .
R5.3.2. Rezolvarea ecuaiei:
ax + b = c, x (a *, b,c ).
ax + b = cb
ax + b b = c b
ax = c b:a
Dac a(c b) atunci ecuaia are soluia: x = (c b) : a
Dac a (c b) atunci ecuaia nu are soluie n .
Exemple:

79
1) 2x 4 = 0+4
2x 4 + 4 = 0 + 4
2x = 4:2
x = 4 : 2
x = 2
S = {2}
2) 3x + 7 = 07
3x + 7 7 = 0 7
3x = - 7:3
x= 7 : 3
S =

3) 5x + 21 = 621
5x + 21 21 = 6 21
5x = 15:5
x = 3
S = {3}
4) 7x + 11 = 411
7x + 11 11 = 4 11
7x = 15:7
x = 13 : 7
S =
R5.3.3. Ecuaii care se rezolv prin metode speciale:
1. Dac x, y , rezolvai ecuaiile:
a) xy = 5
Trebuie s determinm dou numere ntregi a cror produs este egal cu 5
Soluiile sunt:
1) x = 1
y = 5
2) x = 5
y = 1
3) x = 1
y = 5
4) x = 5
y =1

S = {(1; 5); (5; 1); (1; 5); (5; 1)}

b) 2xy 3x = 7
Scoatem factor comun i obinem x(2y 3) = 7, avem urmtoarele situaii:
1) x = 1 x = 1 x = 1 x =1
2y 3 = 7 2y = 7 + 3 2y = 10 y = 5

2) x = 7 x = 7 x = 7 x =7
2y 3 = 1 2y = 1 + 3 2y = 4 y y = 2

3) x = 1 x = 1 x = 1 x =1
2y 3 = 7 2y = 7 + 3 2y = 4 y = 2

4) x = 7 x = 7 x = 7 x =7
2y 3 = 1 2y = 1 + 3 2y = 2 y y = 1

S = {(1; 5); (7; 2); (1; 2); (7; 1)}
c) Rezolvai n x ecuaiile
2xy 3x 4y + 9 = 0
2xy 3x 4y + 6 + 3 = 0
x(2y 3) 2(2y 3) = 3
(2y 3)(x 2) = 3
Avem urmtoarele situaii:
1) 2y 3 = 1 = 2y= 1 + 3 2y = 2 2 y =1
x 2 = 3 x = 3 + 2 x = 5 x = 5







80
2) 2y 3 = 3 2y= 3 + 3 2y = 6 2 y =3
x 2 = 1 x 2 = 1 x = 1 + 2 x = 1

3) 2y 3 = 1 2y= 1 + 3 2y = 4 2 y =2
x 2 = 3 x = 3 + 2 x = 1 y x = 1

4) 2y 3 = 3 = 2y= 3 +3 2y = 0 2 y =0
x 2 = 1 x = 1 + 2 x = 3 y x = 3

S = {(5; 1); (1; 3); (1; 2); (3; 0)}
2. Ecuaii cu modul:
Rezolvai n ecuaiile:
a) x= 5 x = 5
Aplicm proprietatea modulului:
() x , avem x= x
b) x 3= 3 (x 3) = 3
1) x 3 = 3 x = 3 + 3 x = 6
2) x 3 = 3 x = 3 3 x = 0
S = {0, 6}
c) () x , avem x 0
Dac x + 5= 2 S = pentru c x + 5 0 () x
d) x 2 + y + 1= 0
() a
1
, a
2
,, a
n
, dac a
1
+a
2
++ a
n
= 0 a
1
= a
2
= = a
n
= 0
Din x 2+y + 1= 0
x 2= 0
i
y + 1= 0
x 2 = 0
i
y + 1 = 0
x = 2
i
y=1

S = {(2; 1)}




81
e) x + 1+y 2= 2
Dac x, y i x + 1+y 2= 2, atunci avem:

1) x + 1= 0 x + 1 = 0
]
y 2= 2 y 2 = 2
a) x + 1= 0 x = 1
]
y 2 = 2 y = 4
b) x + 1= 0 x = 1
]
y 2 = 2 y = 0

2) x + 1= 2 x + 1 = 2
]
y 2= 0 y 2 = 0
a) x + 1= 2 x = 1
]
y 2 = 0 y = 2
b) x + 1= 2 x = 3
]
y 2 = 0 y = 2

3) x + 1= 1 x + 1 = 1
]
y 2= 1 y 2 = 1
a) x + 1= 1 x = 0
]
y 2 = 1 y = 3


b) x + 1= 1 x = 0
]
y 2 = 1 y = 1

c) x + 1= 1 x = 2
]
y 2 = 1 y = 3
d) x + 1= 1 x = 2
]
y 2 = 1 y = 1

82
Probleme propuse
Rezolvai cu ecuaiile:
P5.3.1. 5(x 1) = 2x + 7
P5.3.2. 2(4x + 1) = 2(5x 7)
P5.3.3. 2(x + 5) = 8(3x + 2) + 56
P5.3.4. 5(x 2) 7(x + 3) = 35
P5.3.5. 3(x + 1) 1 = x + 6
P5.3.6. (x 1)(x + 2) = 0
P5.3.7. (2x + 8)(3x + 9) = 0
P5.3.8. xy 2x = 7
P5.3.9. xy + x 2y 4 = 0
P5.3.10. xy x + 3y 3 = 0
P5.3.11.x 2= 3
P5.3.12. 3x 1= 14
P5.3.13.2x 6 = 8
P5.3.14. 2 10 + 3x= 5
P5.3.15. x= 3
P5.3.16. 10 x + 2= 8
P5.3.17. 2x 1+ 31 x= 10
P5.3.18. 3x 222 x= 11
P5.3.19. x 3+x + 5= 0
P.5.3.20.x+y 2= 3
P5.3.21. x 1+y + 3= 1
P5.3.22. x + 21= 3
P5.3.23.x + 53y 12= 0
P5.3.24. Se dau mulimile:
A= {x / 2x 1(8)}, B = {y /12(3y 1)}
a) Determinai elementele celor dou mulimi;
b) Aflai AB i AB.

P5.3.25.
a) x y+ (2x 6)
2
= 0

b) x 3+ (2x y)
2
= 0
P5.3.26. 3x 2y = 15 i x= 5

Inecuaii n mulimea numerelor ntregi

Definiia 5.3.3. O propoziie cu o variabil scris sub forma unei inegaliti se
numete inecuaie.
Exemple: ax + b > 0, ax + b < 0; ax + b 0; ax + b 0 n care a, b , sunt
fixate, a 0, iar x este variabila ntreag, x se numete necunoscuta inecuaiei.
Pentru a rezolva o inecuaie folosim proprietile inegalitii numerelor ntregi:

83
oricare ar fi a, b, c i a > b atunci a + c > b + c
oricare ar fi a, b , c
+
*, i a > b, atunci a:c > b:c, dac c divide pe a i c
divide pe b.
oricare ar fi a, b i c
-
* i a > b, atunci a:c < b.c, dac c divide pe a i c
divide pe b.

Probleme rezolvate

R5.3.4. Rezolvai inecuaia: 5x + 3 3x 5, x .
Soluie:
5x + 3 3x 53x
5x + 3 3x 3x 5 3x
5x 3x + 3 53
2x + 3 3 5 3
2x 8:2
x 4
x {, 5, 4} S = {, 5, 4}
R5.3.5. Rezolvai inecuaia:
x 3 3, x
1) x 3= 3x 3 = 3 a) x = 6
b) x = 0
2) x 3= 2x 3 = 2 a) x = 5
b) x = 1
3) x 3= 1x 3 = 1 a) x = 4
b) x = 2
4) x 3= 0 x = 3
S = {0, 1, 2, 3, 4, 5, 6}
R5.3.6. Determinai numerele ntregi care verific inegalitatea:x(y 2) > 0
Soluie:
x(y 2) > 0

x > 0, dac x 0
1) Dac x * atunci y {3, 4, 5,}
2) Dac x = 0 inecuaia nu are soluie.



y 2 > 0
y > 2

84
Bibligrafie
C. Hrbor, D.Svulescu, I. Chec, A.ifrea: Matematic pentru clasele V-VIII-
Olimpiadele judeene,interjudeene,naionale, Ed. Teora 1996, pag 72-76
D. Andrica, E. Jecan, D. Vlcan, I. Bogdan, Probleme calitative n matematica de
gimnaziu, Ed. Gil Zalu 1998, pag 21-43
Gheorghe i Alina Drugan; Ion i Mihaela Ghica, Matematica n concursurile colare,
Ed. Teora 1998, pag 5-23
D. Brnzei, D. Zaharia, M. Zaharia : Aritmetic-Algebr-Geometrie, Ed.Paralela 45
2002, pag 5-43
D. Andrica, V. Berinde, Al. Blaga, G.Both, O. Pop, Concursul Grigore Moisil Ed. I-
XV, Ed. Hub Press 22 Baia-Mare 2001, pag 9,12,40
C. Moroti, M. Giurgiu, D. Radu, R. tefan, A. Ciupitu, G. Drugan, I. Ghica,
Matematic-exerciii i probleme pentru clasa a VI-a, Ed. Meteor Press 2002, pag 44-49, semII
D. Mihe, D. Angelescu, I. Chera, C. Popescu i colectivul, Olimpiadele de matematic
1990-1998, clasa a VI-a, Ed. Gil Zalu 1999, pag 37-43
C. Hrbor, D.Svulescu, I. Chec, A.ifrea: Matematic pentru clasele V-VIII-Olimpiadele
judeene,interjudeene,naionale, Ed. Teora 1996, pag 72-76
D. Brnzei i colectivul: Matematica n concursurile colare, Ed. Paralela 45,
2000,2001,2002 pag 27-54,119-135(2000);pag 27-54,117-130(2001);pag 18-34,82-92 (2002)
D. Brnzei, D. Zaharia, M. Zaharia : Aritmetic-Algebr-Geometrie, Ed.Paralela 45
2002, pag 5-43
C. Popovici, I. Ligor, V. Alexianu, Matematic-Aritmetic-Algebr, EDP Bucureti
1996
G. Turcitu, I. Rizea, C. Basarab, M. Duncea, Manual clasa a VI-a, Ed. Radical 1998
T. Udrea, D. Nuescu, Manual clasa a VI-a, EDP 1998
D. Brnzei, D. Zaharia, M. Zaharia : Aritmetic-Algebr-Geometrie PII, Ed. Paralela
45 2002, pag 39-43

85
GEOMETRIE

1. Segmente

n cadrul temei se vor studia noiunile de punct, dreapt, plan, semidreapt,
segment de dreapt, dar i aplicaii n care se vor determina poziiile unor puncte pe o
dreapt, lungimea unui segment, distana dintre mijloacele a dou segmente i
congruena a dou segmente.

1.1 Puncte, drepte, semidrepte, segmente de dreapt
n vederea abordrii temei reamintim noiunile absolut necesare.
Considerm figura:


O B A X
Vom spune c punctul B este ntre O i A.
Definiia 1.1.1 Fiind date punctele O i A pe o dreapt, mulimea format din
punctele dreptei OA situate ntre O i A mpreun cu punctele X de pe dreapt pentru
care A este ntre O i X se numete semidreapt. Punctul O este originea semidreptei
Semidreapt deschis
A
(
Notaie: (OA- semidreapt deschis
Semidreapta nchis:
A

Notaie: [OA- semidreapt nchis ; [OA = (OA { } A
O definiie mai puin riguroas dar uor de reinut de ctre elevi este:
semidreapta este o parte dintr-o dreapt, limitat la unul dintre capete, numit originea
semidreptei.

Definiia 1.1.2 Fiind date dou puncte A i B, mulimea punctelor ce aparin
dreptei AB situate ntre A i B se numete segment deschis i se noteaz (AB)
( )
A B
Segmentul nchis [AB] = (AB) { } B A,
[ ]
A B
Punctele A i B se numesc extremitile sau capetele segmentului.

Definiia 1.1.3 Distana dintre punctele a i B exprimat ntr-o unitate de
msur se numete lungimea segmentului [AB].
Notaie: AB = 5 cm.


86
Definiia 1.4. Dou segmente se numesc congruente dac au msuri egale.
Notm [AB] [CD].
A 3,5 cm B
C 3,5 cm D

Definiia 1.1.5 Mijlocul unui segment este un punct care mparte segmentul n
dou segmente congruente.
M
A B
Notm M este mijlocul segmentului [AB] sau M (AB) i [AM] [MB]
Definiia 1.1.6 Fiind date dou segmente, vom num segmentul sum al lor un
segment care are msura egal cu suma msurilor celor dou segmente, iar segmentul
diferen segmentul care are msura egal cu diferena msurilor segmentelor date.
Dac AB = 15 cm, CD =7 cm i MN= 22 cm, atunci MN=AB+CD
(22=15+7), [MN] este segmentul sum.
Dac AB = 15 cm, CD =7 cm i PQ=8 cm, atunci PQ=AB CD (8=15 7),
[PQ] este segmentul diferen

Probleme rezolvate

R1.1.1 Fiind date 10 puncte distincte dou cte dou i necoliniare trei cte trei,
aflai numrul de drepte determinate de cte dou dintre ele.
Soluie
Dou puncte distincte determin o dreapt i numai una. Fie A
1
,A
2
,A
3
.......,A
10

cele 10 puncte punctul A
1
determin cu punctele A
2
,A
3
.......,A
10,
9 drepte.
Mergnd cu raionamentul din fiecare punct putem duce (10-1) drepte, ns n
acest caz fiecare dreapt o considerm de dou ori, deci numrul dreptelor este dat de
( )
45
2
9 10
2
1 10 10
=


Dac numrul punctelor este n atunci numrul dreptelor este
( )
2
1 n- n
.
R1.1.2 Considerm punctele A,B,C,D pe dreapta d, astfel nct AB=a cm, AC=b
cm, BD=c cm, BC=(a+b)cm, CD=(a+b-c)cm i AD=(c-a)cm. Determinai ordinea
punctelor pe dreapt.
Soluie
Din AB=a, AC=b i BC=a+b rezult c A este ntre B i C.
Dac B aparine lui (AD) atunci CD=a+b+c ceea ce este fals.


C D A B

B A D C
Dac C este ntre B i D atunci CD=c-a-b ceea ce este fals, deci D este ntre B
i C i cum AD=c-a rezult c D este ntre Ai C deci ordinea punctelor este:

87
1) C,D,A,B sau 2) B,A,D,C
R1.1.3 Punctul M
1
este mijlocul segmentului [AB], punctul M
2
este mijlocul
segmentului [AM
1
].
Repetnd procedeul punctul M
10
este, mijlocul segmentului AM
9
. Dac
AB=2
11
3cm, calculai msura segmentului AM
10
.

Soluie ;
2
3 2
AM ;
2
AB
AM
11
1 1

= =






Bibliografie
I. Petric i colectivul, Manual pentru clasa a VI-a, Ed. Petrion 1998
G. Turcitu, I. Rizea, C. Basarab, M. Duncea, Manual clasa a VI-a, Ed. Radical 1998
T. Udrea, D. Nuescu, Manual clasa a VI-a, EDP 1998
D. Brnzei, D. Zaharia, M. Zaharia : Aritmetic-Algebr-Geometrie, Ed.Paralela 45
2002, pag 97-109
C. Hrbor, D.Svulescu, I. Chec, A.ifrea: Matematic pentru clasele V-VIII-
Olimpiadele judeene,interjudeene,naionale, Ed. Teora 1996, pag 107
D. Brnzei, D. i M. Goleteanu, S. Ulmeanu, V. Gorgot, I. erdean: Matematica n
concursurile colare, Ed. Paralela 45, 2000,2001,2002
D. Constantinescu: Olimpiadele colare, Ed. Teora 1997-2002
D. Mihe, N. Angelescu, I. Chera, C. Popescu i colectivul:Olimpiadele de matematic
1990-1998, Ed. Gil Zalu ,clasa a VI-a, pag 56-61
Edwin E. Moise, Floyd I. Downs jr. : Geometria , EDP 1983, pag 38-59


6; 3 2
2
3 2
AM ;
2
AB
AM
10
11
10 10
= =

= =
;
2
3 2
AM ;
2
AM
AM
3
11
3
2
3

= =
;
2
3 2
AM ;
2
AM
AM
2
11
1
1
1

= =

88
A
2. Unghiul

n cadrul temei se vor utiliza noiunile de unghi, unghiuri adiacente, bisectoarea
unui unghi, msura unui unghi, unghiuri congruente, unghi ascuit, drept, obtuz,
unghiuri opuse la vrf, unghiuri n jurul unui punct, drepte perpendiculare, mediatoarea
unui segment i aplicaii n care se vor determina msuri de unghiuri, congruena unor
unghiuri, perpendicularitatea unor drepte i coliniaritatea unor puncte.

2.1 Noiuni teoretice necesare abordrii temei

Definiia 2.1.1
Se numete unghi figura geometric format din dou semidrepte nchise care au
aceeai origine.








[OA i [OB laturile unghiului
O vrful unghiului
Notaie: AOB
Unghi alungit





AOB este alungit dac laturile lui sunt semidrepte opuse
Unghi nul






AOB este unghi nul dac [OA=[OB (semidreptele coincid)


Unitatea de msur pentru unghiuri este gradul. Unghiul de un grad reprezint a
180-a parte dintr-un unghi alungit.
B
O
A
O
B
O
A
B

89
Definiia 2.1.2
Msura unui unghi este un numr care ne arat de cte ori se cuprinde unitatea
de msur n unghiul pe care l msurm.
Notaie: Dac unghiul AOB are msura de 30 de grade notm m(AOB)=30
0

Observaie:
1
0
Msura unui unghi alungit este 180
0

2
0
Msura unui unghi nul este 0
0

Definiia 2.1.3
Dou unghiuri se numesc congruente dac au msurile egale
Notaie: AOB AOB (unghiul AOB este congruent cu unghiul AOB)

Definiia 2.1.4.
Se numete bisectoare interioar a unui unghi nenul o semidreapt interioar
unghiului, cu originea n vrful unghiului, care formeaz cu laturile acestuia dou
unghiuri congruente.
Definiia 2.1.5
Unghiul care are msura de 90
0
se numete unghi drept.
Definiia 2.1.6
Unghiul cu msura cuprins ntre 0
0
i 90
0
se numete unghi ascuit.
Definiia 2.1.7
Unghiul cu msura cuprins ntre 90
0
i 180
0
se numete unghi obtuz.
Definiia 2.1.8
Dou unghiuri se numesc adiacente dac au vrf comun, o latur comun i
interioarele disjuncte.
Definiia 2.1.9
Dou unghiuri se numesc complementare dac suma msurilor lor este 90
0
.
Definiia 2.1.10
Dou unghiuri se numesc suplementare dac suma msurilor lor este 180
0
.
Definiia 2.1.11
Dou unghiuri se numesc opuse la vrf dac au acelai vrf i laturile unuia se
gsesc n prelungirea laturilor celuilalt.
Propoziia 2.1.1
Dou unghiuri opuse la vrf sunt congruente.
Propoziia 2.1.2
Suma msurilor unghiurilor formate n jurul unui punct este 360
0
.
Definiia 2.1.12
Dou drepte se numesc perpendiculare dac formeaz un unghi drept.
Notaie: ab (dreptele a i b sunt perpendiculare)
Definiia 2.1.13
Prin distana de la un punct la o dreapt nelegem distana de la acel punct la
piciorul perpendicularei pe acea dreapt.




90











ab, Aa
ab={A}
A este piciorul perpendicularei din A pe b
AA este distana de la punctul A la dreapta b i notm AA =d(A;b)
Definiia 2.1.14
Mediatoarea unui segment este dreapta perpendicular pe segment n mijlocul
acestuia.

Probleme rezolvate

R2.1.1 Se consider punctele A,O,B coliniare n aceast ordine. n acelai
semiplan determinat de dreapta AB se duc semidreptele [OC i [OD astfel nct [OC
este n interiorul unghiului AOD, iar m(COD)=90
0
. Dac [OE i [OF sunt
bisectoarele unghiurilor AOC respectiv BOD s se afle msura unghiului EOF.
Soluie
AOB alungit m(AOB)=180
0

m(AOC)+m(BOD) =180
0
m(COD)=180
0
90
0
=90
0

[OE bisectoarea AOCm(AOE)=m(EOC)=
2
) AOC ( m

[OF bisectoarea BOD m(BOF)=m(FOD)=
2
) BOD ( m

m(EOF)=m(COD)+m(EOC)+m(DOF)=
= 90
0
+
0
0
0
135
2
90
90
2
) BOD ( m ) AOC ( m
= + =
+

R2.1.2 Se consider unghiul drept AOB i d o dreapt care trece prin O i nu
are puncte n interiorul unghiului sau pe laturile acestuia. Fie punctul C pe dreapta d,
situat n semiplanul determinat de dreapta OB i punctul A, iar D un punct pe dreapta d
astfel ca semidreptele [OC i [OD s fie opuse.
a) Demonstrai c unghiul AOC este ascuit;
a
b
A
A

91
b) Dac punctul M este n interiorul unghiului AOB astfel nct AOC s
fie congruent cu AOM, demonstrai c [OB este bisectoarea unghiului
MOD.
Soluie


















a) COD unghi alungitm(COD)=180
0

AOB unghi drept m(AOB)=90
0

m(AOC)=180
0
[m(AOB)+m(BOD)]=
=180
0
m(AOB) - m(BOD)= 180
0
90
0
m(BOD)=
=90
0
- m(BOD) m(AOC)<90
0
AOC unghi ascuit
b) m(AOC)+m(BOD)=180
0
m(AOB)=180
0
90
0
=90
0

m(BOD)=90
0
m(AOC) (1)
m(AOM)+m(MOB)=90
0
m(BOM)=90
0
m(AOM) (2)
m(AOM)=m(AOC) (3)
Din relaiile (1), (2) i (3) m(MOB)=m(BOD) [OB bisectoarea
MOD
R2.1.3 Se consider unghiul ascuit XOY. n semiplanul determinat de [OX
i n care nu se afl semidreapta [OY , se duc perpendicularele [OA o [OB respectiv
pe [OX i [OY. Se noteaz cu [OC bisectoarea unghiului BOX.
a) Dac msura AOC este cu 16
0
mai mare dect msura XOY,
determinai m( XOY);
b) Artai c dac [OB este bisectoarea AOC , atunci [OX este bisectoarea
COY
B
M
A
D
O
C

92
Soluie









OAOX m(AOX)=90
0
m(AOB)=90
0
m(XOB) (1)
OBOY m(BOY)=90
0
m(XOY)=90
0
m(BOX) (2)
Din relaiile (1) i (2) m(AOB)=m(XOY)
m(AOC)=m(AOB)+m(BOC)=m(XOY)+m(BOC) (3)
m(AOC) m(XOY)=16
0
(4)
nlocuind m(AOC) din relaia (3) n relaia (4) obinem:
m(XOY)+m(BOC) m(XOY)=16
0
m(BOC)=16
0

m(BOX)=2m(BOC)=32
0

m(XOY)=90
0
m(BOX)=90
0
32
0
=58
0




Bibliografie
I. Petric i colectivul, Manual pentru clasa a VI-a, Ed. Petrion 1998
G. Turcitu, I. Rizea, C. Basarab, M. Duncea, Manual clasa a VI-a, Ed. Radical 1998
T. Udrea, D. Nuescu, Manual clasa a VI-a, EDP 1998
D. Brnzei, D. Zaharia, M. Zaharia : Aritmetic-Algebr-Geometrie, Ed.Paralela 45
2002, pag 110-124
C. Hrbor, D.Svulescu, I. Chec, A.ifrea: Matematic pentru clasele V-VIII-
Olimpiadele judeene,interjudeene,naionale, Ed. Teora 1996, pag 108-109, 109-112
D. Brnzei, D. i M. Goleteanu, S. Ulmeanu, V. Gorgot, I. erdean: Matematica n
concursurile colare, Ed. Paralela 45, 2000,2001,2002
D. Constantinescu: Olimpiadele colare, Ed. Teora 1997-2002
D. Mihe, N. Angelescu, I. Chera, C. Popescu i colectivul: Olimpiadele de matematic
1990-1998, Ed. Gil Zalu, clasa a VI-a, pag 61-70,71-77
Edwin E. Moise, Floyd I. Downs jr. : Geometria , EDP 1983, pag 81-105

O
A
B
C
X
Y
93
3. Geometria bazat pe raionament i demonstraii

n problemele de geometrie ne vom baza, n stabilirea unor proprieti pe
judecat (raionament). Punctele principale de plecare ale judecilor pe care le vom
face vor fi cazurile de congruen ale triunghiurilor oarecare. Scopul urmrit va fi acela
de a forma i dezvolta raionamentul geometric i de a deduce cu ajutorul lui
proprietile cele mai importante ale figurilor geometrice.
n matematic (deci i n geometrie) se ntlnesc unele propoziii care exprim
adevruri ce se admit fr demonstraii i care se numesc "axiome" (Cuvntul "axiom"
vine din limba greac: axioma = opinie, tez admis. Termenul a fost folosit, ncepnd
din sec. 6-5 .H. de ctre matematicii din coala lui Pitagora). Spre exemplu: "Prin dou
puncte distincte "trece" o singur dreapt" (axioma dreptei).
Propoziii matematice care exprim adevruri ce trebuie s fie dovedite se
numesc "teoreme" (cuvntul "teorem" vine din limba greac: theorema = examinare,
cercetare. Termenul a fost folosit pentru prima dat de filozoful grec Aristotel n sec. 4
.H.). Spre exemplu: "Dou unghiuri opuse la vrf sunt congruente".
n enunul fiecrei teoreme deosebim dou pri: "ipoteza" (sau premisa), care
este format din toate faptele pe care enunul teoremei le presupune adevrate i
"concluzia", care este format din ceea ce enunul teoremei afirm c se poate deduce
din ipotez. (Cuvntul "ipotez" este compus din dou cuvinte provenite din limba
greac: hypo = sub i thesis = punere. Cuvntul "premis" vine din limba latin:
praemisus = pus nainte, anterior. Cuvntul "concluzie" vine din limba latin: conclusio
= ncheiere).
n exemplul de mai sus, ipoteza este: "dou unghiuri sunt opuse la vrf", iar
concluzia: "aceste dou unghiuri sunt congruente".
n unele cazuri teoremele sunt enunate sub forma unor propoziii ipotetice
(condiionale): ipoteza ncepe cu cuvntul "dac", iar concluzia cu cuvntul "atunci".
Cum teoreme se ntlnesc i n aritmetic, nu numai n geometrie, vom da un exemplu
de teorem prezent sub forma unei propoziii ipotetice (condiionale), ntlnit la
aritmetic n clasa a V-a: "Dac un numr este divizibil cu 3 i cu 5, atunci el este
divizibil cu 15". Se reine cu uurin c "un numr este divizibil cu 3 i cu 5" este
ipoteza, iar "el este divizibil cu 15" este concluzia.
Teoremele trebuie s fie "demonstrate", adic adevrurile din concluzie trebuie
s fie "dovedite" cu ajutorul unor argumente, care sunt adevrurile din ipotez i alte
adevruri (axiome sau teoreme demonstrate anterior).
Pentru primul exemplu:
Ipotez. AOB i A'OB' opuse la vrf
Concluzie. AOBA'OB'.


A
B
O
B'
A'
94
Demonstraie. tim c m(AOB)+m(AOB')=180, iar m(A'OB') +
m(AOB') =180, deci m(AOB)=m(A'OB'), de unde rezult c AOBA'OB'.
Pentru al doilea exemplu:
Ipotez. 3 M n , 5 M n
Concluzie: 15 M n
Demonstraie. tim c: "dac n se divide cu a i b, iar a i b sunt prime ntre ele, atunci
n se divide cu produsul ab".
15 1 ) 5 , 3 ( , 5 , 3 M M M n n n = .
Dac se schimb ntre ele ipoteza i concluzia unei teoreme, se obine o
propoziie nou, care se numete "propoziie reciproc" (Cuvntul "reciproc" vine din
limba latin: reciprocus = care se ntoarce de unde a venit, care inverseaz).
Spre exemplu, "reciproca" primului exemplu ar afirma c: "Dac dou unghiuri
sunt congruente, atunci ele sunt opuse la vrf". Aceast propoziie este fals, deoarece
concluzia ei nu este ntotdeauna adevrat. Pentru a dovedi falsitatea ei este suficient s
dm un singur exemplu din care s rezulte aceasta. Dac vom privi, spre exemplu,
unghiurile CDE i FGH, care sunt congruente (avnd aceeai msur), ne vom da
seama imediat c ele nu sunt opuse la vrf:



Un exemplu care arat c uneori concluzia unei propoziii nu este adevrat, se
numete contraexemplu.
Dac reciproca unei teoreme este o propoziie fals, atunci aceast reciproc nu
este teorem i deci teorema dat nu admite "teorem reciproc".
"Reciproca" celui de al doilea exemplu afirm c: "Dac un numr este
divizibil cu 15, atunci el este divizibil cu 3 i cu 5". Aceast propoziie este adevrat:
Ipotez. 15 M n
Concluzie: 3 M n i 5 M n .
Demonstraie. tim c: "dac n se divide cu m, atunci n se divide cu toi divizorii lui m".
3 5 15 , 3 15 , 15 M M M M n n i 5 M n .
Dac reciproca unei teoreme este o propoziie adevrat, atunci aceast
reciproc devine "teorem reciproc".
95
Se obinuiete ca, fa de teorema reciproc, teorema iniial s se numeasc
"teorem direct". Se mai spune c cele dou teoreme sunt una reciproca celeilalte,
ceea ce nseamn c oricare dintre ele ar putea fi considerat teorem direct.
Dac teorema direct i reciproca ei sunt ambele adevrate, atunci le putem
concentra ntr-o singur teorem, folosind n formularea enunului expresia "dac i
numai dac". Iat un exemplu: "Un numr este divizibil cu 15 dac i numai dac el
este divizibil cu 3 i cu 5". Pentru demonstrarea unei astfel de teoreme trebuie s
facem, de fapt, dou demonstraii: directa i reciproca.
Sunt situaii cnd o teorem (propoziie) poate s admit mai multe reciproce.
Aceasta se ntmpl atunci cnd ipoteza sau concluzia teoremei (propoziiei) date (sau
chiar ambele) conine (conin) dou sau mai multe afirmaii (pri). n acest caz, numim
propoziie reciproc a unei propoziii date acea propoziie n care "ipoteza" este
format din concluzia propoziiei date (sau numai o parte a concluziei) i o parte din
ipoteza propoziiei date, iar "concluzia" este format din partea rmas a ipotezei
propoziiei date (i ceea ce a mai rmas din concluzia propoziiei date).


3.1 Congruena triunghiurilor

Cazurile de congruen a triunghiurilor oarecare
Fie ABC i MNP dou triunghiuri oarecare. Notaia ABCMNP o citim:
"triunghiul ABC este congruent cu triunghiul MNP" i nelegem prin aceasta ase
congruene, care au loc n acelai timp i anume:
[PM] [CA] [NP], [BC] MN], [ ] AB [ ;
MPN ACB MNP, ABC NMP, BAC .
Pentru a scrie cele ase congruene se ine seama c:
1) Laturile i unghiurile celor dou triunghiuri se corespund n ordinea dat
(scris) de congruena celor dou triunghiuri. Ele se mai numesc i elemente (laturi sau
unghiuri) "omoloage". (Cuvntul "omolog" vine din limba greac: homologus = n
armonie).
2) Laturile i unghiurile celor dou triunghiuri congruente, care se corespund
(omoloage), sunt congruente.
Se observ c din ABCMNP nu rezult ABCMPN, dar rezult c
ACBMPN i nc alte patru astfel de relaii (BACNMP, BCANPM,
CABPMN, CBAPNM).
Ilustrm grafic elementele care sunt respectiv congruente:


A
B C P N
M
96
Pentru a arta c dou triunghiuri sunt congruente nu este necesar s
demonstrm toate cele ase congruene ntre elementele lor.
Urmtoarele afirmaii, care se numesc "cazurile de congruen a triunghiurilor
oarecare" sunt adevrate:
Cazul 1 (LUL). Dou triunghiuri oarecare care au cte dou laturi i unghiul
cuprins ntre ele respectiv congruente sunt congruente.


Dac [AB][NM], [BC][NP] i ABCMNP, atunci rezult c
ABCMNP. Ca urmare, rezult c [AC][MP], ACBMPN i BACNMP.
Cazul 2 (ULU). Dou triunghiuri oarecare care au cte o latur i unghiurile
alturate ei respectiv congruente sunt congruente.


Dac [BC][NP], ABCMNP, ACBMPN, atunci rezult c
ABCMNP. Ca urmare, rezult c [AB][MN], [AC][MP] i BACNMP.
Cazul 3 (LLL). Dou triunghiuri oarecare care au laturile respectiv congruente
sunt congruente.


A
B C
M
N P
A
B C
M
N
P
A
B C
M
N P

97
Dac [AB][MN], [BC][NP], [AC][MP], atunci rezult c ABCMNP.
Ca urmare, rezult c BACNMP, ABCMNP i ACBMPN.
Se obinuiete a se rezuma astfel: cazaul 1: LUL (adic, latur-unghi-latur),
cazul 2: ULU (unghi-latur-unghi) i cazul 3: LLL (latur-latur-latur).
Aceste trei cazuri rezult direct din construcia triunghiului.
Remarc. Cazul LUU (latur-unghi-unghi): Dou triunghiuri oarecare care au
cte o latur, unghiul opus ei i un unghi alturat ei, respectiv congruente sunt
congruente.

Dac [BC][NP], BACNMP, ABCMNP, atunci rezult c
ABCMNP. Ca urmare, [AB][MN], [AC][MP] i ACBMPN.
Observaii. Cazul LUU nu rezult din construcia triunghiului, dar
considernd c suma msurilor unghiurilor unui triunghi este 180, adic
m(BAC)+m(ABC)+m(ACB)=180
i m(NMP)+m(MNP)+m(MPN)=180 i innd cont c m(BAC)=m(NMP) i
m(ABC)=m(MNP), rezult c i m(ACB)=m(MPN). Deci, acest caz se reduce
la ULU.

Cazurile de congruen a triunghiurilor dreptunghice
Definiie. Triunghiul care are un unghi drept se numete triunghi dreptunghic.
Laturile unui triunghi dreptunghic ABC, (m(A)=90) se numesc:
catete laturile alturate unghiului drept
ipotenuz latura opus unghiului drept.
[AB], [AC] sunt catete
[BC] este ipotenuza.
Toate triunghiurile dreptunghice au cte un unghi drept. Este de ateptat ca n
cazul particular al triunghiurilor dreptunghice, att cazurile de construcie ct i
cazurile de congruen s fie redate ntr-o form simplificat.
A
B C
M
N
P

C
A
B
98
Cazul C.C. (catet-catet): Dac dou triunghiuri dreptunghice au catetele
respectiv congruente, atunci ele sunt congruente.

Fie triunghiurile dreptunghice ABC i A'B'C' cu m(BAC)=m(B'A'C')=90,
[AB][A'B'] i [AC][A'C']. Putem afirma conform cazului de congruen LUL a
triunghiurilor oarecare c ABCA'B'C'.
Acest caz este o aplicaie direct a cazului LUL de la congruena triunghiurilor
oarecare.
Cazul C.U. (catet-unghi): Dac dou triunghiuri dreptunghice au o catet i
un unghi ascuit, la fel aezat fa de catet, respectiv congruente, atunci ele sunt
congruente.
Avem dou posibiliti:
I. Triunghiurile dreptunghice ABC i A'B'C' au congruente o catet i unghiul
ascuit alturat ei:

Fie triunghiurile dreptunghice ABC i A'B'C' cu m(BAC)=m(B'A'C')=90,
[AB][A'B'] i ABCA'B'C'. rezult, conform cazului de congruen ULU a
triunghiurilor oarecare, c ABCA'B'C'.
Acest caz este o aplicaie direct a cazului ULU de la congruena triunghiurilor
oarecare.
II. Triunghiurile dreptunghice ABC i A'B'C' au congruente o catet i unghiul
ascuit opus ei:


C
A
B
C'
A'
B'

C
A
B
C'
A'
B'

C
A
B
C'
A'
B'

99
Fie triunghiurile dreptunghice ABC i A'B'C' cu m(BAC)=m(B'A'C')=90,
[AB][A'B'] i ACBA'C'B'. Rezult, conform cazului de congruen LUU a
triunghiurilor oarecare, c ABCA'B'C'.
Acest caz este o aplicaie direct a cazului LUU de la congruena triunghiurilor
oarecare.
Cazul I.U. (ipotenuz-unghi): Dac dou triunghiuri dreptunghice au ipotenuza
i un unghi, diferit de unghiul drept, respectiv congruente, atunci sunt congruente.

Fie triunghiurile ABC i A'B'C' cu m(BAC)=m(B'A'C')=90, [BC][B'C']
i ABCA'B'C'. Rezult, conform cazului de congruen LUU a triunghiurilor
oarecare, c ABCA'B'C'.
Acest caz este o aplicaie direct a cazului LUU de la congruena triunghiurilor
oarecare.
Cazul I.C. (ipotenuz-catet): Dac dou triunghiuri dreptunghice au ipotenuza
i o catet respectiv congruente, atunci ele sunt congruente.

Fie triunghiurile dreptunghice ABC i A'B'C' cu m(BAC)=m(B'A'C')=90,
[BC][B'C'] i [AC][A'C']. Trebuie s demonstrm c ABCA'B'C'.
Prelungim [AC] cu [AP][AC] i [A'C'] cu [A'P'][A'C']; cum [AC][A'C'],
rezult c [AP][A'P'] i [CP][C'P']. tim c [AC][AP], m(BAC)=m(BAP)=90,
[AB] latur comun, rezult ABCABP, conform cazului de congruen catet-
catet. De aici, [BC][BP] (1).
Vom demonstra la fel c triunghiurile A'B'C' i A'B'P' sunt congruente:
[A'C'][A'P'], m(B'A'C')=m(B'A'P')=90 i [A'B'] latur comun, rezult c
A'B'C'A'B'P', conform cazului de congruen catet-catet. De aici, [B'C'][B'P'] (2).
Din [BC][B'C'] i relaiile (1) i (2) rezult c [BP][B'P'].
C
A
B
C'
A'
B'

C
A
P
B
C'
A'
P'
B'
100
Vom demonstra c triunghiurile BPC i B'P'C' sunt congruente: [BC][B'C'],
[BP][B'P'] i [CP][C'P'], rezult conform cazului de congruen LLL c
BPCB'P'C'. De aici, ACBA'C'B'.
Acum putem arta c triunghiurile ABC i A'B'C' sunt congruente:
[AC][A'C'], [BC][B'C'] i ACBA'C'B', rezult conform cazului de congruen
LUL c ABCA'B'C', ceea ce trebuia s demonstrm.

Probleme rezolvate
R3.1.1 n figura alturat [AB][BD], [AC][DC]. Atunci:
a) ABCDBC
b) ABEDBE
c) ACEDCE.

Soluie. a) Dac [AB][BD], [AC][DC], [BC] latur comun, atunci rezult,
conform cazului de congruen LLL, c ABCDBC.
b) Din a), ABCDBC, rezult c ABCDBC sau ABEDBE. Dac
[AB][BD], ABEDBE i [BE] este latur comun, rezult conform cazului de
congruen LUL, c ABEDBE.
c) Din b), ABEDBE, rezult c [AE][DE]. Dac [AC][CD], [AE][DE]
i [CE] este latur comun rezult conform cazului de congruen LLL, c
ACEDCE.
R3.1.2 n figura alturat: APBBQA i [AC][BE]. Atunci:
a) APEBQC
b) BAEABC
c) APQBQP.


Soluie. a) tim c
APBBQA, de unde rezult c
[AP][BQ], [PB][QA],
PABQBA. APBBQA i
PBAQAB. tim c
A
B
D
C
E

A
B
O
P
Q
E C
101
[AC][BE] i [AQ][PB], iar prin scdere membru cu membru PB - BE AQ - AC = ,
de unde rezult c [QC][EP]. m(APE)=180-m(APB) i m(BQC)=180-
m(BQA), dar APBBQA, de unde rezult c APEBQC.
Am artat c [AP][BQ], [EP][CQ] i APEBQC, deci, conform cazului
de congruen LUL, rezult c APEBQC.
b) tim c PABQBA, iar din a) rezult c PAEQBC, iar prin
nsumare, m(PAB)+m(PAE)=m(QBA)+m(QBC), de unde rezult c
m(BAE)=m(ABC), deci BAEABC. Avem [AB] latur comun,
BAEABC i ABEBAQ (este de fapt, PBAQAB) i conform cazului de
congruen ULU rezult c BAEABC.
c) tim c [AP][BQ], [AQ][BP], din APBBQA i [PQ] este latur
comun, deci APQBQP, conform cazului LLL de congruen a triunghiurilor.
R3.1.3 n figura alturat: [DP][AP], [PB][PC]. Atunci:
a) DPCAPB
b) CDABAD
c) DCBABC.


Soluie. a) tim c [DP][AP], [PC][PB] i DPCAPB, fiind unghiuri
opuse la vrf, rezult c DPCAPB, conform cazului de congruen LUL.
b) Se d: DP=AP, PB=PC i prin adunare membru cu membru rezult
DP+PB=AP+PC, adic DB=AC, de unde rezult c [DB][AC].
tim c [DB][AC], [AB][DC] (din a) DPCAPB) i [AD] latur comun,
rezult c CDABAD, conform cazului LLL de congruen a triunghiurilor.
c) tim c [DB][AC], [DC][AB] (din a)) i [BC] latur comun, rezult c
DCBABC, conform cazului LLL de congruen a triunghiurilor.
R3.1.4 n figura alturat: DACBCA, DCABAC, [DM][BN].
Atunci:
a) ADCCBA
b) AMCCNA
c) ADMCBN.
D
P
A
B
C
102


Soluie. a) tim c DACBCA, DCABAC i [AC] latur comun,
rezult c ADCCBA, conform cazului ULU de congruen a triunghiurilor.
b) Din a) rezult c [AB][DC], tim c [DM][BN], deci
DM DC BN AB = , de unde AN=CM, deci [AN][CM]. Avem: [AN][CM],
[AC] latur comun i MCANAC, de unde rezult conform cazului LUL, c
AMCCNA.
c) Din a) rezult c [AD][BC]; din c) rezult c [AM][CN] i se d
[DM][BN], deci ADMCBN, conform cazului LLL de congruen a triunghiurilor.


3.2. Metoda triunghiurilor congruente

n geometrie se folosete o metod de stabilire a adevrului caracteristic
tiinelor matematice: demonstraia. Demonstraia este o succesiune de judeci, de
argumente prin care se stabilete un adevr. n particular metoda triunghiurilor
congruente este o metod de demonstraie prin care se demonstreaz de regul c dou
segmente sau dou unghiuri sunt congruente.
Pentru rezolvarea problemelor de geometrie:
citii cu atenie problemele i folosind instrumentele potrivite, desenai figura
geometric
figura desenat s respecte proporiile elementelor date n enumul problemei
figura s fie suficient de mare i clar
pentru a demonstra c dou segmente sau c dou unghiuri sunt congruente, cutai
s le ncadrai n dou triunghiuri a cror congruen poate fi demonstrat
vei trage concluzia c segmentele sau unghiurile respective sunt congruente, dac
sunt elemente omoloage n triunghiuri congruente.

Probleme rezolvate

R3.2.1 Fie segmentele [AE] i [BD] care au un punct comun C, astfel nct
[AC][CD] i [CB][CE]. S se demonstreze c:
a) [AB][DE]
b) DEBABE
D M
C
A N B
103
c) CADCDA.


Ipotez: [AE][BD]={C}
[AC][CD]
[CB][CE]
Concluzie: a) [AB][DE]
b) DEBABE
c) CADCDA.
Demonstraie. a) Pentru a demonstra c dou segmente sunt congruente cutm
s le ncadrm n dou triunghiuri a cror congruen poate fi demonstrat.
Vom demonstra c triunghiurile ACB i DCE sunt congruente, deoarece:
[AC][DC], [CB][CE] i ACBDCE, fiind unghiuri opuse la vrf; rezult
conform cazului de congruen LUL c ACBDCE, de unde rezult c [AB][DE].
b) Pentru a demonstra c dou unghiuri sunt congruente cutm s le ncadrm
n dou triunghiuri a cror congruen poate fi demonstrat. Vom arta c triunghiurile
DEB i ABE sunt congruente.
Avem [AC][CD] i [EC][CB], din ipotez, de unde AC+CE=DC+CB, deci
AE=DB, adic [AE][DB]. Deci, [AE][DB], [AB][DE] (din punctul a) al problemei)
i [BE] latur comun, rezult c DEBABE, conform cazului de congruen a
triunghiurilor LLL. De aici rezult c: DEBABE.
c) Pentru a demonstra c dou unghiuri sunt congruente cutm s le ncadrm
n dou triunghiuri a cror congruen poate fi demonstrat. Vom arta c triunghiurile
EAD i BDA sunt congruente.
Avem [AE][BD], [DE][AB] (din punctul a) al problemei) i [AD] latur
comun, rezult c EADBDA, conform cazului de congruen a triunghiurilor
LLL. De aici rezult c: CADCDA.
R3.2.2 n ABD, O este mijlocul laturii [BD]. Se prelungete segmentul [AO]
cu segmentul [OC] astfel nct CDBABD. S se arate c:
a) [DC][AB]
b) OBCODA.

D
A
B
E
C
104

Ipotez: ABD
O[BD], [OB][OD]
O[AC]
CDBABD
Concluzie: a) [DC][AB]
b) OBCODA
Demonstraie. a) Vom demonstra c triunghiurile ODC i OBA sunt
congruente. Avem: [OD][OB], CDOABO (din ipotez) i DOCBOA, fiind
unghiuri opuse la vrf. Rezult conform cazului de congruen a triunghiurilor ULU c
ODCOBA. De aici rezult c [DC][AB].
b) Din ODCOBA rezult c [OC][OA]. Vom demonstra c triunghiurile
AOD i COB sunt congruente: [OD][OB] (din ipotez), [OA][OC] i
AODCOB, fiind unghiuri opuse la vrf. Rezult conform cazului de congruen a
triunghiurilor LUL c AODCOB. De aici rezult c ODAOBC.
R3.2.3 Fie segmentele congruente [OA] i [OB], astfel nct m(AOB)<90;
AEOB, EOB, BFOA, FOA, AEBF={P}.
S se demonstreze c:
a) [AE][BF]
b) [OP este bisectoarea unghiului AOB.
Ipotez: [OA][OB], m(AOB)<90
AEOB, EOB
BFOA, FOA
AEBF={P}
Concluzie: a) [AE][BF]
b) AOPBOP

A B
C
D
O
105

Demonstraie. a) Vom demonstra c triunghiurile AEO i BFO sunt
congruente. tim c [OA][OB], m(AEO)=m(BFO)=90 i AOB unghi comun
(din ipotez), de unde rezult conform cazului de congruen a triunghiurilor
dreptunghice I.U. c AEOBFO, de unde [AE][BF].
b) Din congruena AEOBFO rezult c [OE][OF]. tim c OA=OB,
OF=OE, rezult c OA-OF=OB-OE, deci FA=EB, de unde [FA][EB].
Avem: m(PFA)=m(PEB)=90 (din ipotez), [FA][EB] i FPAEPB
(unghiuri opuse la vrf), de unde rezuzlt conform cazului de congruen a
triunghiurilor dreptunghice C.U. c AFPBEP. De aici rezult c [PF][PE]. Avem:
[PF][PE], m(PFO)=m(PEO)=90 (din ipotez) i [OP] latur comun, de unde
rezult, conform cazului de congruen a triunghiurilor dreptunghice I.C. c
PFOPEO. De aici rezult c FOPEOP sau AOPBOP.
R3.2.4 Dac un punct aparine bisectoarei unui unghi, atunci el este egal
deprtat de laturile unghiului.
Reciproc: Dac un punct din interiorul unui unghi este egal deprtat de laturile
unghiului, atunci punctul aparine bisectoarei unghiului.
Ipotez: AOCCOB
M[OC
MEOA, EOA
MFOB, FOB
Concluzie: [ME][MF]

A
F
P
B E
O
106

Demonstraie. Vom demonstra c triunghiurile MEO i MFO sunt congruente:
m(MEO)=m(MFO)=90, MOEMOF (din ipotez), [OM] este latur comun,
rezult conform cazului de congruen a triunghiurilor dreptunghice I.U. c
MEOMFO. De aici rezult c [ME][MF].
Reciproc:
Ipotez: AOB
MIntAOB
MEOA, E[OA]
MFOB, F[OB]
[ME][MF]
Concluzie: MOAMOB
Demonstraie. Vom demonstra c triunghiurile MEO i MFO sunt congruente:
m(MEO)=m(MFO)=90, [ME][MF] (din ipotez), [OM] este latur comun,
rezult conform cazului de congruen a triunghiurilor dreptunghice I.C. c
MEOMFO. De aici rezult c MOEMOF sau MOAMOB.
Observaie. Cele dou teoreme de mai sus pot fi formulate ntr-o singur
teorem care se numete proprietatea bisectoarei unui unghi: Un punct aparine
bisectoarei unui unghi dac i numai dac este egal deprtat de laturile unghiului.
Remarc. Se numete loc geometric o mulime de puncte care sunt
caracterizate printr-o aceeai proprietate. Deci, bisectoarea unui unghi este locul
geometric al punctelor din interiorul unghiului egal deprtate de laturile unghiului.
Pentru ca figura presupus s fie loc geometric, trebuie satisfcute urmtoarele
dou propoziii:
a) orice punct al figurii s aib proprietatea enunat
b) orice punct care are proprietatea enunat s aparin figurii.
R3.2.5 Dac un punct aparine mediatoarei unui segment, atunci el este egal
deprtat de extremitile segmentului.
Reciproc: Dac un punct este egal deprtat de extremitile unui segment,
atunci punctul aparine mediatoarei segmentului.
Ipotez: [AB]
dAB
O
E
A
C
M
F B
107
dAB={M}
[MA][MB]
Pd
Concluzie: [PA][PB]

Demonstraie. Vom demonstra c triunghiurile PMA i PMB sunt congruente:
m(PMA)=m(PMB)=90, [MA][MB] (din ipotez) i [PM] latur comun, rezult
c PMAPMB, conform cazului de congruen a triunghiurilor dreptunghice C.C.
De aici rezult c [PA][PB].
Reciproc:
Ipoteza: [AB]
[PA][PB]
Concluzie: P aparine mediatoarei segmentului [AB].
Demonstraie. Construim perpendiculara din P pe AB, care intersecteaz AB n
M. Vom demonstra c triunghiurile PMA i PMB sunt congruente:
m(PMA)=m(PMB)=90, [PA][PB] (din ipotez) i [PM] latur comun, rezult c
PMAPMB, conform cazului de congruen a triunghiurilor dreptunghice I.C. De
aici rezult c [AM][MB], dar PMAB, rezult c PM este mediatoarea segmentului
[AB].
Observaie. Cele dou enunuri de mai sus pot fi formulate ntr-o singur
teorem care se numete proprietatea mediatoarei unui segment: Un punct aparine
mediatoarei unui segment dac i numai dac este egal deprtat de extremitile
segmentului.
Remarc. Mediatoarea unui segment este locul geometric al punctelor din plan
egal deprtate de extremitile segmentului.
R3.2.6 Fie triunghiurile ABC i A'B'C' n care construim ADBC, DBC,
A'D'B'C', D'B'C' i medianele [AM] i [A'M']. Dac [BC][B'C'], [AD][A'D'],
[AM][A'M'], demonstrai c ABCA'B'C'.
Ipotez: ABC, A'B'C'
ADBC, DBC
A'D'B'C', D'B'C'
M[BC], [MB][MC]

P
A M
B
d
108
M'[B'C'], [M'B'][M'C']
[BC][B'C']
[AD][A'D']
[AM][A'M']


Demonstraie. Dac [BC][B'C'] i M, M' sunt mijloacele segmentelor [BC],
respectiv [B'C'] rezult c [BM][MC][B'M'][M'C']. Putem demonstra c
triunghiurile ADM i A'D'M' sunt congruente, deoarece:
m(ADM)=m(A'D'M')=90, [AD][A'D'] i [AM][A'M'] (din ipotez), rezult,
conform cazului de congruen I.C. c ADMA'D'M'. De aici rezult c:
[DM][D'M']. Dar [BM][B'M'], de unde BM-DM=B'M'-D'M', deci BD=B'D', atunci
[BD][B'D'].
Putem demonstra c triunghiurile ADB i A'D'B' sunt congruente, deoarece:
m(ADB)=m(A'D'B')=90, [AD][A'D'] (din ipotez) i [BD][B'D'], rezult
conform cazului de congruen C.C. c ADBA'D'B'. De aici rezult c [AB][A'B']
i ABCA'B'C'.
Acum putem demonstra c triunghiurile ABC i A'B'C' sunt congruente,
deoarece: [AB][A'B'], ABCA'B'C' i [BC][B'C'] (din ipotez), rezult conform
cazului de congruen LUL c ABCA'B'C'.

3.3 Paralelism
3.3.1 Drepte paralele
Reamintim urmtoarele definiii i teoreme ce se vor folosi n acest paragraf:
Dou drepte distincte care au intersecia egal cu mulimea vid se numesc
drepte paralele.
Dac dou drepte intersectate de o secant formeaz o pereche de unghiuri
alterne interne congruente, atunci dreptele sunt paralele.
Dac dou drepte intersectate de o secant formeaz o pereche de unghiuri
alterne externe congruente, atunci dreptele sunt paralele.
Dac dou drepte intersectate de o secant formeaz o pereche de unghiuri
corespondente congruente, atunci dreptele sunt paralele.
Dac dou drepte intersectate de o secant formeaz o pereche de unghiuri
interne de aceeai parte a secantei suplementare, atunci dreptele sunt paralele.
B D M
C
A
A'
B' D' M' C'

109
Dac dou drepte intersectate de o secant formeaz o pereche de unghiuri
externe de aceeai parte a secantei suplementare, atunci dreptele sunt paralele.
Axioma paralelelor (axioma lui Euclid). Printr-un punct exterior unei drepte
date se poate duce o singur paralel la acea dreapt.
Observaie. Axioma lui Euclid ne asigur att de existena unei paralele dus
printr-un punct exterior la dreapt, ct i de unicitatea acestei paralele.
Euclid (Eukleides) a fost matematician grec (sec. 3 .H.). El a ntemeiat o
coal n Alexandria (Egipt). Este autorul primei expuneri sistematice a cunotinelor
de geometrie intitulat "Elemente", care a constituit cartea de cpti a geometriei timp
de 2000 de ani.
Consecinele axiomei paralelelor:
1) Dou drepte paralele cu o a treia sunt paralele ntre ele.
2) Dac dou drepte sunt paralele, atunci orice dreapt care se intersecteaz cu
una dintre ele se va intersecta i cu cealalt.
Dac dou drepte paralele sunt intersectate de o secant, atunci ele formeaz
unghiuri alterne interne congruente dou cte dou, unghiuri alterne externe
congruente dou cte dou, unghiuri corespondente congruente dou cte dou,
unghiuri interne de aceeai parte a secantei suplementare i unghiuri externe de aceeai
parte a secantei suplementare.
Remarc. Teoremele de mai sus pot fi enunate i n felul urmtor:
Dou drepte tiate de o secant sunt paralele dac i numai dac unghiurile:
a) alterne interne sunt congruente dou cte dou;
b) alterne externe sunt congruente dou cte dou;
c) corespondente sunt congruente dou cte dou;
d) interne de aceeai parte a secantei sunt suplementare;
e) externe de aceeai parte a secantei sunt suplementare.

Unghiuri cu laturile respectiv paralele
Teorem. Dou unghiuri cu laturile respectiv paralele sunt congruente dac
sunt ambele ascuite sau ambele obtuze i sunt suplementare dac unul este ascuit, iar
cellalt este obtuz.

O
C
A''
O'
B''
B'
B
A'
A
O
1
B'
1 O'
1
A
1
''
' '
1
B
B
1
A'
1
A
1

110
Demonstraie. n primul caz, cnd ambele unghiuri sunt ascuite, de exemplu:
fie AOB, A'O'B' dou unghiuri astfel nct OA||O'A' i OB||O'B'. Notnd
OBO'A'={C}, din OA||O'A' intersectate de secanta OB rezult AOBA'CB
(unghiuri corespondente), apoi din OB||O'B' intersectate de secanta O'A' rezult
A'CBA'O'B' (unghiuri corespondente), deci AOBA'O'B'.
Teorema este adevrat i pentru A''O'B'', care este opus la vrf un unghiul
A'O'B', adic avem AOBA''O'B''.
n al doilea caz, cnd un unghi este ascuit i cellalt obtuz, fie A
1
O
1
B
1
i
'
1
'
1
'
1
B O A dou unghiuri astfel nct
'
1
'
1 1 1
A O || A O ,
'
1
'
1 1 1
B O || B O , m(A
1
O
1
B
1
)<90,
> 90 ) B O A m(
'
1
'
1
'
1
. Fie
' '
1
'
1
B [O semidreapta opus semidreptei
'
1
'
1
B [O ; rezult c
unghiurile
' '
1
'
1
'
1 1 1 1
B O A B O A , fiind unghiuri cu laturile respectiv paralele ambele
ascuite, dar
' '
1
'
1
'
1
B O A i
'
1
'
1
'
1
B O A sunt unghiuri suplementare, atunci i A
1
O
1
B
1

i
'
1
'
1
'
1
B O A sunt suplementare.
Teorema este adevrat i pentru
' '
1
'
1
' '
1
B O A , care este opus la vrf cu unghiul
'
1
'
1
'
1
B O A , adic avem = + 180 ) B O A m( ) B O A m(
' '
1
'
1
' '
1 1 1 1
.
Model 1. Segmentel [AB] i [CD] au acelai mijloc O. S se demonstreze c
AC||BC.
Ipotez: ABCD={O}
[OA][OB]
[OC][OD]
Concluzie: AC||BD
Demonstraie. Putem demonstra c triunghiurile AOC i BOD sunt congruente:
[OA][OB], [OC][OD] (din ipotez), AOCBOD (opuse la vrf), atunci
AOCBOD (cazul LUL), de unde rezult c OACOBD; deci, dreptele AC i
BD intersectate de secanta AB formeaz o pereche de unghiuri alterne interne
congruente, rezult c AC||BD.
Model 2. n triunghiul ABC, D(AC), E(AB), M(BC), astfel nct
DM||AB, EM||AC. Dac m(EMB)=67 i m(DMC)=43, s se calculeze msurile
unghiurilor triunghiului ABC.
Ipotez: ABC, D(AC), E(AB), M(BC)
DM||AB, EM||AC
m(EMB)=67
m(DMC)=43
Concluzie: m(BAC)
D B
O
A
C
111
m(ABC)
m(ACB)

Demonstraie. Dreptele paralele DM i AB intersectate de secanta BC
formeaz o pereche de unghiuri corespondente congruente, deci
m(DMC)=m(ABC)=43; dreptele paralele EM i AC intersectate de secanta BC
formeaz o pereche de unghiuri corespondente congruente, deci
m(EMB)=m(ACB)=67. Putem afla m(EMD):
m(EMD)=180-(m(DMC)+m(EMB)),
m(EMD)=180-(67+43)=180-110=70.
Dreptele paralele AB i DM tiate de secanta EM formeaz o pereche de
unghiuri alterne interne congruente, deci m(BEM)=m(EMD)=70; dreptele paralele
EM i AC tiate de secanta AB formeaz o pereche de unghiuri corespondente
congruente, deci m(BEM)=m(BAC)=70.
Msurile unghiurilor triunghiului ABC sunt: m(BAC)=70, m(ABC)=43
i m(ACB)=67.


112
Probleme rezolvate

R3.3.1 S se demonstreze c paralele duse prin vrfurile unui triunghi la
laturile opuse determin un triunghi n care vrfurile triunghiului dat sunt mijloace de
laturi.
Ipotez: ABC
APN, PN||BC
BPM, PM||AC
CMN, MN||AB
Concluzie: [AP][AN]
[BP][BM]
[CM][CN]
Demonstraie. Dreptele paralele PN i BC intersectate de secanta AC formeaz
o pereche de unghiuri alterne interne congruente, NACACB; dreptele paralele AB
i MN intersectate de secanta AC formeaz o pereche de unghiuri alterne interne
congruente, ACNBAC.
Avem: NACACB, ACNBAC, [AC] latur comun, atunci
ABCCNA (cazul ULU), de unde rezult c:
[AN][BC] (1)
i
[CN][AB] (2)
Dreptele paralele PN i BC intersectate de secanta AB formeaz o pereche de
unghiuri alterne interne congruente, PABABC; dreptele paralele PM i AC
intersectate de secanta AB formeaz o pereche de unghiuri alterne interne congruente,
PBABAC.
Avem: PABABC, PBABAC, [AB] latur comun, atunci
ABCBAP (cazul ULU), de unde
[AP][BC] (3)
i
[BP][AC] (4)
Dreptele paralele AB i MN intersectate de secanta BC formeaz o pereche de
unghiuri alterne interne congruente, BCMABC; dreptele paralele PM i AC
intersectate de secanta BC formeaz o pereche de unghiuri alterne interne congruente,
CBMACB.
P
A N
B
C
M
113
Avem: BCMABC, CBMACB, [BC] latur comun, atunci
MCBABC (cazul ULU), de unde
[CM][AB] (5)
i
[BM][AC] (6)
Din relaiile (1) i (3) rezult c [AN][AP].
Din relaiile (2) i (5) rezult c [CN][CM].
Din relaiile (4) i (6) rezult c [BP][BM].
R3.3.2 n triunghiul ABC, [BD] este median, D(AC). Construim CE||BD,
unde EAB. Demonstrai c B este mijlocul segmentului [AE].
Ipotez: ABC
D(AC), [DA][DC]
CE||BC, EAB
Concluzie: [AB][BE]
Demonstraie. Pentru demonstrarea acestei probleme facem urmtoarea
construcie auxiliar: BF||AC, F(CE).
Vom demonstra c triunghiurile BCD i CBF sunt congruente: DBCBCF
(unghiuri alterne interne, BD||CE i BC secant), DCBCBF (unghiuri alterne
interne, BF||AC i BC secant), [BC] latur comun, atunci BCDCBF (cazul
ULU), de unde rezult c [CD][BF], dar [CD][AD] (din ipotez), deci [AD][BF].
Vom demonstra c triunghiurile FBE i DAB sunt congruente: [AD][BF],
EBFBAD (unghiuri corespondente, BF||AC i AE secant), EFBECA
(unghiuri corespondente, BF||AC i EC secant), dar ECABDA (unghiuri
corespondente, BD||EC i AC secant), deci EFBBDA; atunci FBEDAB
(cazul ULU), de unde rezult c [BE][AB].
R3.3.3 Fie M, N, P mijloacele laturilor [AB], [BC], [AC] ale unui triunghi
ABC. Fie D i E astfel nct [MD][MC], M(DC), [PE][PN], P(NE). S se
demonstreze c punctele A, D, E sunt coliniare.
Ipotez: ABC
M(AB), [MA][MB]
N(BC), [NB][NC]
A
D
C
F
B
E
114
PAC), [PA][PC]
M(DC), [MC][MD]
P(EN), [PN][PE]
Concluzie: A, D, E coliniare
Demonstraie. Vom demonstra c triunghiurile MAD i MBC sunt congruente:
[MA][MB], [MD][MC] (din ipotez), AMDBMC (unghiuri opuse la vrf),
atunci MADMBC (cazul LUL), de unde rezult c MABMBC, deci dreptele
AD i BC intersectate de secanta AB formeaz o pereche de unghiuri alterne interne
congruente, rezult c AD||BC.
Vom demonstra c triunghiurile APE i CPN sunt congruente: [AP][PC],
[PE][PN] (din ipotez), APENPC (unghiuri opuse la vrf), atunci APECPN
(cazul LUL), de unde rezult c EAPPCN, deci dreptele AE i BC intersectate de
secanta AC formeaz o pereche de unghiuri alterne interne congruente, rezult c
AE||BC.
Avem AD||BC, AE||BC i conform axiomei paralelelor, dreptele AD i AE
coincid, deci A, D, E sunt coliniare.

3.3.2. Linia mjlocie ntr-un triunghi

ntr-un triunghi, segmentul determinat de mijloacele a dou laturi se numete
linie mijlocie.
Remarc. Un triunghi are trei linii mijlocii.
Fie M, N, P mijloacele laturilor [AB], [AC], respectiv [BC] ale triunghiului
ABC. Segmentele [MN], [NP], [PM] sunt cele trei linii mijlocii ale triunghiului ABC.
Teorem. Segmentul care unete mijloacele a dou laturi ale unui triunghi este
paralel cu cea de a treia latur i are lungimea egal cu jumtate din lungimea acestei
laturi.
D
A
E
M
P
B N C
C P B
N
M
A
115
Conform acestei teoreme, avem MN||BC,
2
BC
MN = , NP||AB,
2
AB
NP = ,
MP||AC,
2
AC
MP = .
Reciproca. Dac prin mijlocul M al laturii [AB] din triunghiul ABC, se duce
MN||BC, N(AC), atunci N este mijlocul laturii (AC) i
2
BC
MN = .
Demonstraie. Demonstraia se face prin metoda reducerii la absurd.
Presupunem c N nu este mijlocul laturii [AC], atunci exist un punct N', N'(AC),
N'N, astfel nct [N'A][N'C]. Conform definiiei liniei mijlocii, rezult c [MN'] este
linie mijlocie, deci MN'||BC, dar MN||BC (din ipotez) i MN'MN; contradicie cu
axioma paralelelor.
Presupunerea fcut este fals, rezult c N este mijlocul lui [AC]; deci [MN]
este linie mijlocie, atunci
2
BC
MN = .
Model. Fie M, N, P mijloacele laturilor [AB], [AC] respectiv [BC] ale
triunghiului ABC.
a) Artai c unghiurile triunghiurilor AMN, MBP, NPC, PNM sunt
congruente cu unghiurile triunghiului ABC.
b) Triunghiurile AMN, MBP, NPC, PNM sunt congruente. (Triunghiul PNM
se numete triunghi median sau triunghi complementar triunghiului ABC, M, N, P
fiind "picioarele" medianelor triunghiului ABC).
Ipotez: ABC
M(AB), [MA][MB]
N(AC), [NA][NC]
P(BC), [PB][PC]
Concluzie: a) AMN, MBP, NPC, PNM au unghiurile congruente
cu unghiurile ABC
b) AMNMBPNPCPNM.
B C
N
N'
M
A
116
Demonstraie. a) Conform definiiei liniei mijlocii, [MN], [NP] i [MP] sunt
liniile mijlocii ale triunghiului ABC, deci MN||BC, NP||AB i MP||AC.
n triunghiul AMN avem: MANBAC, AMNABC (unghiuri
corespondente, MN||BC, AB secant), ANMACB (unghiuri corespondente,
MN||BC, AC secant).
n triunghiul MBP, la fel, avem: MBPABC, BMPBAC,
BPMACB (unghiuri corespondente, MP||AC, AB secant, respectiv BC).
n triunghiul NPC, la fel, avem: PCNACB, PNCBAC,
NPCABC (unghiuri corespondente, NP||AB, AC secant, respectiv BC).
n triunghiul MPN avem: MPNPNC (alterne interne, MP||AC, PN
secant), dar PNCBAC, atunci MPNBAC; MNPNPC (alterne interne,
MN||BC, PN secant), dar NPCABC, atunci MNPABC; NMPMPB
(alterne interne, MN||BC, MP secant), dar MPBACB, atunci NMPACB.
c) Segmentele [MN], [NP], [MP] sunt liniile mijlocii ale triunghiului ABC,
rezult c PC BP
2
BC
MN = = = , MB AM
2
AB
NP = = = ,
NC AN
2
AC
MP = = = .
Avem AMNMBPNPCPNM (cazul LLL).

Probleme rezolvate

R3.3.4 n triunghiul ABC punctele D i E sunt mijloacele laturilor [AB],
respectiv [BC]. Dac G este punctul de intersecie al dreptelor AE i CD, demonstrai
c 2DG=GC i 2EG=GA.
Ipotez: ABC
E(BC), [BE][EC]
D(AB), [DA][DB]
AECD={G}
Concluzie: 2DG=GC
2EG=GA
C P B
N
M
A
117
Demonstraie. Fie M i N mijloacele segmentelor [AG], respectiv [CG].
Segmentele [MN] i [DE] sunt linii mijlocii n triunghiurile GAC, respectiv ABC, de
unde rezult c MN||AC,
2
AC
MN = i DE||AC,
2
AC
DE = , deci MN||DE,
[MN][DE].
Vom demonstra c triunghiurile GMN i GED sunt congruente: [MN][DE],
GMNGED, GNMGDE (alterne interne, MN||DE, ME secant, respectiv
ND), atunci GMNGED, de unde rezult c [GM][GE] i [GN][GD], dar
[GM][AM] i [GN][NC], deci GA=2GM=2EG i GC=2GN=2GD.
R3.3.5 S se demonstreze c dreapta determinat de vrful A al unui triunghi
ABC i mijlocul medianei din B intersecteaz latura [BC] ntr-un punct E, astfel nct
BC
3
1
BE = .
Ipotez: ABC
D(AC), [DA][DC]
P(BD), [PB][PD]
APBC={E}
Concluzie: BC
3
1
BE =
Demonstraie. Fie DF||AE, F(BC).
n AEC aplicm reciproca liniei mijlocii: D este mijlocul laturii [AC],
DF||AE, F(EC), atunci F este mijlocul laturii [EC], deci [EF][FC].
A
D
M
G
N
C E
B
A
D
B E F C
P
118
n BDF aplicm reciproca liniei mijlocii: P este mijlocul laturii [BD], PE||DF,
E(BF), atunci E este mijlocul laturii [BF], deci [BE][EF], dar [EF][FC], rezult c
BC
3
1
FC EF BE = = = .
R3.3.6 Dou triunghiuri ABC i ADE au mediana [AM] comun,
{M}=BCDE. Fie P, Q, R, S, respectiv mijloacele laturilor [AB], [AC], [AD], [AE].
Artai c RQ||PS i [RQ][PS].
Ipotez: ABC, ADE
{M}=BCDE
[MB][MC]
[MD][ME]
P(AB), [PA][PB]
Q(AC), [QA][QC]
R(AD), [RA][RD]
S(AE), [SA][SE]
Concluzie: RQ||PS, [RQ][PS]
Demonstraie. Din ipotez avem [MB][MC] i [ME][MD], iar
BMECMD (unghiuri opuse la vrf), atunci BMECMD, de unde rezult c
[BE][CD] i MBEMCD, deci dreptele CD i BE intersectate de secanta BC
formeaz o pereche de unghiuri alterne interne congruente, rezult c ele sunt paralele,
CD||BE. Segmentele [RQ] i [PS] sunt linii mijlocii n triunghiurile ADC, respectiv
ABE, deci RQ||DC, DC
2
1
RQ = i PS||BE, BE
2
1
PS = , dar DC||BE i [DC][BE],
rezult c RQ||PS i [RQ][PS].

3.3.3. Suma msurilor unghiurilor unui triunghi

n rezolvarea problemelor din acest paragraf vom folosi urmtoarele rezultate,
demonstrate la orele de geometrie:
Suma msurilor unghiurilor unui triunghi este 180.
Consecine:
A
R
D
P
S
Q
C
M
B
E
119
1) Toate unghiurile triunghiului echilateral au msura de 60.
2) n orice triunghi dreptunghic, unghiurile ascuite sunt complementare.
Unghiurile ascuite ale unui triunghi dreptunghic isoscel au msura de 45.
3) n orice triunghi poate exista cel mult un unghi drept sau obtuz.
Msura unui unghi exterior al unui triunghi este egal cu suma msurilor celor
dou unghiuri ale triunghiului, neadiacente lui.
Remarc. Bisectoarea unui unghi exterior al unui triunghi se numete
bisectoare exterioar a triunghiului, corespunztoare unghiului respectiv.
[BE este bisectoare interioar
[BF este bisectoare exterioar
Bisectoarea interioar i bisectoarea exterioar duse din acelai vrf al unui
triunghi sunt perpendiculare.
Demonstraie. Unghiurile ABC i ABD sunt adiacente suplementare, deci
m(ABC)+m(ABD)=180. de unde avem = + 90 ABD) m(
2
1
ABC) m(
2
1
,
adic m(ABE)+m(ABF)=90, deci m(EBF)=90, de unde rezult c BEBF.

Probleme rezolvate

R3.3.7 n interiorul unui triunghi isoscel ABC, [AB][AC] i
m(BAC)<120, se consider un punct M, astfel nct m(MBC)=30 i
m(MCB)=15. Notm cu P intersecia dreptei BM cu nlimea din A a triunghiului
ABC. S se afle msurile unghiurilor PMC i BPC.
Ipotez: ABC: [AB][AC]
m(BAC)<120
MIntABC
m(MBC)=30
m(MCB)=15
ADBC, D(BC)
ADBM={P}
Concluzie: m(PMC), m(BPC)
A
E
C B D
F
120

Demonstraie. Unghiul PMC este unghi exterior triunghiului MBC, de unde
rezult c m(PMC)=m(MBC)+m(MCB)=30+15=45.
Vom demonstra c triunghiurile ADB i ADC sunt congruente:
m(ADB)=m(ADC)=90, [AB][AC] (din ipotez), [AD] este latur comun, atunci
ADBADC (cazul I.C.), de unde rezult c [DB][DC].
Acum putem demonstra congruena triunghiurilor PDB i PDC:
m(PDB)=m(PDC)=90 (din ipotez), [DB][DC], [PD] latur comun, atunci
PDBPDC (C.C.), de unde rezult c PBCPCB, dar m(PBC)=30 (din
ipotez), atunci
m(BPC)=180-2m(PBC)=180-230=120.
R3.3.8 n triunghiul ABC avem m(B)=3m(A). Mediatoarea laturii [BC]
intersecteaz dreapta AC n punctul E, astfel nct BAEBEA. S se calculeze
msurile unghiurilor triunghiului ABC.
Ipotez: ABC: m(B)=3m(A)
F(BC), [FB][FC]
FEBC, E(AC)
BAEBEA
Concluzie: m(BAC), m(ABC), m(ACB)
Demonstraie. Notm m(BAC)=x, atunci m(ABC)=3x i m(BEA)=x.
Dac suma msurilor unghiurilor unui triunghi este 180, atunci m(ACB)=180-
(m(BAC)+m(ABC)), deci m(ACB)=180-4x i m(ABE)=180-
A
E
C F
B
121
(m(BAE)+m(BEA)), deci m(ABE)=180-2x. Atunci, m(EBC)=m(ABC)-
m(ABE), deci m(EBC)=3x-(180-2x), efectund calculele se obine
m(EBC)=5x-180. (1)
Vom demonstra c triunghiurile EFB i EFC sunt congruente:
m(EFB)=m(EFC)=90, [FB][FC] (din ipotez), [EF] latur comun, atunci
EFBEFC (cazul C.C.), de unde rezult c EBFECB, dar
m(ECB)=m(ACB)=180-4x, atunci
m(EBF)=m(EBC)=180-4x (2)
Din relaiile (1) i (2) deducem c 5x-180=180-4x sau 9x=360, de unde
x=40.
Vom avea: m(ABC)=340=120, m(BAC)=40, m(C)=20.
R3.3.9 Unghiurile A, B, C ale unui triunghi ABC au msurile invers
proporionale cu numerele 0,(3);
7
1
; 0,125. Fie M(AB) i N(AC), astfel nct
m(ACM)=40 i m(ABN)=20. S se determine msurile unghiurilor triunghiului
ABC i msura unghiului ANM.
Ipotez: {m(A),m(B),m(C)}, i.p.
)
`

8
1
,
7
1
,
3
1

M(AB), N(AC)
m(ACM)=40
m(ABN)=20
Concluzie: m(A), m(B), m(C)
m(ANM)

Demonstraie. ntre mulimile {m(A),m(B),m(C)} i
)
`

8
1
,
7
1
,
3
1
se
stabilete o proporionalitate invers, atunci ntre {m(A),m(B),m(C)} i {3,7,8}
se stabilete o proporionalitate direct, deci se poate scrie:
122
8
C) m(
7
B) m(
3
A) m(
=

, de unde aplicnd proprietatea irului de rapoarte egale,


avem mai departe:
=

=
+ +
+ +
=

10
18
180
8 7 3
) ( ) ( ) (
8
C) m(
7
B) m(
3
A) m( C m B m A m

Din =

10
3
) ( A m
, rezult c m(A)=30; din =

10
7
) ( B m
, rezult c
m(B)=70; din =

10
8
) ( C m
, rezult c m(C)=80.
tim c m(ACM)=40, atunci m(MCB)=m(ACB)-m(ACM), efectund
obinem m(MCB)=40.
tim c m(ABN)=20, atunci m(NBC)=m(ABC)-m(ABN), efectund
obinem m(NBC)=50.
Fie BNCM={P}. n triunghiul PBC:
m(PBC)+m(PCB)=50+40=90,
deci m(BPC)=90, de unde rezult c CPBN. n triunghiul CPN, m(PCN)=40 i
m(CPN)=90, rezult c
m(CNP)=90-m(PCN)=50.
Vom demonstra c triunghiurile CPB i CPN sunt congruente:
m(CPB)=m(CPN)=90, [CP] latur comun i m(CBP)=m(CNP)=50, atunci
CPBCPN (cazul C.U.), de unde rezult c [PB][PN]. Acum putem demonstra c
triunghiurile MPB i MPN sunt congruente: m(MPB)=m(MPN)=90, [MP] latur
comun i [PB][PN], atunci MPBMPN (cazul C.C.), de unde rezult c
MNPMBP, dar m(MBP)=20, de unde rezult c MNPMBP, dar
m(MBP)=20, atunci m(MNP)=20. Acum putem afla m(ANM):
m(ANM)=180-m(MNP)+m(PNC)), nlocuind obinem m(ANM)=180-
(20+50), deci m(ANM)=110.


3.4. Triunghiul isoscel i triunghiul echilateral

n rezolvarea problemelor din acest paragraf vom folosi urmtoarele definiii i
teoreme ce sunt demonstrate n manuale:
Definiie. Triunghiul care are dou laturi congruente se numete triunghi
isoscel.
Notaie. ABC, [AB][AC]
[BC] se numete baza triunghiului isoscel.

123

Teorem. Unghiurile opuse laturilor congruente ale unui triunghi isoscel sunt
congruente.
Reciproc. Dac un triunghi are dou unghiuri congruente, atunci el este
triunghi isoscel (laturile opuse unghiurilor congruente sunt congruente).
Remarc. Un triunghi este isoscel dac i numai dac are dou unghiuri
congruente.
Teorem. Dac un triunghi este isocel i se consider bisectoarea unghiului
opus bazei, atunci ea este i mediana corespunztoare bazei i nlimea corespun-
ztoare bazei i este inclus n mediatoarea bazei.
Afirmaiile de mai sus rmn valabile i pentru mediana corespunztoare bazei
i pentru nlimea corespunztoare bazei.
Teorem. Dac un triunghi este isoscel i se
consider mediana corespunztoare bazei, atunci ea
este i bisectoarea unghiului opus bazei i nlimea
corespunztoare bazei i este inclus n mediatoarea
bazei.
Demonstraie. Fie ABC isoscel, [AB][AC]
i [AD] mediana corespunztoare bazei [BC]; D(BC)
i [DB][DC]. Avem conform cazului de congruen a
triunghiurilor LLL c ADBADC, de unde rezult
c BADCAD, ADBADC, dar
m(ADB)+m(ADC)=180, deci
m(ADB)=m(ADC)=90, ADBC; ADBC i
[DB][DC], D(BC), rezult c dreapta AD este mediatoarea segmentului [BC].
Teorem. Dac un triunghi este isoscel i se consider nlimea
corespunztoare bazei, atunci ea este i bisectoarea unghiului opus bazei i mediana
corespunztoare bazei i este inclus n mediatoarea bazei.
Demonstraie. Fie ABC isoscel, [AB][AC] i [AD] nlimea
corespunztoare bazei, ADBC, D(BC).
A
B C
A
B C
D
124
Avem conform cazului de congruen a
triunghiurilor dreptunghice I.C., c ADBADC,
de unde rezult c BADCAD i [BD][CD];
ADBC i [DB][DC], D(BC), rezult c dreapta
AD este mediatoarea segmentului [BC].
Observaie. n triunghiul isoscel ABC,
[AB][AC], dreapta AD, care conine att
bisectoarea unghiului BAC, ct i nlimea,
mediana i mediatoarea corespunztoare laturii [BC],
este ax de simetrie a triunghiului.
Vom demonstra i alte proprieti ale
triunghiului isoscel.
Teorem. Dac ntr-un triunghi bisectoarea unui unghi este i mediana
corespunztoare laturii opuse unghiului, atunci triunghiul este isoscel.
Ipotez: ABC
BADCAD, D(BC)
[DB][DC]
Concluzie: [AB][AC]

Demonstraie. Demonstrarea acestei
teoreme necesit o construcie auxiliar: fie EAD,
D(AE), [AD][DE].
Vom demonstra c triunghiurile ADB i
EDC sunt congruente: [AD][DE] (prin
construcie), [BD][CD] (din ipotez) i
ADBEDC (unghiuri opuse la vrf); rezult
conform cazului de congruen a triunghiurilor
LUL c ADBEDC. De aici, rezult c
[AB][CE] i BADCED, dar BADCAD
(din ipotez), atunci CEDCAD, deci CAE
este un triunghi isoscel, [CA][CE]. Avem:
[AB][CE] i [AC][CE], de unde rezult c
[AB][AC].
Teorem. Dac ntr-un triunghi bisectoarea
unui unghi este i nlime, atunci triunghiul este
isoscel.
Ipotez: ABC





BADCAD, D(BC)

D
A
B C

B
D
C
A
E
125
ADBC
Concluzie: [AB][AC]


Demonstraie. Se poate demonstra c triunghiurile ADB i ADC sunt
congruente: m(ADB)=m(ADC)=90 (din ipotez), BADCAD (din ipotez),
[AD] latur comun; rezult conform cazului de congruen a triunghiurilor
dreptunghice C.U. c ADBADC, de unde [AB][AC].
Teorem. Dac ntr-un triunghi mediana corespunztoare unei laturi este i
nlime, atunci triunghiul este isoscel.
Ipotez: ABC
D(BC), [DB][DC]
ADBC
Concluzie: [AB][AC]


Demonstraie. Vom demonstra c triunghiurile ADB i ADC sunt congruente:
m(ADB)=m(ADC)=90 (din ipotez), [DB][DC] (din ipotez), [AD] latur
comun; rezult conform cazului de congruen a triunghiurilor dreptunghice C.C. c
ADBADC, de unde [AB][AC].

D
A
B C
D
A
B C
126
n rezolvarea problemelor din acest paragraf sunt utile definiia triunghiului
echilateral i urmtoarele proprieti ale sale, considerate cunoscute:
Definiie. Triunghiul care are toate laturile congruente se numete triunghi
echilateral.
Notaie. [AB][BC][AC]

Teorem. Unghiurile unui triunghi echilateral sunt congruente, avnd msurile
egale cu 60.
Reciproc. Dac ntr-un triunghi unghiurile sunt congruente, atunci triunghiul
este echilateral.
Observaie. Reciproca de mai sus se poate enuna i n felul urmtor: dac un
triunghi are dou unghiuri cu msurile de 60, atunci el este echilateral. (Este evident
c i al treilea unghi al triunghiului are msura de 60, deci cele trei unghiuri ale
triunghiului sunt congruente).
Remarc. Un triunghi isoscel care are un unghi cu msura de 60 este un
triunghi echilateral.
Comparnd definiia triunghiului echilateral cu cea a triunghiului isoscel vor
rezulta noi proprieti specifice triunghiului echilateral (deoarece triunghiul echilateral
poate fi considerat ca fiind triunghi isoscel cu oricare din laturi ca baz):
Teorem. ntr-un triunghi echilateral toate liniile importante ce pornesc din
acelai vrf coincid.
Observaie. Triunghiul echilateral are trei axe de simetrie.

A
B C
127

Cu ajutorul proprietilor triunghiului isoscel i echilateral putem demonstra
dou proprieti ale triunghiului dreptunghic ce vor fi foarte des folosite n rezolvarea
problemelor de geometrie.
Teorem. Dac ntr-un triunghi dreptunghic msura unui unghi este de 30,
atunci lungimea catetei opuse acestui unghi este jumtate din lungimea ipotenuzei.
Ipotez: ABC: m(A)=90
m(B)=30
Concluzie:
2
BC
AB=


Demonstraie. Fie C'AC, astfel nct A(CC'), [AC][AC'].
n triunghiul BCC', [BA] este nlime (din ipotez) i median (din
construcie), deci el este un triunghi isoscel, dar m(BCA)=60, rezult c BCC este
triunghi echilateral. Avem
2
CC'
AC = (din construcie), dar CC'=BC, rezult c
2
BC
AC= .
Teorem. ntr-un triunghi dreptunghic, lungimea medianei corespunztoare
ipotenuzei este jumtate din lungimea ipotenuzei.
A
B
C
128
Ipotez: ABC: m(A)=90
O(BC), [OB][OC]
Concluzie:
2
BC
AO =


Demonstraie. Fie PAO, astfel nct O(AP) i [OA][OP]. Vom demonstra
c triunghiurile AOB i POC sunt congruente: [AO][PO] (prin construcie),
[BO][CO] (din ipotez), AOBCOP (unghiuri opuse la vrf); rezult conform
cazului de congruen a triunghiurilor LUL c AOBPOC, de unde [AB][PC] i
OABOPC; din ultima congruen rezult c CP||AB (deoarece tiate de secanta
AP formeaz o pereche de unghiuri alterne interne congruente). Avem ABAC,
CP||AB, atunci CPAC. Vom demonstra c triunghiurile PCA i BAC sunt
congruente: m(PCA)=m(BAC)=90, [AC] latur comun, [PC][BA]; atunci,
conform cazului de congruen a triunghiurilor dreptunghice C.C. avem c
PCABAC, deci [AP][BC] i cum
2
AP
AO = , rezult c
2
BC
AO = .
Observaie. Se remarc relaia [AO][BO][CO], punctul O este egal deprtat
de punctele A, B, C, este centrul cercului circumscris triunghiului dreptunghic ABC.


C
P
A B
O
129
Probleme rezolvate

R3.4.1 Se d triunghiul isoscel ABC, cu [AB][AC]. Pe latura [AB] se iau
punctele M, N, P, astfel nct [AM][MN][NP][PB] i pe latura [AC] se iau punctele
E, F, G, astfel nct [AE][EF][FG][GC]. Fie {S}=PEBC, {T}=MGBC i
{H}=EPMG. Artai c:
a) [MG][PE]; b) [MT][ES]; c) AHBC; d) SMPTEG.
Ipotez: ABC: [AB][AC]
M, N, P(AB)
[AM][MN][NP][PB]
E, F, G(AC)
[AE][EF][FG][GC]
PEBC={S}
MGBC={T}
EPMG={H}
Concluzie: a) [MG][PE]
b) [MT][ES]
c) AHBC
d) SMPTEG.


Demonstraie. a) Avem
4
AB
PB NP MN AM = = = = i
4
AC
GC FG EF AE = = = = ,
dar AB=AC, de unde rezult c
A
F
G
M
E
N
H
P
S
B C T
130

4
AB
GC FG EF AE PB MN AM = = = = = = = (1).
Vom demonstra c triunghiurile AMG i AEP sunt congruente: [AM][AE],
A este unghi comun,
4
3AB
AP AG = = (din relaia (1)), atunci, conform cazului de
congruen a triunghiurilor LUL avem c AMGAEP, de unde rezult c
[MG][PE].
b) Din AMGAEP, rezult c AMGAEP, de unde i suplementele lor
vor fi egale, deci BMTCES.
Avem: BMTCES, MBTECS (la baza triunghiului isoscel ABC),
4
3AB
EC MB = = (din relaia (1)). Atunci, conform cazului de congruen a
triunghiurilor ULU, MBTECS, de unde rezult c [MT][ES].
c) Din MBTECS rezult c MTSEST, atunci triunghiul HST este
isoscel, avnd dou unghiuri congruente, deci [HT][HS]; dar [MT][ES] i scznd
membru cu membru obinem MT-HT=ES-HS, de unde MH=EH. Avem:
4
AB
AE AM = = (din relaia (1)), [MH][EH], [AH] latur comun, atunci
AMHAEH (cazul LLL de congruen a triunghiurilor), de unde rezult c
MAHEAH. Avem: n triunghiul isoscel ABC de baz [BC], [AH este bisectoarea
unghiului BAC, deci AH este i nlimea corespunztoare bazei, prin urmare AHBC.
d) Din AEPAMG, rezult c APEAGM, deci ele au suplemente
egale, prin urmare MPSEGT.
Din AEPAMG, rezult c [PE][GM]; avem [ES][MT] i scznd
membru cu membru se obine ES-PE=MT-GM, de unde [PS][GT].
Avem:
2
AB
EG MP = = (din relaia (1)), MPSEGT, [PS][GT], atunci,
rezult conform cazului de congruen LUL a triunghiurilor c SMPTEG.
R3.4.2 Demonstrai c dou triunghiuri ABC i A'B'C' care au perimetrele
egale, [CA][C'A'] i CC', sunt congruente.
Ipotez: ABC, A'B'C'
P
ABC
=P
A'B'C'

[CA][C'A]
CC'
Concluzie: ABCA'B'C'

A
D
B C
A'
D'
B' C'

131
Demonstraie. Se consider punctele D i D' astfel nct B(DC), [BD][AB],
B'(D'C'), [B'D'][A'B']; DC=DB+BC=AB+BC=P
ABC
-AC i
D'C'=D'B'+B'C'=A'B'+B'C'=P
A'B'C'
-A'C', dar [AC][A'C'] (din ipotez), de unde rezult
c [DC][D'C'].
Vom demonstra c triunghiurile ADC i A'D'C' sunt congruente: [DC][D'C'],
[AC][A'C'] i CC' (din ipotez); atunci avem ADCA'D'C' (cazul LUL), de
unde rezult c ADCA'D'C' i DACD'A'C'.
Triunghiul ABD este isoscel, [AB][BD], atunci unghiurile de la baz sunt
congruente, BDABAD; la fel, triunghiul A'B'D' este isoscel, [A'B'][B'D'], atunci
B'D'A'B'A'D', dar ADBA'D'B', deci ADBA'D'B'BADB'A'D' i
DACD'A'C', de unde rezult c BACB'A'C' (diferene de unghiuri
congruente).
Avem: BACB'A'C', [AC][A'C'] i CC', atunci ABCA'B'C'
(cazul ULU de congruen a triunghiurilor).
R3.4.3 n triunghiul ABC, cu AC
3
2
AB= i m(BAC)=60, ducem mediana
[CM], M(AB). Demonstrai c [CM][CB].
Ipotez: ABC
AC
3
2
AB=
m(BAC)=60
M(AB): [MA][MB]
Concluzie: [CM][CB]



Demonstraie. Se face o construcie auxiliar: se ia NAB, B(MN) i
[BM][BN], dar AC
3
1
AC
3
2
2
1
AB
2
1
MA MB = = = = , deci
132
AC
3
1
BN MB AM = = = , de unde rezult c AC
3
1
3 3AM AN = = , AN=AC.
Avem: [AN][AC] i m(BAC)=60, de unde rezult c ANC este echilateral.
Avem: [AM][BN], MACBNC (unghiuri ale triunghiului echilateral
ANC) i [AC][NC], atunci MACBNC (conform cazului LUL de congruen a
triunghiurilor), de unde rezult c [CM][CB].
R3.4.4 Un triunghi este isoscel dac i numai dac are dou nlimi
congruente.
I. nlimile corespunztoare laturilor congruente ale unui triunghi isoscel sunt
congruente.
Ipotez: ABC: [AB][AC]
BEAC, EAC
CFAB, FAB
Concluzie: [BE][CF]
Distingem dou cazuri, dup cum m(BAC)<90 sau m(BAC)>90:


Demonstraie. Artm c triunghiurile FBC i ECB sunt congruente:
m(CFB)=m(BEC)=90, [BC] latur comun, FBCECB (la baza triunghiului
isoscel), atunci FBCECB (cazul I.U.), de unde rezult c [CF][BE].
II. Dac un triunghi are dou nlimi congruente, atunci el este triunghi
isoscel.
Ipotez: ABC
BEAC, EAC
CFAB, FAB
[BE][CF]
Concluzie: [AB][AC]
Demonstraie. Artm c triunghiurile FBC i ECB sunt congruente:
m(CFB)=m(BEC)=90. [BC] latur comun, [CF][BE], atunci FBCECB
(cazul I.C.), de unde rezult c FBCECB, deci ABC este isoscel, [AB][AC],
avnd dou unghiuri congruente.
A
F E
B C
A
E F
B C

133
Remarc. Demonstraiile celor dou probleme de mai sus se pot face
considernd triunghiurile AEB i AFC.
R3.4.5 Un triunghi este isoscel dac i numai dac are dou mediane
congruente.
I. Medianele corespunztoare laturilor congruente ale unui triunghi isoscel sunt
congruente.
Ipotez: ABC: [AB][AC]
M(AC), [MA][MC]
N(AB), [NA][NB]
Concluzie: [BM][CN]


Demonstraie. Avem: AC
2
1
MC AM = = , AB
2
1
NB AN = = , dar AB=AC,
deci [AM][MC][AN][NB].
Vom demonstra c triunghiurile NBC i MCB sunt congruente: [BC] latur
comun, [NB][MC], NBCMCB (la baza triunghiurilui isoscel ABC), atunci
NBCMCB, de unde rezult c [BM][CN].
Remarc. Demonstraia se putea face considernd triunghiurile AMB i ANC.
Observaie. Notm BMCN={G}, centrul de greutate al triunghiului ABC. n
plus se poate demonstra c GBC este isoscel (din NBCMCB rezult c
NCBMBC) i c AGBC ( MG GB - MB GC - NC NG = = = ; [AN][AM] i
[AG] latur comun, atunci ANGAMG (LLL), de unde rezult c NAGMAG,
dar ABC este isoscel de baz [BC], deci bisectoarea unghiului BAC este i nlime,
atunci AGBC).
II. Dac un triunghi are dou mediane congruente, atunci el este isoscel.
Ipotez: ABC
M(AC). [MA][MC]
N(AB), [NA][NB]
[BM][CN]
Concluzie: [AB][AC]
A
N M
B C
G
134


Demonstraie. Pentru a putea arta c triunghiurile NBC i MCB sunt
congruente, trebuie s demonstrm c NCBMBC.
Construcia auxiliar: fie MP||NC, PBC. Avem NCBMPB, fiind
unghiuri corespondente i PMCNCB, fiind unghiuri alterne interne.
Conform definiiei, [MN] este linie mijlocie n ABC, deci MM||BC, de unde
rezult c NMCMCP, unghiuri alterne interne.
Avem: NMCMCP, NCMPMC, [MC] latur comun, atunci
NMCPCM (ULU), de unde rezult c [MP][NC], dar [MB][NC], deci
[MP][MB]. Triunghiul MBP este isoscel, [MP][MB], de unde rezult c
MPBMBC, dar MPBNCB, deci MBCNCB. Avem: MBCNCB,
[BC] latur comun, [MB][NC] (din ipotez), atunci NBCMCB (cazul LUL), de
unde rezult c NBCMCB, deci ABC este isoscel, [AB][AC] deoarece are
dou unghiuri congruente.
R3.4.6 Un triunghi este isoscel dac i numai dac are dou bisectoare
congruente.
I. Bisectoarele unghiurilor congruente ale unui triunghi isoscel sunt
congruente.
Ipotez: ABC: [AB][AC]
ABDDBC, D(AC)
ACEECB, E(AB)
Concluzie: [BD][CE]


Demonstraie. Unghiurile de la baza unui triunghi isoscel sunt congruente i
[BD, [CE sunt bisectoarele lor, deci ABDDBCACEECB. Avem:
EBCDBC (la baza triunghiului isoscel), ECBDBC, [BC] latur comun,
atunci EBCDCB (ULU), de unde rezult c [BD][CE].
A
N
M
B
C P
135
Remarc. Demonstraia se putea face
considernd triunghiurile ADB i AEC.
Observaie. Notm BDCE={I}, centrul
cercului nscris n triunghiul ABC. Avem
IBCICB, de unde rezult c triunghiul IBC este
isoscel. Se mai poate demonstra c AIBC
( DI IB - DB IC - EC EI = = = ;
AD DC - AC EB - AB AE = = = ; [AI] latur
comun, atunci AIEAID (LLL), de unde rezult
c EAIDAI, dar ABC este isoscel de baz [BC],
bisectoarea unghiului BAC este i nlime, deci
AIBC).
II. Dac un triunghi are dou bisectoare
congruente, atunci el este isoscel.
Ipotez: ABC
ABDDBC, D(AC)
ACEECB, E(AB)
[BD][CE]
Concluzie: [AB][AC]


Demonstraie. Folosim metoda reducerii la absurd. Presupunem c
[AC] / [AB] . Ar nsemna, de exemplu, c AC<AB. Ar rezulta c
m(ABC)<m(ACB), deci m(DBC)<m(ECB) (1) i s-ar obine
DC<EB (2).
Vom considera paralele prin D la BE i prin E la BD i vom nota cu F
intersecia acestor paralele.
A
E D
B
C
F

A
E D
I
B
C
136
Avem: BEDEDF (alterne interne), BDEDEF (alterne interne) i
[DE] latur comun, atunci EDBDEF (cazul ULU), de unde rezult [BE][DF].
Inegalitatea (2) s-ar scrie DC<DF, atunci n CDF, am avea
m(DFC)<m(DCF) (3).
Din EDBDEF rezult c EBDEFD, dar i EBDDBC, atunci
EFDDBC. innd seama de (1) am putea scrie: m(EFD)<m(ECB) sau
m(EFD)<m(ECD) (4).
Adunnd membru cu membru inegalitile (3) i (4) am obine:
m(DFC)+m(EFD)<m(DCF)+m(ECD) sau m(EFC)<m(ECF). Ar nsemna c
n triunghiul ECF: EC<EF (unghiului cu msur mai mic i se opune o latur mai
mic). Dar cum [EC][BD] (din ipotez), ar nsemna c BD<EF, ceea ce este absurd
deoarece [BD][EF] (din EDBDEF).
Am ajuns la un rezultat absurd pentru c am pornit de la o presupunere fals,
anume aceea c ] AC [ / ] AB [ . Deci, triunghiul ABC este isoscel, [AB][AC].
R3.4.7 n triunghiul dreptunghic ABC, m(A)=90 i m(B)=15; se
construiesc nlimea [AD], bisectoarea [AE] i mediana [AO]. Dac DE=a i CD=b, s
se calculeze perimetrul triunghiului AOE.
Ipotez: ABC: m(A)=90
m(B)=15
ADBC, D(BC)
CAEEAB, E(BC)
O(BC), [OB][OC]
Concluzie: P
AOE



Demonstraie. Mediana corespunztoare ipotenuzei are lungimea egal cu
jumtate din lungimea ipotenuzei, deci AO=OB=OC, de unde rezult c AOB este
isoscel i m(OAB)=m(OBA)=15, iar m(AOC)=30, fiind unghi exterior
triunghiului OAB.
Avem [AE bisectoarea unghiului BAC, deci
m(EAB)=m(EAC)=45, m(OAB)=15,
atunci
m(EAO)=45-15=30, dar m(EOA)=30,
deci triunghiul AOE este isoscel, [AE][EO].
137
Avem triunghiul dreptunghic ADC (m(ADC)=90) i m(C)=75, deci
m(CAD)=15, iar m(DAE)=m(CAE)-m(CAD)=45-15=30. n triunghiul
dreptunghic ADE (m(ADE)=90), m(DAE)=30, iar DE=a, rezult c AE=2DE=2a
(cateta opus unghiului de 30 este jumtate din ipotenuz), dar [AE][EO], atunci
AE=EO=2a.
tim c CD=b, DE=a, EO=2a, OC=CD+DE+EO, deci OC=b+3a, dar OC=OA,
rezult OA=3a+b. Putem calcula perimetrul triunghiului AOE:
P
AOE
=AO+AE+EO=3a+b+2a+2a=7a+b.
R3.4.8 n triunghiul ABC, [AB][AC], m(BAC)=20. Fie E(AB) i
D(AC), astfel nct m(ACE)=30. m(ABD)=20. Aflaia msura unghiului
AED.
Ipotez: ABC: [AB][AC]
m(BAC)=20
E(AB)
D(AC)
m(ACE)=30
m(ABD)=20
Concluzie: m(AED)

Demonstraie. n triunghiul isoscel ABC, [AB][AC], m(BAC)=20, atunci
=

= = 80
2
20 180
ACB) m( ABC) m( , m(ACE)=30, rezult c
m(ECB)=m(ACB)-m(ACE)=80-30=50.
n triunghiul BEC calculm
m(BEC)=180-m(EBC)+m(ECB))=180-(80+50)=50,
deci m(BEC)=m(BCE)=50, atunci BEC este isoscel, [BC][BE] (1).
Construim F(AC) astfel nct m(FBC)=20. dar m(BCF)=80, rezult c
m(BFC)=180-(m(BCF)+m(FBC))=180-100=80, atunci BFC este isoscel,
[BC][BF] (2).
138
Din (1) i (2) rezult c [BF][BE] i cum m(EBF)=60, atunci BEF este
echilateral, [EF][BF] (3).
Avem:
m(DBF)=m(EBC)-(m(EBD)+m(FBC))=80-220=40
i m(BDC)=180-(m(DBC)+m(DCB))=180-(60+80)=40, deci FDB este
isoscel, [FD][FB] (4).
Din (3) i (4) rezult c [EF][FD], deci FED este isoscel, dar m(EFD)=180-
(m(EFB)+m(BFC))=180-(60+80)=40, atunci
=

= = 70
2
40 - 180
FDE) m( FED) m( , i cum m(FEB)=60, rezult c
= + = FED)) m( FEB) m( ( 180 ) AED m(
= = = + 50 130 180 ) 70 60 ( 180 .


3.5. Patrulatere

Importana se reflect n locul pe care-l ocup n programa colar, patru-
laterele fiind studiate att n clasa a VI-a ct i n clasa a VII-a. Poate i multitudinea
aplicaiilor practice i confer temei un mare avantaj.
Vom reaminti cteva definiii i teoreme de care avem nevoie n abordarea
temei.
Definiia 3.5.1. Un patrulater se numete convex dac, oricare ar fi o latur a
sa, celelalte dou vrfuri, nesituate pe latura considerat, se afl de aceeai parte a
dreptei n care este inclus latura respectiv.
Teorema 3.5.1. Suma msurilor unghiurilor unui patrulater convex este de
360.

Paralelogramul
Definiia 3.5.2. Patrulaterul convex cu laturile opuse paralele se numete
paralelogram.
Proprieti
Teorema 3.5.2. n orice paralelogram laturile opuse sunt congruente i reci-
procele:
- orice patrulater convex n care laturile opuse sunt congruente este parale-
logram
- orice patrulater convex n care dou laturi opuse sunt congruente i paralele
este paralelogram.
Teorema 3.5.3. n orice paralelogram unghiurile opuse sunt congruente i
reciproc, orice patrulater convex n care unghiurile opuse sunt congruente este parale-
logram.
Teorema 3.5.4. n orice paralelogram diagonalele se intersecteaz n pri
congruente i reciproc, orice patrulater convex n care diagonalele se intersecteaz n
pri congruente este paralelogram.
139

Construcia paralelogramelor
1) Trasm dou drepte paralele pe care le intersectm cu alte dou drepte
paralele. Punctele de intersecie vor fi vrfurile paralelogramului.
2) Desenm dou segmente paralele i congruente. Extremitile lor vor fi
vrfurile paralelogramului.
3) Intersectm dou segmente necongruente care au acelai mijloc.
Extremitile acestor segmente vor fi vrfurile paralelogramului.

Dreptunghiul
Definiia 3.5.3. Paralelogramil cu un unghi drept se numete dreptunghi.

Proprieti
Teorema 3.5.5. n orice dreptunghi toate unghiurile sunt congruente i deci
drepte i reciproc orice patrulater convex n care toate unghiurile sunt congruente i
deci drepte este dreptunghi.
Teorema 3.5.6. n orice dreptunghi diagonalele sunt congruente i reciproc,
orice paralelogram cu diagonalele congruente este dreptunghi.

Construcia dreptunghiului
1) Desenm un triunghi dreptunghic i prin vrfurile unghiurilor ascuite
ducem paralele la catete care se vor intersecta ntr-un punct ce va fi al patrulea vrf al
dreptunghiului (vrfurile triunghiului dreptunghic vor fi celelalte trei vrfuri).
2) Desenm dou segmente congruente care s aib acelai mijloc.
Extremitile lor vor fi vrfurile dreptunghiului.

Rombul
Definiia 3.5.4. Paralelogramul care are dou laturi consecutive congruente se
numete romb.

Proprieti
Teorema 3.5.7. ntr-un romb toate laturile sunt congruente i reciproc orice
patrulater cu toate laturile congruente este romb.
Teorema 3.5.8. ntr-un romb diagonalele sunt perpendiculare i reciproc,
orice paralelogram cu diagonalele perpendiculare este romb.
Teorema 3.5.9. ntr-un romb diagonalele sunt bisectoarele unghiurilor i reci-
proc, orice paralelogram n care o diagonal este i bisectoarea unui unghi este romb.

Construcia rombului
1) Desenm dou drepte perpendiculare. Fixm pe fiecare dreapt dou puncte
simetrice fa de a doua dreapt astfel ca segmentele formate pe prima dreapt s nu fie
congruente cu cele de pe a doua dreapt. Cele patru puncte sunt vrfurile rombului.
2) Desenm un triunghi isoscel (vrfurile lui vor fi trei dintre vrfurile
rombului). Construim simetricul vrfului triunghiului fa de baz. Acesta va fi al
patrulea vrf al rombului.

140
Ptratul
Definiia 3.5.5. Dreptunghiul cu dou laturi consecutive congruente se
numete ptrat.

Construcia ptratului
1) Desenm dou drepte perpendiculare, iar cu centrul n punctul lor de
intersecie trasm un cerc. Interseciile cercului cu cele dou drepte perpendiculare vor
fi cele patru vrfuri ale ptratului.
2) Desenm un unghi drept i lum pe laturile lui dou segmente congruente,
ambele avnd unul din capete n vrful unghiului. Prin capetele segmentelor diferite de
vrful unghiului ducem paralele la laturile unghiului care se vor intersecta ntr-un
punct (al patrulea vrf al ptratului).
3) Desenm un triunghi dreptunghic isoscel i apoi construim simetricul
unghiului drept fa de ipotenuz.

Trapezul
Definiia 3.5.6. Patrulaterul care are dou laturi paralele i celelalte dou
neparalele.
Trapezul este isoscel cnd laturile neparalele sunt congruente.
Teorema 3.5.10. Unghiurile alturate bazelor unui trapez isoscel sunt congru-
ente i reciproc, dac unghiurile alturate bazelor unui trapez sunt congruente, trapezul
este isoscel.
Teorema 3.5.11. Diagonalele unui trapez isoscel sunt congruente i reciproc
dac diagonalele unui trapez sunt congruente trapezul este isoscel.

Vom prezenta n continuare probleme rezolvate n care sunt evideniate
proprietile patrulaterelor.


Probleme rezolvate

R3.5.1 Pe laturile unui paralelogram, ca baze, se construiesc n afar
triunghiuri echilaterale. S se demonstreze c vrfurile acestor triunghiuri, diferite de
vrfurile paralelogramului sunt vrfurile unui paralelogram.
Demonstraie. Fie ABCD paralelogramul i triunghiurile ABF, ADE, DQC,
CBG, echilaterale construite pe laturile paraleloagramului.
141
Avem: FGBQED (LUL), deoarece
(BF)(QD)
FBGQDE (240-m(B) i 240-m(D), DB)
(BG)(DE)
Din congruena triunghiurilor de mai sus obinem c
(FG)(EQ) (1)
i triunghiurile AEF i CGQ sunt congruente (LUL), deoarece
(AE)(CG)
(EAF)(GCQ)
(AF)(CQ)
Din congruena triunghiurilor AEF i CGQ rezult
(EF)(GQ) (2)
Din relaiile (1) i (2) rezult c patrulaterul EQGF are laturile opuse
congruente i deci este paralelogram.
R3.5.2 n patrulaterul convex ABCD, L este mijlocul laturii (AB) i P este
mijlocul laturii (CD). S se demonstreze c mijloacele segmentele AP, CL, BP, DL
sunt vrfurile unui paralelogram.
Demonstraie. Fie T mijlocul lui (AP), R mijlocul lui (DL), E mijlocul lui
(LC), iar Q mijlocul lui (PB) i O mijlocul lui (PL).
E
A B
C D
G
F
Q
142
n triunghiul APL, [TO] este linie mijlocie. Deci TO||AL i
2
AL
TO = . n
triunghiul LPB, [OQ] este linie mijlocie. deci OQ||LB i
2
LB
OQ = . Prin punctul O
trece o singur paralel la AB, deci punctele T, O, Q sunt coliniare. Fiindc (AL)(LB)
i
2
AL
TO = iar
2
LB
OQ = , obinem c (TO)(QO), adic O este mijlocul
segmentului (TQ) (1).
n triunghiul DLP, (RO) este linie mijlocie, deci
RO||DP i
2
DP
RO = .
n triunghiul PLC, (OE) este linie mijlocie, deci
OE||PC,
2
PC
OE = .
Prin O trece o singur paralel la DC, rezult c punctele R, O, E sunt
coliniare. Din (DP)(PC) i
2
DP
RO = ,
2
PC
EO = rezult
(RO)(OE) (2)
Din (1) i (2) rezult c diagonalele patrulaterului TRQE se taie n pri
congruente, deci el este paralelogram.
R3.5.3 Se consider patrulaterul convex ABCD n care m(A)+m(C)=180.
n exteriorul patrulaterului se construiesc dreptunghiurile ABPF, BCRH, CDTQ,
DAML astfel ca (BP)(CD), (AM)(BC), (DT)(AB), (CR)(AD). Demonstrai c
centrele acestor dreptunghiuri sunt vrfurile unui nou dreptunghi.
O
E
Q
R
T
A L
B
C
P
D
143
Fie O
1
, O
2
, O
3
, O
4
centrele dreptunghiurilor ABPF, CBHR, CQTD, DLMA.
Trebuie s artm c O
1
O
2
O
3
O
4
este dreptunghi, adic paralelogram cu un unghi drept.
[O
1
O
2
] este linie mijlocie n triunghiul FBR. Deci
2
FR
O O
2 1
= .
n triunghiul MQD, [O
4
O
3
] este linie mijlocie, deci
2
MQ
O O
3 4
= .
Pentru a demonstra c [O
1
O
2
][O
4
O
3
] este suficient s artm c [FR][MQ].
Suma msurilor unghiurilor formate n jurul unui punct este 360.
n punctul B, m(ABP)+m(ABC)+m(CBH)+m(HBP)=360, dar
m(ABP)+m(CBH)=180, deci
m(ABC)+m(HBP)=180 (1)
Suma msurilor unghiurilor unui patrulater convex este 360. Deci
m(ABC)+m(BCD)+m(CDA)+m(DAB)=360.
Din ipotez m(DAB)+m(BCD)=180, i atunci obinem c
m(ABC)+m(CDA)=180 (2)
Comparnd relaiile (1) i (2) obinem:
m(HBP)=m(CDA) (3)
Din triunghiurile dreptunghice congruente FPB i QCD ce au (FP)(CQ) i
(BP)(CD) obinem c
(FBP)(QDC), (FB)(DQ) (4)
i triunghiurile dreptunghice BRH i DMA sunt congruente, avnd
(BH)(AD) i (RH)(AM) i atunci
(RBH)(MDA), (BR)(MD) (4*)
Atunci
O
1
Q
L
M
O
4
D
A
F P
B
O
2
H
R
C
O
3
T

144
m(FBR)=360-[m(FBP)+m(PBH)+m(HBR)] (5)
iar
m(MDQ)=360-[m(QDC)+m(CDA)+m(MDA)] (6)
Din (5) i (6) obinem c
FBRMDQ (7)
deoarece (FBP)(QDC), (PBH)(CDA) i (HBR)(MDA).
Din relaiile (4), (4*), (7) obinem c:
FBRQDM (LUL) (8)
Din congruena triunghiurilor de la (8) obinem c (FR)(QM), de unde
2
QM
2
FR
= , adic
O
1
O
2
=O
4
O
3
(9)
Analog se arat c i (O
1
O
4
)(O
3
O
2
).
Atunci patrulaterul O
1
O
2
O
3
O
4
, avnd laturile opuse congruente este
paralelogram. Pentru a fi dreptunghi mai trebuie s artm c are i un unghi drept.
Avem m(O
2
O
1
O
4
)=m(AO
1
B)-[m(AO
1
O
4
)+m(BO
1
O
2
)] i
m(O
4
O
3
O
2
)=m(CO
3
D)+m(CO
3
O
2
)+m(DO
3
O
4
).
Triunghiurile AO
1
O
4
i CO
3
O
2
sunt congruente deoarece
3 1
CO
2
CT
2
AP
AO = = =
2 4
CO
2
CH
2
AL
AO = = =
3 2 4 1
O O O O =
Din congruena celor dou triunghiuri rezult c AO
1
O
4
CO
3
O
2
.
Analog se arat c BO
1
O
2
DO
3
O
4
. Atunci
m(O
4
O
1
O
2
)+m(O
2
O
3
O
4
)=m(AO
1
B)+m(CO
3
D)
Triunghiurile O
1
BP i DO
3
C, avnd toate laturile congruente sunt congruente,
de unde rezult c m(DO
3
C)m(BO
1
P). Obinem
m(O
4
O
1
O
2
)+m(O
2
O
3
O
4
)=m(AO
1
B)+m(BO
1
P)=180
Deci m(O
4
O
1
O
2
)+m(O
2
O
3
O
4
)=180. Dar ntr-un paralelogram unghiurile
opuse sunt congruente, deci O
4
O
1
O
2
O
2
O
3
O
4
atunci fiind i suplementare rezult
c m(O
4
O
1
O
2
)=90.
Paralelogramul O
4
O
1
O
2
O
3
avnd un unghi drept este dreptunghi.
R3.5.4 Se consider ptratul ABCD iar M i N sunt mijloacele laturilor AB,
respectiv AD. Dac BDCN={L}, BDCM={K} i BCNK={T} s se arate c
patrulaterul ALCK este romb.
Demonstraie. Fie O intersecia diagonalelor ptratului ABCD.
n triunghiul ABC, CM i BO sunt mediane, rezult c punctul K este centrul
de greutate al triunghiului. Deci
OB
3
1
OK = (1)
145
n triunghiul ADC, CN i DO sunt mediane, rezult c punctul L este centrul
de greutate al triunghiului. Deci
DO
3
1
OL = (2)
Dar (OB)(DO) i atunci din (1)
i (2) obinem c OK=OL, adic O este
mijlocul segmentului (LK). Dar
(OA)(OC) i ACLK. Patrulaterul
ALCK are diagonalele perpendiculare i
se njumtesc, rezult c el este romb.
R3.5.5 Se consider triunghiul
isoscel ABC cu (AB)(AC) i AB=2BC.
Se duce nlimea AP, mediana BM, i
bisectoarea CL. Pe latura AC se ia un
punct D astfel ca (BD)(BC).
Fie {N}=BDCL. S se
demonstreze c patrulaterul BLMN este
romb.
Demonstraie. Fie {O}=LNBM.
n triunghiul isoscel BCM cu (BC)(CM),
CL este bisectoare, deci i median i nlime. Rezult c diagonalele patrulaterului
LBNM sunt perpendiculare.
Punctele N i L fiind situate pe mediatoarea segmentului [BM] avem
relaiile:(NB)(NM) i (LM)(LB). Triunghiul ABC fiind isoscel cu BC, iar
triunghiul CBD isoscel cu BCDCDB, rezult c CBDCAB.
n triunghiul isoscel BCM avem:
m(CMB)m(CBM) (1)
B P C
D
N
O
L
M
A

O
D C
N
L
K
T
B
M
A
146
Unghiul CMB este exterior BMA, atunci
m(CMB)=m(MAB)+m(MBA) (2)
i
m(CBM)=m(CAB)+m(DBM) (3)
Din (1), (2) i (3) rezult c m(MBA)=m(DBM), adic (BM este
bisectoarea NBL.
n triunghiul BLN, (BO fiind bisectoare i nlime rezult c el este isoscel cu
(BL)(BN). Obinem c patrulaterul BNML are toate laturile congruente i deci el este
paralelogram n care diagonalele sunt perpendiculare, deci el este romb.
R3.5.6 Pe laturile unui ptrat ABCD se construiesc, n exterior triunghiurile
echilaterale ABE, BCT, CDR, DAP, care au respectiv centrele de greutate G
1
,G
2
,G
3
,G
4
.
Demonstrai c patrulaterul G
1
G
2
G
3
G
4
este ptrat.
Demonstraie.
Triunghiurile echilaterale ABE, BCT, CDR, DAP sunt congruente. Atunci
segmentele AG
1
, BG
1
, BG
2
, CG
2
, CG
3
, DG
3
, DG
4
, AG
4
sunt congruente avnd fiecare
lungimea ca fiind
3
2
din nlimea unui triunghi echilateral (din cele considerate).
Unghiurile G
4
AG
1
, G
1
BG
2
, G
3
CG
2
, G
4
DG
3
sunt congruente avnd msura
(30+90+30) 150. (Am inut seama c mediana n triunghiul echilateral este i
bisectoare.) Atunci triunghiurile isoscele G
4
AG
1
, G
1
BG
2
, G
2
CG
3
, G
3
DG
4
sunt
congruente (LUL). Din congruena celor patru triunghiuri de mai sus obinem c
(G
1
G
2
)(G
2
G
3
)(G
3
G
4
)(G
4
G
1
), adic patrulaterul G
1
G
2
G
3
G
4
este romb.
O
G
3
C D
P
G
4
A
B
T
R
E
G
1
G
2

147
n triunghiul isoscel G
1
AB avem m(G
1
AB)=m(G
1
BA)=30. Din triunghiul
isoscel AG
1
G
4
cu m(G
1
AG
4
)=150 obinem m(AG
1
G
4
)=15.
Analog din triunghiul isoscel G
1
GB
2
obinem m(BG
1
G
2
)=15. Atunci
m(G
4
G
1
G
2
)=m(AG
1
B)-[m(AG
1
G
4
)+m(BG
1
G
2
)]=
=120-(15+15)=90.
Am obinut c rombul G
1
G
2
G
3
G
4
are un unghi drept, deci este ptrat.
Prin vrfurile unui ptrat ABCD se duc dreptele AE, BF, CT, DL cu E(BC),
F(CD), T(AD), L(AB), astfel nct
m(BAE)=m(DCT)=m(ADL)=m(CBF)=30.
Dac {M}=AEBF, {P}=BFCT, {R}=CTDL, {Q}=DLAE, s se
demonstreze c MPRQ este ptrat.
Demonstraie. Avem m(DAE)=60, m(DTC)=60 i au poziia de unghiuri
corespondente, rezult c
AE||TC (1)
i unghiurile FBA i DLA au msura 60 i deci
LD||BF (2)
Din relaiile (1) i (2) rezult c QMPR este paralelogram avnd laturile opuse
paralele.
Triunghiurile dreptunghice ADL, BFC, CTD, ABE sunt congruente (C.U.).
Atunci obinem c
(AL)(FC)(TD)(BE) (3)
Atunci i
(BL)(DF)(AT)(CE) (4)
Din relaiile (3) i (4) rezult c (QM)(MP)(PR)(QR) (paralele cuprinse
ntre paralele). Atunci paralelogramul QMPR este romb.
n triunghiul FPC, m(FPC)=180-(30+60)=90.
Deci m(RPM)=90. Rombul QMPR avnd un unghi drept este ptrat.
R3.5.7 Se consider triunghiul isoscel ABC cu m(A)=90. Pe laturile (AB)
i (AC) construim spre exterior triunghiurile echilaterale ABD respectiv ACE. S se
demonstreze c patrulaterul BCED este trapez isoscel.
Q
M
P
R T
D
F
C
E
B
L
A
148
Demonstraie. Din ipotez rezult c (AD)(AE), deci ADE este isoscel cu
m(DAE)=360-(90+60+60)=150.
Atunci m(EDA)=m(DEA) (15).
Avem m(EDB)+m(CBD)=(15+60)+(45+60)=180, adic unghiurile
EDB i CBD sunt interne de aceeai parte a secantei suplementare i deci DE||BC,
adic BCED este trapez. Dar (BD)(CE) i atunci trapezul BCED este isoscel.
Pe baza mic [CD] a trapezului isoscel ABCD se construiete ptratul CDMN.
Demonstrai c patrulaterul ABMN este trapez isoscel.
Demonstraie. a) Ptratul este construit spre interior.
Din MN||DC i AB||DC rezult c MN||AB, deci ABNM este trapez. Trapezul
ABCD fiind isoscel are unghiurile alturate unei baze congruente. Deci
ADCBCD, de unde rezult c ADMBCN. Atunci ADMBCN (LUL)
avnd (AD)(BC) (ABCD trapez isoscel), ADMBCN, (DM)(CN) ca laturi ale
ptratului DCNM. Din congruena celor dou triunghiuri rezult c (AM)(BN), adic
trapezul ABNM este isoscel.
b) Ptratul este construit spre exterior.
D E
A
B C
D C
M N
A
B
D C
A
B
M N
149
Trapezul ABCD fiind isoscel are unghiurile alturate unei baze congruente.
Deci
ADCBCD (1)
Atunci m(MDA)=360-[90+m(ADC)] sau
m(MDA)=270-m(ADC) (2)
Analog
m(NCB)=270-m(BCD) (3)
Din relaiile (1), (2) i (3) obinem:
MDANCB (4)
Atunci MDANCB (LUL), avnd (MD)(MC), laturi ale ptratului MNCD,
MDANCB din relaia (4), (AD)(CB) din trapezul isoscel ABCD.
Din congruena celor dou triunghiuri obinem c
(MA)(NB) (5)
Fiindc DC||MN, DC||AB obinem
MN||AB (6)
Din relaiile (5) i (6) rezult c trapezul ABNM este isoscel.


3.6. Concurena liniilor importante n triunghi

Pentru demonstrarea concurenei liniilor importante din triunghi folosim
proprietile patrulaterelor, a liniei mijlocii n triunghi i proprietile de loc geometric
ale unor linii importante din triunghi.
Teorema 3.6.1. n orice triunghi medianele sunt concurente.
Demonstraie. Fie A',B',C' mijloacele laturilor [BC], [CA], [AB] ale
triunghiului ABC, iar G punctul de concuren a medianelor [AA'] i [CC'].
Considerm punctele T i E astfel nct T este mijlocul lui [AG] i E este mijlocul lui
[GC].
Atunci [TE] este linie mijlocie n triunghiul ACG. Avem
ET||AC, AC
2
1
ET = (1)
n triunghiul BCA, [A'C'] este linie mijlocie, rezult c
L
E
T
G
B A' C
C' B'
A
150
A'C'||AC i AC
2
1
C' A' = (2)
Din relaiile (1) i (2) obinem c patrulaterul C'A'ET este paralelogram avnd
dou laturi opuse paralele i congruente. Cum G este punctul de intersecie al
diagonalelor paralelogramului C'A'ET avem relaiile:
AA'
3
1
GT GA' = = i CC'
3
1
GE GC' = = (3)
Fie L mijlocul lui [BG]. Atunci [LT] este linie mijlocie n triunghiul GAB i
putem scrie relaiile:
TL||AB,
2
AB
TL = (4)
n triunghiul CAB, [A'B'] este linie mijlocie, atunci
A'B'||AB, AB
2
1
B' A' = (5)
Din (4) i (5) rezult c patrulaterul A'B'TL este paralelogram i
BB'
3
1
GL GB' = = (6)
Din relaiile (3) i (6) rezult c {G}=AA'BB'CC' i G se gsete pe fiecare
median la
3
1
de baz i la
3
2
de vrf.
Teorema 3.6.2. n orice triunghi bisectoarele interioare sunt concurente.
Demonstraie. Fie [AA
1
i [BB
1
bisectoarele unghiurilor A i B, iar I
punctul lor de intersecie.
Bisectoarele [AA
1
i [BB
1
nu pot fi paralele pentru c ar nsemna c ABB
1
i
BAA
1
ar fi interne de aceeai pare a secantei AB, i suma msurilor lor ar fi 180 ca
i suma msurilor unghiurilor triunghiului ABC, ceea ce este imposibil. Deci [AA
1
i
[BB
1
sunt concurente n I. Punctele de pe bisectoare fiind egal deprtate de laturi avem
relaiile:
(IP)(IT), (IP)(IQ), cu IPAB, ITBC, IQAC i
P(AB), Q(AC), T(BC).
I
B
T A1
C
B1
Q
C1
P
A
151
Din relaiile de mai sus rezult c (IT)(IQ), deci punctul I se gsete i pe
bisectoarea ACB.
Teorema 3.6.3. Mediatoarele laturilor unui triunghi sunt concurente.
Demonstraie. Fie punctele Q i P mijloacele laturilor [AB] i [BC] ale
triunghiului ABC, iar O punctul de intersecie al mediatoarelor laturilor [AB] i [BC].
Aceste dou mediatoare sunt concurente, cci dac ar fi paralele, punctele A, B, C ar fi
coliniare, ceea ce este imposibil fiind vrfurile triunghiului ABC.
Punctele de pe mediatoarea unui segment fiind egal deprtate de extremitile
segmentului; avem relaiile:
(OA)(OB) (OQ fiind mediatoarea laturii AB)
(OB)(OC) (OP fiind mediatoarea laturii BC)
Din relaiile de mai sus obinem (OA)(OC), adic punctul O este situat pe
mediatoarea laturii (AC).
Teorema 3.6.4. nlimile unui triunghi sunt concurente.
Demonstraie. Considerm triunghiul ABC cu nlimile AA', BB', CC'
(AA'BC, BB'AC, CC'AB).
Prin vrfurile triunghiului ABC ducem paralele la laturile opuse care se
intersecteaz dou cte dou n punctele A'', B'', C''. Patrulaterele ABCB'' i C''BCA,
avnd laturile opuse paralele sunt paralelograme, rezult c laturile opuse sunt
congruente, deci (AB'')(BC) i (AC'')(BC), de unde (AB'')(AC''), de unde rezult c
A este mijlocul lui [C''B'']. Deci AA' este mediatoarea lui [C''B''] (1).
Din paralelogramele AB''CB i ACA''B rezult c (AB)(B''C) i (AB)(CA''),
de unde (B''C)(A''C), adic C este mijlocul lui [A''B'']. Atunci CC' este mediatoarea
laturii [A''B''] (2).
Din paralelogramele ACA''B i ACBC'' rezult c (AC)(BA'') i (AC)(C''B),
de unde (C''B)(BA''), adic B este mijlocul lui [C''A'']. Atunci B''B este mediatoarea
laturii [C''A''] (3).
P
A
Q
M
O
B
C
A C''
B''
C
C'
B'
H
A' B
A''
152
Din (1), (2) i (3) rezult c nlimile triunghiului ABC sunt i mediatoarele
laturilor triunghiului A
1
B
1
C
1
. Cum concurena mediatoarelor a fost demonstrat,
rezult c i nlimile sunt concurente.


3.7. Probleme de coliniaritate

Problemele a cror concluzie solicit demonstrarea apartenenei unor puncte la
o aceeai dreapt le vom numi probleme de coliniaritate.
n continuare enumerm cteva dintre procedeele cele mai des ntlnite pentru
soluionarea problemelor de coliniaritate la nivelul clasei a VI-a:
a) Demonstrarea coliniaritii cu ajutorul unghiului alungit (unghiuri
adiacente suplementare)
Dac punctele A i B sunt situate de o parte i de alta a dreptei CD i
m(ACD)+m(DCB)=180, atunci punctele A, C, B sunt coliniare.
b) Demonstrarea coliniaritii folosind reciproca teoremei unghiurilor
opuse la vrf
Dac punctul B este situat pe dreapta EF, iar punctele A i C sunt situate de o
parte i de alta a dreptei EF i ABFCBE, atunci punctele A, B, C sunt coliniare.
c) Demonstrarea coliniaritii prin identificarea unei drepte ce conine
punctele respective
d) Demonstrarea coliniaritii folosind postulatul lui Euclid
Dac dreptele AB i BC sunt paralele cu o dreapt d, atunci n baza
postulatului lui Euclid, punctele A, B, C sunt coliniare.
e) Demonstrarea coliniaritii folosind axioma de construcie a unghiului
Dac B i C sunt n acelai semiplan determinat de dreapta AA' i
A'ABA'AC, atunci A, B, C sunt coliniare.
f) Demonstrarea coliniaritii punctelor A, B, C demonstrnd c
AB+BC=AC
Vom exemplifica procedeele prin probleme rezolvate n continuare.

a) Demonstrarea coliniaritii cu ajutorul unghiului alungit

R3.7.1 Pe laturile consecutive AB i BC ale ptratului ABCD se construiesc
triunghiurile echilaterale ABE i BCF, primul interior i al doilea exterior ptratului. S
se arate c punctele D, E, F sunt coliniare.
Soluie. Fiindc triunghiul ABE este echilateral, m(BAE)=60 i atunci
m(DAE)=90-60=30. Triunghiul ADE este isoscel cu (AD)(AE) i atunci
m(ADE)=m(AED)=(180-30):2=75. Triunghiul EBF este dreptunghic deoarece
m(EBC)+m(CBF)=30+60=90. Fiindc (EB)(BF), triunghiul EBF este
dreptunghic isoscel. Atunci m(BEF)=45. Deci
m(DEA)+m(AEB)+m(BEF)=75+60+45=180, adic punctele D, E, F sunt
coliniare.
153
R3.7.2 S se demonstreze c ntr-un trapez mijloacele laturilor paralele i
intersecia diagonalelor sunt trei puncte coliniare.
Soluie. Fie O punctul de intersecie al diagonalelor AC i BD ale trapezului
ABCD, iar L i T mijloacele bazelor (AB) i (CD). Deoarece AB||DC rezult c
DTLBLT (alterne interne), iar TDBLBD (alterne interne) i atunci obinem c
DOTBOL (1)
Mai avem c AODBOC (opuse la vrf) (2).
i ALTCTL (alterne interne), LACTCA, atunci obinem c
AOLCOT (3)
Cu relaiile (1), (2), (3) n jurul punctului O avem:
2m(AOL)+2m(AOD)+2m(DOT)=360,
de unde
m(AOL)+m(AOD)+m(DOT)=180,
rezult c punctele T, O, L sunt coliniare.
R3.7.3 Pe ipotenuza (BC) a triunghiului dreptunghic ABC se consider un
punct arbitrar D. Fie K i P simetricele lui D fa de AB respectiv AC. S se arate c
punctele K, A, P sunt coliniare.
Soluie. Punctul A se gsete pe mediatoarea segmentului [DK], el va fi egal
deprtat de capetele segmentului. Deci (AK)(AD). n triunghiul ADK, nlimea AB
este i bisectoare, deci
KABBAD (1)
D
A
B
C
F
A L
B
O
D T C
154
Punctul A se gsete i pe mediatoarea [DP], rezult c (AD)(AP). n
triunghiul isoscel APD, nlimea AC este i bisectoare, deci
PACCAD (2)
Din relaiile (1) i (2) obinem:
m(KAB)+m(BAC)+m(CAP)=2m(BAC)=180
deci punctele K, A, P sunt coliniare.

b) Demonstrarea coliniaritii folosind reciproca teoremei unghiurilor opuse la
vrf

R3.7.4 Se consider patrulaterul ABCD cu E mijlocul lui [AB] i R mijlocul
lui [CD]. Prin E se duc EF paralel la BC i EQ paralel la AD, iar prin vrfurile C i D
se duce cte o paralel la AB. Obinem paralelogramul BCFE i AEQD. S se arate c
vrfurile F i Q ale acestor paralelograme sunt coliniare cu R.
Demonstraie. BCFE este paralelogram deci
[CF][BE] (1)
Din paralelogramul AEQD obinem
[EA][QD] (2)
i
[BE][EA] (3)
deoarece E este mijlocul lui AB.
Din (1), (2) i (3) rezult c
[CF][QD] (4)
Punctul R este mijlocul lui [CD], deci
[CR][DR] (5)
Mai avem c CF||BA||QD, deci CF||QD. atunci
FCRQDR (6)
Din (4), (5), (6) obinem c triunghiurile CFR i DQR sunt congruente (LUL).
Din congruena celor dou triunghiuri obinem c CRFDRQ i cum F i Q sunt de
o parte i de alta a dreptei CD, deci punctele C, R, Q sunt coliniare.
P
A
K
B D C
155
c) Demonstrarea coliniaritii prin identificarea unei drepte ce conine punctele
respective
R3.7.5 n triunghiul ABC cu m(B)=2m(C), notm cu I intersecia
bisectoarelor [BB' i [AA', B'(AC), A'(BC). Perpendiculara din B pe AA'
intersecteaz perpendiculara din B' pe BC n D. S se demonstreze c punctele I, D, C
sunt coliniare.
Demonstraie. Din m(B)=2m(C) rezult m(A'BB')=m(C), deci
triunghiul B'BC este isoscel i atunci [B'D va fi bisectoarea BB'C. Unghiul BA'A
este exterior triunghoului AA'C i deci
|
.
|

\
|
+ =
2
A
m C) m( A) BA' m( (1)
Unghiul BIA' este exterior triunghiului ABI. Deci
|
.
|

\
|
+ = + =
2
A
m C) m( IAB) m( IBA) m( ) BIA' m( (2)
Din (1) i (2) rezult c triunghiul BIA' este isoscel. Atunci nlimea BD a
triunghiului IBA' este i bisectoare. Deci punctul D este punctul de intersecie a
bisectoarelor [BD i [B'D ale unghiurilor triunghiului B'BC. Fiindc [CI este
bisectoarea unghiului C ea conine i punctul D, deci punctele C, D, I sunt coliniare.
R3.7.6 Se consider triunghiul ascuitunghic ABC, iar AD bisectoarea
unghiului BAC, cu D(BC). Fie L i K simetricul lui D fa de AB, respectiv AC, iar
Q intersecia paralelei prin L la AC cu paralela prin K la AB. Artai c punctele A, D,
Q sunt coliniare.
Soluie. Fiindc LDAB i AB||QK, rezult c LDQK sau
C
B
E
A
D
Q
R
F
A
B'
D
C L A' B
I
156
LRQK (1)
RKQ i KDAC, LQ||AC rezult c KDLQ sau
KPLQ (2)
PLQ.
Din (1) i (2) rezult c punctul D este ortocentrul triunghiului LKQ, de unde
QDLK.
Fie {S}=ARAB i {T}=KPAC. Din congruena triunghiurilor dreptunghice
ADS i ADT obinem c (DS)(DT) sau 2DS=2DT, care se mai scrie DL=DK.
Deci triunghiul LDK este isoscel iar [DA este bisectoare (din ADSADT),
ea va fi i mediatoarea lui (LK). Deci DALK. Din unicitatea perpendicularei din D pe
(LK) rezult c punctele A, D, Q sunt coliniare.
R3.7.7 Fie trapezul ABCD cu AD||BC. Bisectoarele interioare ale unghiurilor
A i B se intersecteaz n L, iar bisectoarele interioare din C i D se intersecteaz n Q.
Dac T este mijlocul diagonalei (AC). S se arate c punctele L, T, Q sunt coliniare.
Demonstraie. Fiindc DA||CB rezult c CBABAF (alterne interne).
dar m(DAB)+m(BAF)=180 =
|
.
|

\
|
+ 90
2
CBA
m BAL) m(
m(BAL)+m(LBA)=90. Deci (ALB)=90, adic ALB este dreptunghic n L. Fie
R mijlocul lui (BA). Atunci LR este median n triunghiul dreptunghic BLA, deci
2
AB
LR = . Obinem astfel c LRA este isoscel, de unde RLALARDAL, de
unde obinem c RL||AD. Fie F mijlocul lui [CD], atunci [QF] este median n
triunghiul dreptunghic CQD. Obinem
2
CD
QF = , adic triunghiul QFD este isoscel
cu FQDFDQQDA, de unde rezult c FQ||DA. [FR] este linie mijlocie n
trapez, deci TR||AD. Deci linia mijlocie a trapezului conine punctele Q, L i T (TR
este linie mijlocie n ABC, RT||BC).
S
A
L
K
T
B
P
R
Q
C
D
157

d) Demonstrarea coliniaritii folosind postulatul lui Euclid

R3.7.8 n triunghiul ABC, punctele B' i C' sunt mijloacele segmentelor [AC],
respectiv [AB]. Dac L este simetricul punctului B fa de B' i T este simetricul
punctului C fa de C', s se demonstreze c punctele T, A, L sunt coliniare.
Demonstraie. n patrulaterul ACBT, diagonalele [AB] i [TC] se taie n pri
congruente, rezult c ACBT este paralelogram. Deci
AT||BC (1)
i diagonalele patrulaterului ABCL se mjumtesc i deci ABCL este
paralelogram. Atunci
AL||BC (2)
Din (1) i (2) innd seama c prin A trece numai o singur paralel la BC
obinem c punctele T, A, L sunt coliniare.
R3.7.9 Se d un triunghi ABC. S se arate c mijloacele laturilor AB, BC i
proiecia vrfului B pe bisectoarea unghiului A sunt trei puncte coliniare.
Demonstraie. Fie T mijlocul lui [AB] i Q mijlocul lui [BC]. Fie BLAE,
L(AE). Atunci triunghiul ABL este dreptunghic, iar TL este median,
corespunztoare ipotenuzei [AB], deci AT
2
AB
TL = = . Obinem astfel c triunghiul
ATL este isoscel cu (TA)(TL), deci
TALTLA (1)
Dar
CALLAT (2)
Din (1) i (2) rezult c CALALT, deci
AC||TL (3)
n triunghiul ABC, [TQ] este linie mijlocie i atunci
C
B
F
Q
D A
F
R
T
L
T
A
L
C' B'
B C
158
AC||TQ (4)
Din (3) i (4), innd seama c prin T trece o
singur paralel la AC obinem c punctele T, L, Q
sunt coliniare.
Pe latura [BC] a triunghiului ABC se
consider un punct D astfel nct BC=3DC. Dac R
este mijlocul medianei CM, M(AB), s se arate c
punctele A, R, D sunt coliniare.
Soluie. Dac T este mijlocul lui BD, atunci
[MT] este linie mijlocie n triunghiul ABD i deci
AD||MT (1)
n triunghiul CMT, [RD] este linie mijlocie, rezult c
RD||MT (2)
Din (1) i (2) i faptul c prin punctul R trece numai o singur paralel la MT,
rezult c punctele A, R, D sunt coliniare.

3.8. Probleme de concuren

Vom numi problem de concuren o problem de geometrie a crei concluzie
cere demonstrarea faptului c trei sau mai multe drepte (cercuri) au ca intersecie
acelai punct.
Dintre procedeele mai des utilizate pentru demonstrarea concurenei (la nivelul
clasei a VI-a) amintim:
a) Demonstrm c punctul de intersecie a dou dintre drepte aparine i celei
de-a treia dreapt. (Trebuie demonstrat c punctul de intersecie a dou dintre drepte
exist, iar apartenena lui la cea de-a treia dreapt se demonstreaz de obicei artnd c
punctele acesteia sunt caracterizate de o anumit proprietate specific, pe care o are i
punctul obinut ca intersecie a celor dou drepte.)
b) Demonstrarea concurenei a trei drepte prin identificarea acestora cu trei
ceviene remarcabile concurente (bisectoare, nlimi, mediane, etc.) dintr-un triunghi
din configuraia problemei.
c) Demonstrarea concurenei prin coliniaritate.
Vom exemplifica procedeele enunate mai sus n rezolvarea unor probleme.
B T D C
R
M
A

B E Q C
L
T
A
159
Probleme rezolvate

R3.8.1 Fie M un punct n interiorul triunghiului ABC. Dac D, E, T sunt
simetricele punctului M fa de mijloacele laturilor [BC], [CA], [AB], s se
demonstreze c dreptele AD, BE, CT sunt concurente.
Demonstraie. Fie G, H, K mijloacele laturilor [AB], [AC], [BC]. Diagonalele
patrulaterului AMBT se taie n pri congruente, rezult c el este paralelogram, deci
AT||MB, (AT)(MB) (1)
Analog patrulaterul MCDB este paralelogram fiindc diagonalele lui se taie n
pri congruente. Atunci
MB||CD, (MB)(CD) (2)
i
BD||MC, [BD][MC] (3)
Din (1) i (2) obinem c patrulaterul ATDC este paralelogram, avnd 2 laturi
opuse paralele i congruente. Diagonalele lui se intersecteaz n O. Diagonalele
patrulaterului AMCE se taie n pri congruente i atunci el este paralelogram, deci
AE||MC, (AE)(MC) (4)
Din relaiile (3) i (4) rezult c AE||BD i (AE)(BD), atunci ABDE este
paralelogram i BE intersecteaz pe AD n O. Obinem astfel c dreptele AD, BE, CT
sunt concurente n O.
R3.8.2 Pe diagonala (BD) a paralelogramului ABCD se consider punctele E
i M astfel nct (BE)(EM)(MD). Dac {T}=BCAE, {Q}=AMCD,
{L}=ABCE i {P}=ADCM, s se demonstreze c dreptele AC, EM i LQ sunt
concurente.
Demonstraie. Triunghiurile ADM i CBE sunt congruente avnd (AD)(BC),
(ADM)(CBE) (alterne interne), (DM)(BE). Din congruena acestor dou
triunghiuri obinem c
(AM)(CE) (1)
i triunghiurile ADE i CBM sunt congruente avnd (AD)(BC),
(ADE)(CBM) i (DE)(BM). Din congruena lor obinem c
A
T
B
M
G
H
E
C
K
O
D
160
(AE)(CM) (2)
Din relaiile (1) i (2) rezult c patrulaterul AECM, avnd laturile opuse
congruente este paralelogram, i deci (EM) trece prin mijlocul O al diagonalei (AC).
Patrulaterul ALCQ, avnd laturile opuse paralele este paralelogram (AQ||LC din faptul
c AECM este paralelogram). Deci diagonala (LQ) trece prin mijlocul O al diagonalei
(AC). Atunci dreptele AC, EM, LQ sunt concurente.
Dreptele care trebuie s demonstrm c sunt concurente sunt mediane,
bisectoare, nlimi, mediatoare, etc. pentru un anumit triunghi.
R3.8.3 Se consider triunghiul ABC dreptunghic n A i AD nlime,
D(BC). n C, pe cateta AC se ridic o perpendicular pe care se consider un segment
(CM)(AC), iar n B pe cateta AB se construiete o perpendicular pe care se ia un
segment BP cu (BP)(AB). S se demonstreze c dreptele BM, CP i AD sunt
concurente.
Soluie. Prelungim nlimea DA cu un segment (AR)(BC). Avem
m(RAC)=180=m(DAC)=180-[90-m(ACD)]=
=90+m(ACD)=m(BCM)
Atunci triunghiurile RAC i BCM sunt congruente fiindc au (RA)(BC),
m(RAC)m(BCM), (AC)(MC). Din congruena celor dou triunghiuri rezult c
(RCA)(CMB).
Fie {Q}=BMRC. Atunci
m(CQM)=180-[m(QCM)+m(CMQ)]=
=180-[m(QCM)+m(RCA)]=180-90=90
A
L
B
T
E
O
C
Q
M
P D
B D C
P
T
A
Q
M
R
161
Deci BQRC, adic BQ este nlime n triunghiul RBC.
Analog artm congruena triunghiurilor RAB i CBP. Avem
m(RAB)=180-m(DAB)=180-(90-m(ABD))=
=90+m(ABD)=m(PBD)
Deci RABCBP avnd (RA)(BC), RABPBD i (AB)(BP).
Din congruena acestor triunghiuri obinem: (ABR)(BPC).
Fie {T}=PCAB. Avem
m(PTB)=180-[m(TPB)+m(PBT)]=
=180-[m(ABR)+m(PBT)]=180-90=90
Deci PTBR, adic CT este nlime n triunghiul RBC. Deci dreptele CT, BQ,
AD sunt concurente fiind nlimi n triunghiul RBC.
R3.8.4 Fie M un punct pe diagonala [BD] a ptratului ABCD. S se
demonstreze c perpendicularele duse din M, B, D respectiv pe AB, AM, CM se
ntlnesc pe diagonala [AC].
Soluie. Fie L i K picioarele perpendicularelor duse din B pe MA i respectiv
din M pe AB. n triunghiul MAB nlimile MK i BL se intersecteaz n P.
Diagonalele ptratului sunt perpendiculare, deci AO este nlime n triunghiul MAB,
deci perpendicularele PK i BL din M pe AB, respectiv B pe MA se ntlnesc pe
diagonala (AC). Fiindc MPAB, rezult c MPDC. Dar DMPC, i atunci M este
ortocentrul triunghiului DCP, rezult c CMDP. Astfel perpendicularele din M, B, D
respectiv pe AB, AM, CM se ntlnesc pe (AC).

3.9. Construcii geometrice

Prin probleme de construcie vom nelege acele probleme de geometrie n care
se cere construirea unor figuri geometrice ce satisfac anumite proprieti, folosind
numai rigla i compasul.
nainte de a considera probleme de construcie cu rigla i compasul, Edwin
Moise n lucrarea "Geometrie elementar" face cteva precizri:
i) Cnd vorbim de rigl i compas, nelegem o "rigl ideal" i un "compas
ideal", care traseaz liniile drepte i cercurile exact.
A K B
C D
M
T
L
P
O
162
ii) Rigla nu are un marcaj pe ea. O putem utiliza pentru a desena drepte ntre
dou puncte date, dar aceasta este tot ceea ce putem face cu ea. Nu o putem utiliza
pentru a msura distanele dintre puncte sau pentru a vedea dac dou segmente sunt
congruente.
iii) Compasul se poate utiliza astfel. Fie un punct P i un punct Q n plan.
Putem desena atunci cercul cu centrul n P i care trece prin Q. Aceasta este tot ce
putem face cu el. Altfel spus, dndu-se un al treilea punct P' nu este permis s mutm
vrful compasului n P' i apoi s desenm un cerc cu centrul n P' i de raz PQ. Din
acest motiv compasul este numit nerigid; nu i se poate muta vrful deoarece "cnd
ridici vrful de pe hrtie, compasul se nchide".
S reamintim cteva dintre construciile elementare studiate i justificate la
clas.

3.9.1. Construcia mediatoarei unui segment i a mijlocului su

Considerm un segment [AB].
1) Cu o raz mai mare dect jumtate din lungimea segmentului [AB] desenm
dou arce de cerc cu centrul n A deoparte i de cealalt a dreptei AB.
2) Cu aceeai raz ca mai sus desenm dou arce de cerc cu centrul n B, de o
parte i de cealalt a dreptei AB, care intersecteaz arcele cu centrul n A n M i N.
3) Dreapta MN este mediatoarea segmentului [AB]. Punctul de intersecie P,
dintre AB i MN este mijlocul segmentului [AB].


3.9.2. Construcia bisectoarei unui unghi dat XOY

Considerm un punct O drept centru i desenm un arc de cerc care va
intersecta laturile unghiului n A i B. Cu aceeai raz mai mare dect AB
2
1
desenm
un arc de cerc cu centrul n A i unul cu centrul n B. Ce dou arce se intersecteaz n
M. Semidreapta [OM este bisectoarea unghiului XOY.
M
P
A B
N
163


3.9.3. Construcia perpendicularei pe o dreapt dintr-un punct al ei

Considerm punctul O pe dreapta a. Cu o deschidere oarecare a compasului se
construiesc dou arce ale unui cerc cu centrul n O i care intersecteaz dreapta a n A
i B. Perpendiculara cutat este mediatoarea segmentului AB.

3.9.4. Construcia perpendicularei pe o dreapt dintr-un punct exterior al ei

Considerm o dreapt a i un punct O nesituat pe ea. Desenm un arc de cerc
cu centrul n O i cu raza mai mare dect distana de la O la a, care intersecteaz pe a n
A i B. Mai construim un punct C egal deprtat de A i B i atunci OCa.

3.9.5. Construcia unui unghi congruent cu un unghi dat

Fiind dat unghiul XOY, trebuie s construim un unghi congruent cu el
X'O'Y'.
Cu centrele n O i O' desenm dou arce de cerc i obinem punctele A, B, C.
Descriem un arc de cerc cu centrul n C i raza AB. Obinem astfel punctul D. Latura
[O'Y' este semidreapta [O'D.
O
B
A
M
X
Y
O
A B
a
C
164
3.9.6. Construcia cercului circumscris triunghiul ABC

Se construiesc mediatoarele a dou laturi ale triunghiului. Intersecia lor, O, va
fi centrul cercului circumscris. Raza R va fi OA.



3.9.7. Construcia cercului nscris n triunghiul ABC

Se construiesc bisectoarele a dou unghiuri. Intersecia lor, I, va fi centrul
cercului nscris. Raza, r, va fi lungimea unei perpendiculare duse din I pe o latur.

3.9.8. Construcia centrului de greutate G

Construim mijloacele a dou laturi i apoi medianele corespunztoare.

D
C
O' X'
Y
Y'
B
A
O X'
Y

O
A
B C
A
B C
r
I
165
3.9.9. Construcia ortocentrului H

Se construiesc perpendicularele din dou vrfuri ale triunghiului pe laturile
opuse.

3.9.10. Construcia tangentei la cercul C(O,R), ntr-un punct A al cercului

Se construiete perpendiculara CD pe raza OA.

3.9.11. Construcia tangentelor la cercul C(O,R) dintr-un punct exterior M

Se determin mijlocul segmentului [OM], O
1
, i apoi se construiete cercul de
diametru [OM]. Cercul astfel construit va intersecta cercului C(O,R) n A i B.
Tangentele cutate sunt MA i MB.


3.9.12. Rezolvarea problemelor de construcii geometrice

n rezolvarea problemelor de construcii geometrice parcurgem urmtoarele
etape:
1) Analiza (gsirea soluiei). Se consider figura construit i identificm
elementele construibile folosind construciile elementare.
2) Construcia. Se prezint succesiunea elementelor care se construiesc.
O
C
A
D
O1
O
B
A
M
166
3) Demonstraia. Conine argumentarea faptului c elementele construite
satisfac cerinele enunate n problem.
4) Discuia. Se comenteaz existena soluiilor i numrul lor.


Probleme rezolvate

R3.9.1 S se construiasc un triunghi dreptunghic cunoscnd suma lungimilor
catetelor (b+c) i lungimea ipotenuzei (a).
Demonstraie. Considerm problema rezolvat i fie BAC triunghiul cutat.
Construim triunghiul dreptunghic isoscel DAC cu (AC)(AD).
Analizm triunghiul BCD i deducem modul de construcie.
Construcia figurii
Pe segmentul DC=c+b (suma lungimilor catetelor), n punctul D, construim un
unghi de 45. Din B se duce un arc de cerc cu raza ct lungimea ipotenuzei a.
Triunghiul cerut este ABC.
Discuie. Dac arcul de cerc taie pe DF ntr-un singur punct avem soluie, dac
taie pe DF n dou puncte, avem dou soluii i dac nu-l taie n nici un punct nu avem
soluie.
R3.9.2 S se construiasc un triunghi cunoscnd lungimile medianelor sale.
B
A
D
C
D A B
C
F
45
o
167
Demonstraie. Fie G
1
simetricul centrului de greutate G al triunghiului ABC
fa de mijlocul A' al laturii [BC]. Atunci
a a 1
m
3
2
m
3
1
2 GA' 2 GG = = =
b 1 c
m
3
2
GB C G , m
3
2
GC = = =
Construim triunghiul GG
1
C de laturi
a 1
m
3
2
GG = ,
c
m
3
2
GC = i
b 1
m
3
2
C G =
Simetricul lui G
1
fa de G ne d vrful A. Simetricul lui C fa de mijlocul lui
[GG
1
] ne d vrful B.
Problema este posibil numai dac m
a
, m
b
, m
c
satisfac relaiile de condiie
dintre laturile unui triunghi.
R3.9.3 S se construiasc un triunghi dreptunghic cnd se cunosc nlimea i
mediana ce pleac din vrful unghiului drept.
Presupunem problema rezolvat. n triunghiul ABC cunoatem nlimea AD
i mediana AA'.
Construcia
Construim triunghiul dreptunghic ADA' n care cunoatem cateta (AD) i
ipotenuza (AA'). Fiindc ntr-un triunghi dreptunghic mediana ce pleac din vrful
unghiului drept are lungimea egal cu jumtate din lungimea ipotenuzei, rezult c
A
B
G
1
C
A'
G
B D A'
C
A
168
putem construi segmentele (A'B) i (A'C) fiecare congruente cu AA', obinem astfel
vrfurile B i C i problema este rezolvat.
R3.9.4 S se construiasc un triunghi ABC cunoscnd mijloacele B', C' ale
laturilor [AC] i [AB] precum i piciorul D al nlimii care cade din vrful A pe [BC].
Demonstraie. Presupunem problema rezolvat i fie ABC triunghiul cutat. n
acest triunghi cunoatem punctul D piciorul nlimii din A, punctele B' i C' mijloacele
laturilor (AC) respectiv (AB).
Analiznd figura deducem c A este simetricul lui D fa de linia mijlocie
[B'C'].
Construcia
Unim C' cu B'. Construim punctul A ca simetricul lui D fa de [B'C']. Ducem
n D o perpendicular pe AD. Dreapta AC' ntlnete perpendiculara n B. Dreapta AB'
ntlnete perpendiculara n C. Triunghiul ABC astfel construit corespunde ipotezei.


3.9.13. Probleme de construcii imposibile ale antichitii

Problemele de construcie cu rigla i compasul au preocupat matematicienii
Greciei antice, care au dat soluii la unele, dar la altele, pentru a le putea aborda era
nevoie de ramuri ale matematicii pe care grecii nu le descoperiser.
Dintre problemele de construcii imposibile ale antichitii poate cele mai
celebre ar fi: problema triseciunii unghiului, duplicarea cubului, cuadratura cercului.
i) Triseciunea unghiului
Dndu-se un unghi BAC se cere s construim semidreptele AD i AE cu D,
E situate n interiorul unghiului BAC astfel nct s avem:
m(BAD)=m(DAE)=m(EAC), folosind numai rigla i compasul.
B D C
C'
B'
A
B
A
D
E
C
169
Problema triseciunii unghiului este imposibil. (Exist unele unghiuri pentru
care semidreptele care realizeaz triseciunea nu pot fi construite. Sunt unele unghiuri
care pot fi mprite n trei pri congruente, exemplu unghiul drept.)
Problema triseciunii unghiului devine posibil dac schimbm puin ipoteza,
n sensul c permitem ca pe rigl s putem face dou gradaii.
ii) Duplicarea cubului
Fiind dat un segment AB, dorim s construim un segment CD astfel nct
cubul cu muchia CD s aib volumul exact de dou ori mai mare dect volumul
cubului cu muchia AB, adic CD
3
=2AB
3
.
iii) Cuadratura cercului
Dndu-se un cerc, dorim s construim un ptrat care s aib aceeai arie ca
cercul, adic
2 2
R l = , adic = R l .
Problemele de mai sus au fost studiate peste dou mii de ani, descoperindu-se
n final c sunt imposibile. (Demonstraiile depesc cadrul de fa.)



Bibliografie

[1] I. I. Alexandrov, Probleme de construcii geometrice, Ed. Tehnic, 1951.
[2] G. Buicliu, Probleme de construcii geometrice cu rigla i compasul,
Ed. Tineretului, 1967.
[3] I. Dncil, Construcii cu rigla i compasul, Ed. Sigma, 2000.
[4] I. Petersen, Methodes et theories pour la resolution des problemes de
constructions geometriques. Paris, 1908.
[5] A. Toth, Noiuni de teoria construciilor geometrice, E.D.P., 1963.

Você também pode gostar